Top 400 Current Affairs MCQS With Explanation Do You Success Telegram Channel

You might also like

Download as pdf or txt
Download as pdf or txt
You are on page 1of 242

1. Consider the following statements with reference to Konark Wheel.

1. It was built during the 10th century under the reign of King Chmapa -2.
2. The wheel with 24 spokes representing the wheels of Lord Surya's sun chariot.
3. Konark Sun temple's architecture is an excellent example of the Vesara style of
architecture.
Which of the above given Statements is/are correct ?
A. 2 Only B. 2 and 3 Only
C. 1 and 3 Only D. 1, 2 and 3
Explanation : A
● A replica of Konark Wheel from the Sun temple in Odisha served as the backdrop of Prime
Minister Shri. Narendra Modi's welcome handshake with G20 leaders.
Konark Wheel
● Konark Wheel was built during the 13th century under the reign of King Narasimhadeva-I.
● The wheel with 24 spokes representing the wheels of Lord Surya's sun chariot.
● The wheel has also been adapted into the Indian National Flag.
● Its rotating motion symbolises time, Kaalchakra, as well as progress and continuous change.

Konark Sun Temple


● It was built c. 1250 CE by King Narasimhadeva I (1238-1264 CE) of the Eastern Ganga
dynasty.
● It is a UNESCO World Heritage Site known for its exquisite architecture and intricate stone
carvings.
● The temple is in the shape of a chariot, with 12 exquisitely carved wheels and galloping
horses.
● The temple's architecture is an excellent example of the Kalinga style of architecture (subset
of the nagara style).

2. Consider the following statements regarding Bubble Of Galaxies.


1. It is an unimaginably huge cosmic structure measuring one billion light-years.
2. The heart of this galaxy is surrounded by a vast void sometimes called "the Great
Nothing".
3. The bubble contains several other galaxy superclusters like Sloan Great Wall.

AMIGOS IAS 1
Which of the above given Statements is/are correct ?
A. 1 and 2 Only B. 2 and 3 Only
C. 1 Only D. 1, 2 and 3
Explanation : D
● Astronomers have discovered the first "bubble of galaxies,” recently.
● Bubble of galaxy is an unimaginably huge cosmic structure measuring one billion light-
years across that is thought to be a fossilised remnant from just after the Big Bang.
● The bubble spans 820 million light years away from the milkyway.
● The heart of the bubble is the Bootes supercluster of galaxies, which is surrounded by a
vast void sometimes called "the Great Nothing".
● The bubble contains several other galaxy superclusters, including the massive structure
known as the Sloan Great Wall.

3. Consider the following statements regarding Incremental Cash Reserve Ratio (I-CRR).
1. It is an additional cash balance that RBI can ask banks to maintain over and above CRR for
a specific period.
2. RBI has recently increased the percentage of (I-CRR).
Which of the above given Statements is/are correct ?
A. 1 Only B. 2 Only
C. Both 1 and 2 D. Neither 1 nor 2
Explanation : A
I-CRR
● I-CRR is an additional cash balance that RBI can ask banks to maintain over and above CRR
for a specific period.
● CRR is the minimum amount of total deposits that banks have to keep with the central
bank.
● Earlier RBI mandated banks to set aside an additional 10% of net demand and time
liabilities to absorb surplus liquidity.
● Surplus was caused by the return of ₹ 2,000 banknotes to the banking system, surplus
transfer to the government by the RBI, etc.

4. Consider the following statements regarding a Wildlife Sanctuary that has been in the news.
1. It is spread over three districts-Chamarajanagar, Mandya and Ramanagara of Karnataka.
2. It provides a vital link between Bannerghatta National Park in the north and BRT Tiger
Reserve and Male Mahadeshwara Hills Wildlife Sanctuary in the south.
3. Animals found in sanctuary are tiger, elephant, leopard, bison, wild dog, sambar, spotted
deer, etc.
Choose the correct answer from the below given Codes:

AMIGOS IAS 2
A. Brahmagiri Wildlife Sanctuary B. Cauvery Wildlife Sanctuary
C. Dandeli Wildlife Sanctuary D. Jogimatti Wildlife Sanctuary
Explanation : B
● Researchers discovered ‘white’ sambar in Cauvery Wildlife Sanctuary.
● Leucism is a condition in which the pigmentation of an animal’s skin is missing, causing
white or pale skin.
Cauvery Wildlife Sanctuary
● Cauvery Wildlife Sanctuary spread over three districts-Chamarajanagar, Mandya and
Ramanagara of Karnataka.
● It provides a vital link between Bannerghatta National Park in north and Biligiri
Rangaswamy
● Temple Tiger Reserve and Male Mahadeshwara Hills Wildlife Sanctuary in the south.
Forest is primarily of dry deciduous and scrub types.
● Animals found in sanctuary are tiger, elephant, leopard, bison, wild dog, sambar, spotted
deer, etc.
5. Consider the following statements regarding the Food Price Index (FFPI) released recently.
1. It is released by the Food for Poor an NGO
2. It is a measure of the monthly change in international prices of a basket of food
commodities.
3. According to FFPI, the World Food Price Index rose to a two year high.
Which of the above given Statements is/are correct ?
A. 1 and 2 Only B. 2 and 3 Only
C. 3 Only D. 2 Only
Explanation : D
● According to FFPI, the World Food Price Index fell at two years low.
FPI
● FPI, released by Food and Agriculture Organization (FAO) is a measure of the monthly
change in international prices of a basket of food commodities.
● It consists of the average of five commodity group price indices - cereals, oilseeds, dairy
products, meat and sugar, weighted with the average export shares of each of the groups.
● FAO is a specialized agency of the United Nations that leads international efforts to defeat
hunger.

6. The report named “Working without borders: the Promise And Peril Of Online Gig work” has
been recently released by ?
A. Asian Development Bank B. World Bank
C. International Labour Organisation D. World Economic Forum
Explanation : B

AMIGOS IAS 3
● WORLD BANK recently released “Working Without borders: The Promise and Peril Of
Online Gig Work” Report.
Key findings
● Online gig workers constitute about 4.4 to 12.5 percent of global workforce.
● Most online gig work platforms are regional/ local which attracts only 29% of overall
traffic.
● Globally, women participate in online gig work to a greater extent than in general labor
market.
● However, potential of online gig work to support female labor force participation has not
been fully tapped.
● Demand for online gig workers is growing faster in developing nations than in
industrialized ones.
Note
● Gig workers are independent contractors or freelancers who typically do short-term work
for multiple clients. There are two types of platform-based gig jobs.
7. Consider the following statements regarding Payments Council of India (PCI).
1. It is a body representing over 85% of the non-bank companies in the Payments ecosystem.
2. It aims to encourage the growth of non-banking payment systems for ushering in a " less-
cash society " in India.
3. The Global Fintech Fest 2023 was recently organised by PCI.
Which of the above given Statements is/are correct ?
A. 1 and 2 Only B. 3 Only
C. 2 and 3 Only D. 1,2 and 3
Explanation : D
● The Global Fintech Fest 2023 was recently organized by the National Payments
Corporation of India (NPCI), Payments Council of India (PCI) and Fintech Convergence
Council (FCC).

Payments Council of India (PCI):


● Payments Council of India (PCI) is a body representing over 85% of the non-bank
companies in the Payments ecosystem and was formed to effectively cater to the needs
of the digital payments industry.
● The objective of PCI is to identify and build opportunities to address and help resolve
industry-level issues and barriers that require industry-level discussion and action,
andproactively encourage the growth of non-banking payment systems for ushering in a
" less-cash society " in India.
● The council works with all its members to promote payments industry growth and scale
the vision of the payments ecosystem in India.

AMIGOS IAS 4
8. Consider the following statements regarding Idukki Dam.
1. It is a double curvature Arch dam constructed across the Cauvery River
2. It is one of the highest arch dams in Asia and third tallest arch dam.
3. The dam stands between the two mountains – Kuravanmala and Kurathimala.
Which of the above given Statements is/are correct ?
A. 2 Only B. 3 Only
C. 2 and 3 Only D. 1 and 2 Only
Explanation : C

Major security breach reported from Idukki dam.


Idukki dam
● It is a double curvature Arch dam constructed across the Periyar River in a narrow gorge
between two granite hills locally known as Kuravan and Kurathi in Mariyapuram village in
Idukki District in Kerala, India.
● It is one of the highest arch dams in Asia and third tallest arch dam.
● The dam stands between the two mountains – Kuravanmala and Kurathimala.

9. Consider the following statements regarding India-Middle East-Europe Economic corridor (IMEC).
1. The IMEC comprises of an Northern Corridor connecting India to the Gulf region
2. The project includes India, UAE, Saudi Arabia, European Union, France, Italy, Germany
and the US.
3. It will include a railway and ship-rail transit network and road transport routes.
Which of the above given Statements is/are correct ?
A. 2 Only B. 1 Only C. 3 Only D. 2 and 3 Only
Explanation : D
• Recently, the Prime Minister of India announced the launch of the India-Middle East-Europe
mega economic corridor.
India-Middle East-Europe Economic Corridor:
● The project includes India, the UAE, Saudi Arabia, the European Union, France, Italy,
Germany and the US.
● The project will aim to enable greater trade among the involved countries, including
energy products.
● The IMEC comprises of an Eastern Corridor connecting India to the Gulf region and a
Northern Corridor connecting the Gulf region to Europe.
● It will include a railway and ship-rail transit network and road transport routes.
● The rail and shipping corridor is part of the Partnership for Global Infrastructure
Investment (PGII).

AMIGOS IAS 5
10. Consider the following statements with reference to Global Biofuels Alliance.
1. It is an India-led Initiative which includes Governments, International organisations and
Industry to facilitate the adoption of biofuels.
2. A total of 190 countries and 50 international organisations have so far agreed to join the
alliance.
3. India is the Only founding member of the alliance.
Which of the above given Statements is/are correct ?
A. 1 and 2 Only B. 2 and 3 Only
C. 1 Only D. 3 Only
Explanation : C
• Recently, the Prime Minister of India announced the launch of the Global Biofuels Alliance.
Global Biofuels Alliance:
● It is an India-led Initiative to develop an alliance of Governments, International
organisations and Industry to facilitate the adoption of biofuels.
● A total of 19 countries and 12 international organisations have so far agreed to join the
alliance, including both G20 members and non-member countries.
● India, Brazil and the US is the founding members of the alliance.
● This Alliance will be aimed at facilitating cooperation and intensifying the use of
sustainable biofuels, including in the transportation sector.
11. Consider the following statements with reference to Nataraja Sculpture.
1. Shiva was first portrayed in sculpture as Nataraja in the 5th century AD.
2. The present form of Nataraja sculpture evolved under the Hoyasalas.
3. Traditional ‘lost-wax’ casting method was used to make these idols.
Which of the above given Statements is/are correct ?
A. 1 and 2 Only B. 3 Only
C. 2 Only D. 1 and 3 Only
Explanation : D
● The World’s tallest 27-foot Nataraja sculpture has been installed at Bharat Mandapam,
venue of the G20 Leaders' Summit.
● Although Shiva was first portrayed in sculpture as Nataraja in the 5th century AD, its
present form evolved under the Cholas.
The design draws inspiration from 3 revered Nataraja idols
● The Thillai Nataraja Temple in Chidambaram,
● The Uma Maheswarar Temple in Konerirajapuram, and
● The Brihadeeswara (Big) Temple, a UNESCO World Heritage Site, in Thanjavur.
● The crafting process adopted was the traditional ‘lost-wax’ casting method, indigenous to
the Chola era.

AMIGOS IAS 6
12. Consider the following Pairs regarding the Types of dark patterns.
Dark Patterns Purpose
1. Drip pricing The practice whereby-elements of prices
are not revealed upfront
2. Nagging The practice in which users face an overload
of requests,options, or interruptions.
3. False urgency Falsely stating or implying the sense of
urgency
4. Interface Interference The practice of advertising a particular
outcome based on the user’s action
5. Bait and Switch The design element that manipulates the user
interface
How many of the above given pairs is /are matched correctly ?
A. Only four B. Only two
C. Only three D. All five
Explanation : C
● The Department of Consumer Affairs seeks public comments on Draft Guidelines for
Prevention and Regulation of Dark Patterns recently.
● A dark pattern is a user interface that has been carefully crafted to trick users into doing
things, such as buying overpriced insurance with their purchase or signing up for recurring
bills.
False urgency
● Falsely stating or implying the sense of urgency.
Interface Interference
● The design element that manipulates the user interface.
Bait and Switch
● The practice of advertising a particular outcome based on the user’s action.
Drip pricing
● The practice whereby-elements of prices are not revealed upfront.
Nagging
● The practice in which users face an overload of requests, information, options, or
interruptions; unrelated to the intended purchase of goods or services, which disrupts
the intended transaction.
13. Consider the following statements regarding Nation First Transit Cards.
1. It aims to facilitate a seamless and convenient customer commuting experience.
2. Individuals can also use this card for making retail and e-commerce payments.
3. It is powered by RuPay and National Common Mobility Card (NCMC) technology.

AMIGOS IAS 7
Which of the above given Statements is/are correct ?
A. 2 Only B. 3 Only
C. 2 and 3 Only D. 1,2 and 3
Explanation : D

Nation First Transit Card:


● It aims to facilitate a seamless and convenient customer commuting experience and
ensure easy digital ticketing fare payments in metro, buses, water ferries, parking, etc.,
through a single card.
● In addition, individuals can also use this card for making retail and e-commerce payments.
● It is powered by RuPay and National Common Mobility Card (NCMC) technology.

14. Consider the following statements regarding Indian oil sardine:


1. It is a very important pelagic fish species.
2. It contributes to about 50% of the total marine fish production in India.
3. It is distributed on the entire west coast.
4. Highest abundance and large-scale shoaling are observed off Andhra Pradesh coast.
How many of the above given Statements is/are correct ?
A. Only one B. Only two
C. Only three D. All four
Explanation : B
• Recently, a team of scientists at the ICAR-Central Marine Fisheries Research Institute here have
decoded the complete genome of the Indian oil sardine (Sardinella longiceps).
Indian oil sardine:
● It is a very important pelagic fish species.
● It contributes to about 15% of the total marine fish production in India.
● Local names of oil sardine are, Mathi, Nalla mathi, Nei chala (Malayalam); Bhutai
(Kannada), Tarli (Marathi and Hindi).
● It is distributed on the entire west coast from Gujarat to Kerala and also on the Tamil
Nadu, Andhra Pradesh and Orissa coasts in the east.
● Highest abundance and large-scale shoaling are observed off Kerala and Karnataka coasts.

15. Consider the following statements regarding the Wide Field Survey Telescope.
1. It is the largest time-domain survey facility in the Southern Hemisphere.
2. Its core scientific objective is to explore and monitor dynamic astronomical events.
3. It will enable researchers to detect faint and distant celestial signals, including those from
galaxies beyond the Milky Way and galaxy clusters.

AMIGOS IAS 8
Which of the above given Statements is/are correct ?
A. 1 and 2 Only B. 3 Only
C. 2 and 3 Only D. 1,2 and 3
Explanation : C
• China is set to launch the Wide Field Survey Telescope (WFST) in September.
Wide Field Survey Telescope:
● It is the largest time-domain survey facility in the Northern Hemisphere.
● The core scientific objective is to explore and monitor dynamic astronomical events and
conduct time-domain astronomical observation research.
● This will enable researchers to detect faint and distant celestial signals, including those
from galaxies beyond the Milky Way and galaxy clusters.
● It will also seek out planets or their moons in the Kuiper Belt and beyond.
● The telescope is located at the Lenghu astronomical observation base in northwest
China's Qinghai Province.
● The WFST project was launched in July 2019 through a collaboration between the
University of Science and Technology of China and the Purple Mountain Observatory
under the Chinese Academy of Sciences.

16. Consider the following statements with reference to the difference between a fixed and floating
interest rate:
1. Fixed interest rates do not change during the tenure of the loan whereas floating interest
rates are subject to market dynamics and the base rate.
2. Floating interest rates are generally higher than fixed interest rates.
3. Floating interest rate loans draw a prepayment penalty.
4. The fixed rate-based regime endows a borrower with greater certainty and security.
How many of the above given Statements is/are correct ?
A. Only one B. Only two
C. Only three D. All four
Explanation : B
• Recently, the Reserve Bank of India (RBI) issued guidelines enabling a borrower to transition
from a floating interest rate-based loan to one with a fixed interest rate.
Difference between a fixed and floating interest rate:
● Fixed interest rates are those that do not change during the tenure of the loan.
● On the other hand, floating interest rates are subject to market dynamics and the base
rate — therefore, the risk differentiation.
● As also contended by several lending entities, floating interest rates are generally lower
than fixed interest rates. For example, if the floating interest rate for home loans is 10.5%,
the fixed interest rate would be 12%.

AMIGOS IAS 9
● It has been widely argued that preference for the floating rate-based regime is to better
adjust their positions as per the evolving market dynamics.
● The advantages are transmitted onto the borrower’s savings pool, but the opposite also
holds true in a rising benchmark rate regime.
● Also noteworthy is the fact that floating interest rate loans do not draw any prepayment
penalty— unlike fixed rate loans.
● However, the fixed rate-based regime endows a borrower with greater certainty and
security. This also helps in better planning and structuring of individual budgets.
17. Consider the following statements regarding the African Union.
1. It is a continental body consisting of the 55 member states
2. It was officially launched in 2002, in Durban, South Africa.
3. The African Union (AU) has recently joined the G-20 as negotiators agreed on clearing its
membership.
Which of the above given Statements is/are correct ?
A. 1 and 2 Only B. 3 Only
C. 1 Only D. 1 ,2 and 3
Explanation : D
● The African Union (AU) is set to join the G-20 as negotiators agreed on clearing its
membership.
● It will mean the 55-member AU will join the European Union as the only two regional
bodies in the G-20.
African Union:
● The African Union (AU) is a continental body consisting of the 55 member states that
make up the countries of the African Continent.
● It was officially launched in 2002, in Durban, South Africa, as a successor to the
Organisation of African Unity (OAU, 1963-1999).
● The AU is guided by its vision of “An Integrated, Prosperous and Peaceful Africa, driven
by its own citizens and representing a dynamic force in the global arena.”

18. Consider the following statements regarding Swachh Vayu Sarvekshan 2023 Rankings.
1. It ranks cities on the basis of the implementation of activities approved under the city
action plan.
2. Cities are required to do the self-assessment as per the framework provided on the VAYU
online portal.
3. Chennai (million plus population) emerged as the first ranker in the index.
Which of the above given Statements is/are correct ?
A. 1 and 2 Only B. 3 Only
C. 2 and 3 Only D. 1 Only

AMIGOS IAS 10
Explanation : D
Swachh Vayu Survekshan
● Swachh Vayu Survekshan is an initiative by the Ministry of Environment, Forest and
Climate Change (MoEF & CC).
● It ranks cities on the basis of the implementation of activities approved under city action
planand air quality in 131 National Clean Air Programme cities.
● Cities are required to do the self-assessment as per the framework provided on the
PRANA online portal.
● Major Components of Assessment: solid waste management, road dust management,
etc.
● First ranked cities: Indore (million plus population), Amravati (3-10 lakhs population) and
Parwanoo (less than 3 lakhs population).

19. Consider the following statements regarding Cheriyal Scroll painting.


1. They are unique to Odisha, having Geographical Indication (GI) status.
2. It is a popular and modified version of Nakashi art.
3. They are used in story-telling by the balladeer community called Kaki Padagollu
How many of the above given Statements is/are incorrect ?
A. Only one B. Only two
C. All three D. None
Explanation : A
Cheriyal Scroll painting
● It is a popular and modified version of Nakashi art, highly rich in local motifs.
Features:
● Painted in narrative format and depict stories from Indian mythology as well as shorter
stories related to the Puranas and Epics.
● Used in story-telling by the balladeer community called Kaki Padagollu.
● Painted in mostly primary colors, showing a predominance of red color in the background.

20. Consider the following statements regarding Gati Shakti Vishwavidyalaya (GSV).
1. It is a State University in Vadodara (Gujarat), established in 2023.
2. It is India’s first University in the Transportation and Logistics sectors.
3. It subsumes the previously-existing National Rail and Transportation Institute (NRTI).
Which of the above given Statements is/are correct ?
A. 1 and 2 Only B. 2 Only
C. 2 and 3 Only D. 1 Only
Explanation : C

AMIGOS IAS 11
Gati Shakti Vishwavidyalaya (GSV)
● GSV is a Central University in Vadodara (Gujarat), established through an Act of
Parliament in 2022.
● It is India’s first University in the Transportation and Logistics sectors.
● It subsumes the previously-existing National Rail and Transportation Institute (NRTI).
● Sponsored by the Ministry of Railways.
● Mandate: applied education, training, skilling, and research in the logistic sector.

21. Consider the following statements with reference to Apis mellifera.


1. It is the most common of the 7–12 species of honey bees worldwide.
2. It is native to Europe, western Asia, and Africa.
3. Today, it can be found on every continent except Antarctica.
4. They can be employed as biomonitors for determining the spread of antimicrobial
resistance (AMR).
How many of the above given Statements is/are correct ?
A. Only one B. Only two
C. Only three D. All four
Explanation : D
● A recent study has found that Apis mellifera can be an effective biomonitor for
determining the spread of antimicrobial resistance (AMR) in urban neighbourhoods.
Biomonitoring
● It is a tool for measuring biodiversity and pollution through the analysis of pollen collected
by bees, which act as natural drones and bioindicators.
Apis mellifera
● The western honey bee or European honey bee (Apis mellifera) is the most common of
the 7–12 species of honey bees worldwide.
● It is native to Europe, western Asia, and Africa.
● Now it can be found on every continent except Antarctica.
22. Consider the following Pairs :
UPI Product Specifications
1. Hello! UPI Conversational payments on UPI
2. UPI LITE X To enhance adoption of QR code and NFC
technology.
3. UPI Tap & Pay It was launched for offline payments
4. Credit Line on UPI Financial inclusion through pre-sanctioned
credit lines from banks via UPI.

AMIGOS IAS 12
How many of the above given pairs is /are correct ?
A. Only one B. Only two
C. Only three D. All four
Explanation : B
• The NPCI has recently launched a slew of new payment options on popular payments platform
UPI.
1. Hello! UPI
● It is a feature for conversational payments on UPI.
● It will help users make voice-enabled UPI payments via apps, telecom calls, and IoT
devices in Hindi and English.
2. BillPay Connect
● Through a nationalized number for bill payment across India it enables users to fetch and
pay their bills by sending a 'Hi' on the messaging app across the country.
● It has been introduced by the NPCI’s subsidiary Bharat BillPay.
3. Credit Line on UPI
● To increase access to credit, and promote financial inclusion and innovation through pre-
sanctioned credit lines from banks via UPI.
4. UPI LITE X
● It was launched for offline payments
● It was built based on the UPI Lite feature, which was launched in 2022.
● UPI LITE X will be accessible to anyone with a compatible device that supports Near Field
Communication (NFC).
5. UPI Tap & Pay
● To enhance adoption of QR code and NFC technology, NPCI introduced ‘UPI Tap & Pay’.
23. The Minister of Fisheries, Animal Husbandry & Dairying has recently laid down the foundation
stone for the establishment of Multi-Purpose Seaweed Park recently in ?
A. Karnataka B. Telangana
C. Uttarakhand D. Tamil Nadu
Explanation : D
● Minister of Fisheries, Animal Husbandry & Dairying has recently laid down the foundation
stone for the establishment of Multi-Purpose Seaweed Park recently in Tamil Nadu.
● India’s 1st Fisheries Aquapark in form of ‘Multipurpose Seaweed Park in Tamil Nadu’ was
announced in the Union Budget 2021.
● Model - Hub-and-spoke model.
● It aims to promote the cultivation and conservation of seaweed.
● The seaweed park will promote the sea weed cultivation in 136 coastal fishing villages in
6 coastal districts of Tamil Nadu.

AMIGOS IAS 13
24. Consider the following statements with reference to Purana Qila.
1. It was built by Mughal Emperor Shahjahan as a part of his new city of Dinpanah in the
16th century.
2. Qila-e-Kuhna mosque, Sher Mandal, Lal Darwaza and Khairul Manazil etc are it's major
architectural features
3. Latest excavations indicate only the “poor” class of citizens used to stay here.

How many of the above given Statements is/are correct ?


A. Only one B. Only two
C. All three D. None
Explanation : A
● Latest excavations indicate that an “elite” class of citizens were staying there.

Major Findings
● Brick platform and potter’s kiln, Gold foil embedded in a wooden object, engraved lotus
symbol on red sandstone from the Kushan period.
● Brick jelly floor from Gupta period.
● Purana Qila was built by Mughal Emperor Humayun as a part of his new city of Dinpanah
in the 16th century.
● Major architectural features: Qila-e-Kuhna mosque, Sher Mandal, Lal Darwaza and Khairul
Manazil etc.
● Excavations here have provided evidence that Delhi has a continuous history for almost
3,000 years.

25. Consider the following pairs regarding AYUSH Public Health Programmes.
Programme Aim
1. AYURVIDYA Provides maternal & neonatal Intervention
for both the foetus and mother.
2. SUPRAJA Promote Healthy Lifestyle through AYUSH
for School Children
3. VAYO MITRA It aims to provide palliative services.
4. KARUNYA It aims to provide Geriatric Healthcare
Services to the elderly.
How many of the above given pairs is/are matched incorrectly ?
A. Only one B. Only two
C. Only three D. All four

AMIGOS IAS 14
Explanation : D
• Union Ayush Minister has called for strengthening of existing Ayush health programs namely:
1. AYURVIDYA: It aims to promote Healthy Lifestyle through AYUSH for School Children in
75000 Schools.
2. SUPRAJA: It aims to provide maternal & neonatal Intervention for both the foetus and
mother.
3. VAYO MITRA: It aims to provide Geriatric Healthcare Services to the elderly.
4. KARUNYA: It aims to provide palliative services.
26. Consider the following statements with reference to the Marine Sand Watch report.
1. It is the first-ever global data platform on sand and other sediment extraction in marine
environment.
2. The Platform is developed by the Intergovernmental Panel On Climate Change (IPCC).
3. Marine sand extraction expected to rise to 10 to 16 billion tonnes per year.

How many of the above given Statements is/are correct ?


A. Only one B. Only two
C. All three D. None
Explanation : B
Marine Sand Watch
● Marine Sand Watch is the first-ever global data platform on sand and other sediment
extraction in marine environment, including hotspots like the North Sea, Southeast Asia.
● Platform is developed by GRID-Geneva, a Centre for Analytics within the UN Environment
Programme (UNEP).
Key Findings
● Some 6 billion tonnes of sand is being extracted annually from oceans.
● Marine sand extraction expected to rise to 10 to 16 billion tonnes per year, the natural
replenishment rate to maintain coastal and marine ecosystems.

27. Consider the following statements regarding Kunbi Community.


1. They are an agriculture community in Karnataka.
2. They are grouped under the Other Backward Class category in Maharashtra.
3. The Maharashtra Cabinet decided to give Kunbi caste certificates to members of the
Maratha community who hail from the Marathwada region.
Which of the above given Statements is/are correct ?
A. 1 and 2 Only B. 1 and 3 Only
C. 2 and 3 Only D. 1, 2 and 3
Explanation : C

AMIGOS IAS 15
● The Maharashtra Cabinet recently decided that Kunbi caste certificates will be issued to
members of the Maratha community who hail from the Marathwada region.
● Kunbis, the community associated with agriculture are grouped under the Other
Backward Class category in Maharashtra.
● Those belonging to the Maratha community in Marathwada region of Maharashtra with
references of being Kunbis in the Nizam-era revenue and education documents will get
Kunbi certificates.
● A five-member panel headed by (retired) judge Sandeep Shinde will determine the
Standard Operating Procedure for giving caste certificates to members of the community
referred to as Kunbis in Nizam-era documents.
28. The “Schizostachyum andamanicum” was discovered on the island about three decades ago and
now its economic potential has received a boost with the granting of the patent. This species is
characterized by a thin large hollow erect culm with long internodes and has potential for
developing into a straw. The species being discussed is a species of?
A. Banana B. Silk
C. Teak D. Bamboo
Explanation : D

● The patent office of Government of India has granted a patent to Botanical Survey of India
for ‘reusable straw and its manufacturing’.
● The reusable straw is developed from a species of endemic bamboo
plant(Schizostachyum andamanicum) which is found in the Andamans and Nicobar
Islands.
● The Schizostachyum andamanicum was discovered on the island about three decades ago
and now its economic potential has received a boost with the granting of the patent for
reusable straw and its manufacture.
● This species of bamboo is characterized by a thin large hollow erect culm (stem) with long
internodes and has potential for developing into a straw.

29. Consider the following statements regarding India’s Main Battle Tank Arjun Mk IA.
1. It is developed indigenously with modern battle tank technologies.
2. It is a state-of-the-art weapon platform with 14 major upgrades over Arjun MBT Mk I.
Which of the above given Statements is/are correct ?
A. 1 Only B. 2 Only
C. Both 1 and 2 D. Neither 1 nor 2
Explanation : C
● Raksha Rajya Mantri visited DRDO’s Combat Vehicles Research & Development
Establishment (CVRDE) of DRDO in Avadi, Chennai.
● He got familiarised with the advanced features of this indigenously developed Main Battle
Tank (Arjun Mk I A).

AMIGOS IAS 16
India’s Main Battle Tank (Arjun Mk IA):
● It is developed indigenously with modern battle tank technologies, and has enhanced the
country’s self-reliance in Armoured Fighting Vehicles.
● It is a state-of-the-art weapon platform with superior firepower, excellent mobility,
increased protection and crew comfort with 14 major upgrades over Arjun MBT Mk I
already in service with the Indian Army.
● After extensive field trial evaluation by the Users, two regiments of Arjun MBT Mk IA have
been ordered on Armoured Vehicles Nigam Limited (a Defence PSU).

30. Consider the following statements with reference to the Heat Index.
1. It is a crucial indicator that measures how the temperature feels to humans based on a
combination of earth temperature and albedo.
2. It provides a more accurate representation of how hot the weather actually feels
compared to just the air temperature.
3. Low heat index values can lead to heat stress, heat exhaustion, and even heatstroke,
posing serious health risks.
How many of the above given Statements is/are correct ?
A. Only one B. Only two
C. All three D. None
Explanation : A
● Recently, Iran recorded a scorching heat index of 70 degrees Celsius (°C) in the coastal
part of the country.
● The heat index, also known as apparent temperature, is a crucial indicator that measures
how the temperature feels to humans based on a combination of air temperature and
relative humidity.
● It provides a more accurate representation of how hot the weather actually feels
compared to just the air temperature.
Uses
● High heat index values can lead to heat stress, heat exhaustion, and even heatstroke,
posing serious health risks. Monitoring the heat index is vital for public safety, especially
during extreme heat events.
● It helps authorities issue heatwave alerts, imple-ment necessary precautions, and adapt
to extreme heat conditions by adjusting work schedules and providing cooling solutions
to protect public health and safety.

31. Consider the following statements regarding Erg Chech 002.


1. It is a sandy region of the Sahara in western Algeria and northern Mali.
2. It is an “ungrouped achondrite
3. It consists largely of shifting dunes.

AMIGOS IAS 17
4. It contains abundant lead-206 and lead-207, as well as undecayed uranium-238 and
uranium-235.
How many of the above given Statements is/are correct ?
A. Only one B. Only two
C. Only three D. All four
Explanation : D
● In May 2020, some unusual rocks containing distinctive greenish crystals were found in
the Erg Chech sand sea.
Erg Chech
● It is a sandy region of the Sahara in western Algeria and northern Mali.
● It consists largely of shifting dunes.
● On close inspection of the rocks containing distinctive greenish crystals, it turned out to
be from outer space, left over from the dawn of the Solar System.
● Erg Chech 002 is an “ungrouped achondrite” (its parent body and family relationship is
unknown.)
● Achondrites are rocks formed from melted planetesimals, which is what we call solid
lumps in the cloud of gas and debris that formed the Solar System. Ex – Angrites, Erg
Chech 002.
● Erg Chech 002 contains abundant lead-206 and lead-207, as well as undecayed uranium-
238 and uranium-235.

32. Recently, the Supreme Court sought views of the Central and the State Governments on the
Justice Amitava Roy Committee report. It is a committee on ?
A. Personal Data Protection B. Prison Reforms
C. Prohibition of Drugs D. Poverty Alleviation
Explanation : B
Justice Amitava Roy Committee:
● In September 2018, the Supreme Court had formed a Committee on Prison Reforms
chaired by former judge, Justice Amitava Roy.
● Purpose – To examine the various problems plaguing prisons in the country, from
overcrowding to lack of legal advice to convicts to issues of remission and parole.
● The decision was taken based on a letter from former Chief Justice of India R.C. Lahoti
highlighted the overcrowding in prisons, unnatural deaths of prisoners, gross inadequacy
of staff and the lack of trained staff.

33. Consider the following statements regarding the Kuiper Belt.


1. It is a flat ring of small icy bodies that revolve around the Sun beyond the orbit of the
planet Jupiter.
2. It is primarily composed of dwarf planets, asteroids, and comets.

AMIGOS IAS 18
3. Pluto is one of the most well-known Kuiper Belt objects.
Which of the above given Statements is/are correct ?
A. 1 and 2 Only B. 2 and 3 Only
C. 1 and 3 Only D. 1,2 and 3
Explanation : B
• Astronomers have recently found compelling evidence suggesting the existence of an Earth-like
planet in the distant Kuiper Belt of our solar system.
Kuiper Belt:
● The Kuiper belt, also called the Edgeworth-Kuiper belt, is a flat ring of small icy bodies that
revolve around the Sun beyond the orbit of the planet Neptune.
● It is named after Gerard Kuiper, a Dutch-American astronomer who hypothesised the
existence of such a region in the 1950s.
● There are millions of these icy objects, collectively referred to as Kuiper Belt objects
(KBOs) or trans-Neptunian objects (TNOs), in this belt.
● The Kuiper Belt is a region of leftovers from the solar system's early history.
● It is thought to be the source of most of the observed short-period comets, particularly
those that orbit the Sun in less than 20 years.
● The Kuiper Belt is primarily composed of small icy bodies, such as dwarf planets, asteroids,
and comets.
● Pluto, once considered the ninth planet in our solar system, is one of the most well-known
Kuiper Belt objects.

34. Consider the following statements regarding Lunar Reconnaissance Orbiter (LRO):
1. It is an ISRO spacecraft that was launched on June 18, 2009.
2. It has also been used to study the Moon's geology, mineralogy, and environment.
3. It orbits the Moon in an eccentric polar mapping orbit.

Which of the above given Statements is/are incorrect ?


A. 1 and 2 Only B. 3 Only
C. 2 and 3 Only D. 1 Only

Explanation : D
Lunar Reconnaissance Orbiter (LRO):
● It is a NASA spacecraft that was launched on June 18, 2009.
● It's objective is to make a 3D map of the Moon's surface from lunar polar orbit.
● It has also been used to study the Moon's geology, mineralogy, and environment.
● It orbits the Moon in an eccentric polar mapping orbit.

AMIGOS IAS 19
35. Consider the following statements with reference to Project Naman.
1. The Project will entail establishing facilitation and grievance redressal centers for
veterans and the kin of personnel who have lost their lives.
2. It will offer all government-to-customer services and also facilitate the updating of
pensioners' accounts on the SPARSH portal of veterans, next of kin and dependents.
Which of the above given Statements is/are correct ?
A. 1 Only B. 2 Only
C. Both 1 and 2 D. Neither 1 nor 2
Explanation : C
Project Naman:
● The Project will entail establishing facilitation and grievance redressal centres for
veterans and the kin of personnel who have lost their lives.
● The first such centre will be established in Delhi Cantonment.
● 'Naman' will house a Common Service Centre, which will facilitate the veterans and their
relatives.
● It will offer all government-to-customer services and also facilitate the updating of
pensioners' accounts on the SPARSH portal of veterans, next of kin and dependents.

36. Consider the following statements with reference to the Indian Green Building Council.
1. It is part of the Ministry of Environment and Forests and was formed in the year 2001.
2. It is India's Premier certification body.
3. The council provides services like developing new green building rating programmes,
certification services and green building training programmes.
4. It is headquartered in New Delhi.
How many of the above given Statements is/are correct ?
A. Only one B. Only two
C. Only three D. All four
Explanation : B
● Vijayawada Railway Station was awarded the ‘Green Railway Station’ certification with
the highest rating of Platinum by the Indian Green Building Council (IGBC).
Indian Green Building Council:
● The Indian Green Building Council (IGBC) is part of the Confederation of Indian Industry
(CII) and was formed in the year 2001.
● It is India's Premier certification body.
● The council offers a wide array of services, which include developing new green building
rating programmes, certification services and green building training programmes.

AMIGOS IAS 20
● The council also organises the Green Building Congress, its annual flagship event on green
buildings.
● The rating is based on six environmental categories, which include sustainable station
facility, health, hygiene and sanitation, energy efficiency, water efficiency, smart and
green initiatives and innovation and development.
● Headquarter: Hyderabad.

37. Consider the following statements with respect to the ‘Minister without portfolio.’
1. A minister without a portfolio means s/he will not have a department to handle or an
office to attend.
2. Such a minister draws salary and emoluments in the capacity of a minister
3. Business rules of either house of parliament do not allow such a mechanism.
Which of the above given Statements is/are incorrect ?
A. 1 and 2 Only B. 3 Only
C. 2 Only D. 2 and 3 Only
Explanation : D
Minister without Portfolio
● Government of India (Allocation of Business) Rules, 1961 allows President on advice of
PM to entrust responsibilities of specified business to a union minister without portfolio.
● Several leaders such as Lal Bahadur Shastri, V.K. Krishna Menon (at Centre) and
J.Jayalalithaa (in Tamil Nadu) were inducted as ministers without portfolio.
● A minister without a portfolio means s/he will not have a department to handle or an
office to attend. But s/he is entitled to all perquisites and privileges of a minister.
● Such a minister does not draw salary and emoluments in the capacity of a minister but
only as a member of the legislative assembly.
38. Consider the following statements with reference to UPI-ATM.
1. UPI-ATM provides customers with unified and secure card-less cash withdrawals.
2. It allows users to withdraw cash from multiple accounts using the UPI app.
3. It is set-up as a Brown Label ATM in association with RBI.
4. Airtel Payment Services has unveiled India’s first ever’ United Payments Interface (UPI) -
ATM.
How many of the above given Statements is/are incorrect ?
A. Only one B. Only two
C. Only three D. All four
Explanation : B
United Payments Interface (UPI) -ATM.
● Hitachi Payment Services has unveiled India’s first ever’ United Payments Interface (UPI)
-ATM.

AMIGOS IAS 21
● Set-up as a White Label ATM (WLA) in association with National Payments Corporation of
India (NPCI).
● WLA is owned and operated by non-banks entities.
● UPI-ATM will provide customers with unified and secure card-less cash withdrawals.
● It allows users to withdraw cash from multiple accounts using the UPI app.

39. Consider the following statements with reference to Malaviya Mission.


1. It is a student training Programme organized by the NCERT in association with the
Ministry of Culture.
2. The programme shall help develop innovative teaching methods and high-level
institutional facilities in all the constituent areas of higher education.
Which of the above given Statements is/are incorrect ?
A. 1 Only B. 2 Only
C. Both 1 and 2 D. Neither 1 nor 2
Explanation : A
Malaviya Mission
● The Teacher Training Programme is organized by the University Grants Commission
(UGC), in association with the Ministry of Education.
● The two-week online programme shall focus on various themes identified for course
curriculum/content for capacity building of faculty members at higher educational
institutions.
● The 8 themes include
○ Holistic and Multidisciplinary Education,
○ Indian Knowledge Systems (IKS),
○ Academic Leadership,
○ Governance and Management,
○ Higher Education and Society,
○ Research and Development,
○ Skill Development,
○ Student Diversity and Inclusive Education and
○ Information and Communication Technology.
● The programme shall help develop innovative teaching methods and high-level
institutional facilities in all the constituent areas of higher education.
● It aims to improve the quality of teachers training, build leadership skills in teachers and
help realize the goals of National Education Policy (NEP).

AMIGOS IAS 22
40. Consider the following statements regarding the Zero Draft Plastic Pollution Treaty.
1. It is prepared by Intergovernmental Negotiating Committee (INC) of United Nations
Environment Assembly (UNEA)
2. It is a legally non binding instrument to end plastic pollution including in the marine
environment.
3. India had not agreed with this proposal.
How many of the above given Statements is/are correct ?
A. Only one B. Only two C. All three D. None
Explanation : A
Zero Draft Plastic Pollution Treaty:
● It is prepared by INC but contains the objective and mandate of United Nations
Environment Assembly (UNEA) resolution 5/14.
○ UNEA resolution (5/14), formed INC, in order to develop "the instrument," that addresses
the full life cycle of plastic, including its production, design and disposal.
● It is an international legally binding instrument to end plastic pollution including in the
marine environment.
● The zero draft has 10 placeholders to discuss issues like the preamble, definitions,
principles and scope, in addition to the institutional arrangements and the final
provisions.
● Member states such as Saudi Arabia, Iran, and China at INC-2 stressed the importance of
defining the scope of the legally binding instrument.
● India had agreed with this proposal.

41. Consider the following statements regarding Flex-fuel vehicle technology :


Statement-I :
• Toyota recently unveiled a prototype of the Innova Hycross with a flex-fuel hybrid powertrain,
the world’s first BS6 Stage II-compliant flex-fuel vehicle.
Statement-II :
• A flex-fuel vehicle typically does not have an internal combustion engine (ICE) and it can not
run on more than one type of fuel, or a mixture of these fuels.
Which one of the following is correct in respect of the above statements?
A. Both Statement-I and Statement-II are correct and Statement-II is the correct explanation
for Statement-I
B. Both Statement-I and Statement-II are correct and Statement-II is not the correct
explanation for Statement-I
C. Statement-I is correct but Statement-II is incorrect
D. Statement-I is incorrect but Statement-II is correct

AMIGOS IAS 23
Explanation : C
● Toyota recently unveiled a prototype of the Innova Hycross with a flex-fuel hybrid
powertrain, the world’s first BS6 Stage II-compliant flex-fuel vehicle.
● Toyota claims the prototype can run on petrol with more than 20% ethanol blending that
is currently mandated in India.
Flex-fuel technology
● A flex-fuel vehicle typically has an internal combustion engine (ICE), but unlike a regular
petrol vehicle, it can run on more than one type of fuel, or a mixture of these fuels. The
most common versions use a blend of petrol and ethanol or methanol.
● Flex-fuel vehicles (prototype Hycross) can run on blends of ethanol that are far higher
than the current standard 20% mix (E20).
● This is made possible by equipping the engine with a fuel mix sensor and an engine control
module (ECM) programming that senses and automatically adjusts for any ratio of
designated fuels.

42. Consider the following statements with reference to Hubble Constant.


1. The Hubble Constant is the unit of measurement used to describe the intensity of death
of stars in the milky way.
2. Gravitational Waves can be used to ascertain Hubble Constant.
Which of the above given Statements is/are correct ?
A. 1 Only B. 2 Only
C. Both 1 and 2 D. Neither 1 nor 2
Explanation : B
● A group of scientists may have found a way to determine the rate of expansion of the
universe also known as the Hubble constant.

What is Hubble Constant?


● The Hubble Constant is the unit of measurement used to describe the expansion of the
universe.
● The Universe has been getting bigger since the Big Bang kick-started the growth about
13.82 billion years ago.

So far, scientists have used three methods to get these details:


● They compare the observed brightness of a stellar explosion, called a supernova, with its
expected brightness to figure how far away it could be.
● They use changes to the cosmic microwave background (CMB) – radiation leftover from
the Big Bang event.
● They use gravitational waves, ripples in spacetime produced when massive astronomical
objects.

AMIGOS IAS 24
43. Consider the following statements regarding Invasive alien species.
1. They are animals, plants and microbes that have been introduced by humans to new
regions.
2. Target 6 of Kyoto protocol aims to eliminate these species by 2030.
Which of the above given Statements is/are incorrect ?
A. 1 Only B. 2 Only
C. Both 1 and 2 D. Neither 1 nor 2

Explanation : B
● The Intergovernmental Science-Policy Platform on Biodiversity and Ecosystem Services
(IPBES) released a report on the status of invasive alien species (IAS).
● Alien species are animals, plants and microbes that have been introduced by humans to
new regions. Of these, invasive alien species have negative impacts on nature.
Kunming-Montreal Global Biodiversity Framework (KMGBF)
● It was adopted in the 15th Conference of parties to the UN Convention on Biological
Diversity.
● Target 6 of the framework is to eliminate, minimize, reduce and or mitigate the impacts
of invasive alien species on biodiversity and ecosystem services.
● It also states that the world has to prevent and reduce the rate of introduction and
establishment of invasive alien species by at least 50 per cent by 2030.

44. Consider the following statements regarding a declaration signed recently.


1. The declaration reaffirmed global commitments towards indigenous knowledge,
biodiversity and traditional, Complementary and integrative medicine (TCIM).
2. It aims to accelerate the production, regulation, and formal utilization of scientifically
proven TCIM products and practices.
3. It aims to fully recognize, respect and protect the rights of Indigenous Peoples.
Choose the correct answer from the below given codes :
A. Delhi Declaration
B. Bangalore Declaration
C. Hyderabad Declaration
D. Gujarat Declaration
Explanation : D
● Recently the two day WHO Traditional Medicine Global Summit 2023 was held at
Gandhinagar, Gujarat.
● The World Health Organization (WHO) has released the outcome document of first WHO
Traditional Medicine Global Summit 2023 in the form of “Gujarat Declaration”.

AMIGOS IAS 25
● The declaration will focus on the integration of traditional medicines in national health
systems and help unlock the power of traditional medicine through science.
Outcomes
● The declaration reaffirmed global commitments towards indigenous knowledge,
biodiversity and traditional,complementary and integrative medicine (TCIM).
● Accelerate the production, regulation, and formal utilization of scientifically proven TCIM
products and practices.
● Advance policies that promote standardized TCIM documentation, including through
expanded and accelerated use of the WHO International Classification of Diseases (ICD-
11) to enable integration of evidence and data collection on TCIM in a standardized way
within routine health information systems.

45. Consider the following statements regarding the Stump-tailed Macaque.


1. It is a species of macaque native to South Asia and Southeast Asia.
2. It is primarily a Herbivore.
3. It is listed as endangered on the IUCN Red List of species.
4. In India it is found in Arunachal Pradesh, Nagaland, Manipur, Mizoram and Tripura etc.
How many of the above given Statements is/are correct ?
A. Only one B. Only two
C. Only three D. All four
Explanation : B
• The Delhi Zoo has recently introduced a new animal species, the stump-tailed macaque, to its
collection.
Stump-tailed Macaque
● It is also called the bear macaque, is a species of macaque native to South Asia and
Southeast Asia.
● In India, it is found in evergreen forests in South Asia, including parts of Northeast India
in forests south of the Brahmaputra.
● Its range in India extends from Assam and Meghalaya to eastern Arunachal Pradesh,
Nagaland, Manipur, Mizoram and Tripura.
● It is primarily frugivorous, but eats many types of vegetation, such as seeds, leaves and
roots, but also hunts freshwater crabs, frogs, bird eggs and insects.
● The stump-tailed macaque has long, thick, dark brown fur covering its body, but its face
and its short tail are hairless.
● Males are larger than females.
● It is listed as vulnerable on the IUCN Red List of species.

AMIGOS IAS 26
46. Consider the following statements with reference to PM-DAKSH Yojana.
1. It is a National Action Plan for skilling marginalised persons covering SCs and ST's Only
2. The age criterion of the scheme is between 18-45 years.
3. The main objective of the Scheme is to enhance the competency level of the target groups
to make them employable both in self-employment and wage-employment.
Which of the above given Statements is/are correct ?
A. 1 and 2 Only B. 2 Only
C. 3 Only D. 2 and 3 Only
Explanation : D
PM-DAKSH Yojana:
● The Pradhan Mantri Dakshata Aur Kushalata Sampanna Hitgrahi (PM-DAKSH) Yojana is a
Central Sector Scheme.
● It was launched during 2020-21.
● The main objective of the Scheme is to enhance the competency level of the target groups
to make them employable both in self-employment and wage-employment for their
socio-economic development.
● It is a National Action Plan for skilling marginalised persons covering SCs, OBCs, EBCs,
DNTs, Sanitation workers and waste pickers.
● The age criterion of the scheme is between 18-45 years, and the income criteria are no
income limit for SCs, SafaiKaramcharis, including waste pickers and DNT.
● The annual family income should be below Rs.3 lakh for OBCs, and the EBCs (Economically
Backward Classes) annual family income should be below Rs.1 lakh.
● Nodal Ministry: Ministry of Social Justice & Empowerment (MoSJ&E).
47. Consider the following statements with reference to ‘Gramodyog Vikas Yojana’.
1. Target beneficiaries under the scheme include SCs, OBCs, EBCs, DNTs, Sanitation workers
and waste pickers.
2. Any person who is a Traditional Artisan, has Knowledge of Skill, and has experience in Art
and Craft can be a beneficiary under this scheme.
Which of the above given Statements is/are correct ?
A. 1 Only B. 2 Only
C. Both 1 and 2 D. Neither 1 nor 2
Explanation : B

Gramodyog Vikas Yojana :


● It is one of the two components of the Khadi Gramodyog Vikas Yojana, which aims to
promote and develop the village industries through common facilities, technological
modernisation, training, etc.
● It includes the activities carried out under different village industries.

AMIGOS IAS 27
● Target beneficiaries: Artisans, Unemployed Youth and Self Help Groups (SHGs)
● Eligibility criteria: Any person who is a Traditional Artisan, has Knowledge of Skill, and has
experience in Art and Craft.

48. Consider the following statements regarding Maritime Infrastructure Perspective Plan (MIPP).
1. It is recently released by the Ministry of External Affairs, Government Of India
2. It aims to synchronize infrastructure requirements of the Navy through a comprehensive
perspective plan model for the next 15 years.
3. It is also in compliance with broader policy directives on PM Gati Shakti project, disaster
resilience, transition to net zero, etc.
Which of the above given Statements is/are incorrect ?
A. 1 and 2 Only B. 3 Only
C. 2 and 3 Only D. 1 Only
Explanation : D

● The Minister of State for Defence released the Maritime Infrastructure Perspective Plan
(MIPP) 2023-37 at the 2nd edition of the biennial Naval Commanders Conference
recently.
● MIPP aims to synchronize and enmesh the infrastructure requirements of the Navy
through a comprehensive prospective plan model for the next 15 years.
● The Plan Document is aligned with the Government’s vision on creation of sustainable
infrastructure.
● It is also in compliance with broader policy directives on PM Gati Shakti project, disaster
resilience, transition to net zero, etc.

49. Which of the following best describes the “Arogya Maitri Cube” that has been in the news
recently.
A. It is the Group of Countries coming together for SDG 5 Coordination.
B. It is a mechanism through which people of developing countries' access to vaccines can
be improved.
C. It is the World’s first portable disaster hospital developed by India.
D. India's is the World's first such mechanism to promote one health concept.
Explanation : C
AAROGYA MAITRI CUBE
● India has built the world’s first portable disaster hospital, called the “Aarogya Maitri
Cube''.
● The hospital can be airlifted and assembled into 72 cubes that contain essential medical
equipment and supplies.

AMIGOS IAS 28
● The cubes can support 200 survivors for 48 hours during natural disasters or crises.
● It has been developed indigenously under the Project BHISHM (Bharat Health Initiative
for Sahyog Hita and Maitri).
● The hospital includes Operation theatres, mini-ICUs, Ventilators, Blood test equipment,
an X-ray machine, a Cooking station, Food, Water, a Shelter, Power generator.

50. These acts are unlawful or wanton acts committed with knowledge that there is a substantial
likelihood of severe and either widespread or long-term damage to the environment being
caused by those acts. While some Indian court judgments have casually used this term, the
concept has not been formally integrated into Indian law. The concept being discussed in the
above paragraph is ?
A. Acute Biocide B. Ecocide
C. Bioconcentration D. Cleanup
Explanation : B

Ecocide
● Ecocide, derived from Greek and Latin, translates to 'killing one's home' or 'environment'.
● Although there is currently no universally recognized legal description of ecocide, a group
of lawyers convened by an NGO named Stop Ecocide Foundation in June 2021 crafted a
definition that would place environmental devastation within the same realm as crimes
against humanity.
● According to their proposal, ecocide is defined as "unlawful or reckless actions carried out
with the awareness that there exists a substantial probability of causing severe and either
extensive or enduring harm to the environment."
● While some Indian court judgments have casually used the term 'ecocide,' the concept
has not been formally integrated into Indian law.
○ Chandra CFS and Terminal Operators Pvt. Ltd. v. The Commissioner of Customs and Ors
2015.
○ The T.N. Godavarman Thirumulpad vs Union Of India & Ors 1995 case in the Supreme
Court.

51. Consider the following statements regarding Pulikkali.


1. It is the biggest and the most important festival in the state of Kerala.
2. It is a harvest festival celebrated in the first month of the Malayalam Calendar.
3. The main theme of this folk art is tiger hunting.
4. It is celebrated as part of Onam festival

AMIGOS IAS 29
How many of the above given Statements is/are correct ?
A. Only one B. Only two
C. Only three D. All four
Explanation : D
Pulikkali:
● It is the biggest and the most important festival in the state of Kerala.
● It is a harvest festival celebrated at the beginning of the month of Chingam, the first
month of the Malayalam Calendar (Kollavarsham).
● Pulikkali (Tiger Dance) is one of the folk art forms of Kerala.
● On the fourth day of the Onam festival, artists paint their bodies like tigers with stripes of
yellow, red and black and dance to the rhythm of traditional percussion instruments such
as thakil, udukku and chenda.
● The main theme of this folk art is tiger hunting, with participants playing the role of tiger
and hunter.
● According to a popular legend, the festival is celebrated to welcome King Mahabali,
whose spirit is said to visit Kerala at the time of Onam.

52. Consider the following statements with reference to Aatmanirbhar Bharat Rozgar Yojana.
1. This scheme works through incentivising unemployed individuals.
2. It aims at the creation of new job opportunities by extending financial support to
employees of establishments.
3. It was launched in 2020.
How many of the above given Statements is/are correct ?
A. Only one B. Only two
C. All three D. None
Explanation : B
Aatmanirbhar Bharat Rozgar Yojana:
● It was launched in 2020.
● This was designed to stimulate the creation of new job opportunities by extending
financial support to employers of establishments registered with the Employees'
Provident Fund Organization (EPFO).
● This scheme aimed to incentivise employment of unemployed individuals, including those
who lost their jobs due to the pandemic, by covering both the employee and employer
contributions (24% of wages) for establishments with up to 1000 employees.

53. Consider the following statements regarding Autonomous Hill (Or District) Councils ( ADC's )
1. They are established under the recommendations of Padmanabhaiah Committee.
2. ADCs have up to 50 members with a term of three years.

AMIGOS IAS 30
3. Term of the District Councils is for five years from the date of their constitution.
4. ADCs all powers except to form courts
How many of the above given Statements is/are correct ?
A. Only one B. Only two
C. Only three D. All four
Explanation : A
Autonomous Hill (Or District) Councils ( ADC's )
● When India became independent, this provision was adopted with improvements into the
Sixth Schedule of the Constitution.
● This was based on recommendations made by a committee under then Assam premier
Gopinath Bordoloi.
● ADCs have up to 30 members with a term of five years, of whom four are nominated by
the governor and the remaining 26 are elected on the basis of adult franchise.
● Term of the District Councils is for five years from the date of their constitution.
● ADCs can make laws, rules and regulations with regard to land, forest, water, agriculture,
village councils, health, sanitation, village and town level policing, inheritance of property,
marriage and divorce, social customs, and mining, among other issues.
● ADCs also have powers to form courts to hear cases where both parties are members of
Scheduled Tribes and the maximum sentence is less than 5 years in prison.

54. Consider the following statements regarding DIKSHA Platform.


1. It is a national platform for the higher education sector.
2. It is an initiative of the University Grants Commission (UGC).
3. The core building blocks of DIKSHA comprise majority of National Digital Education
Architecture (NDEAR)
Which of the above given Statements is/are correct ?
A. 1 and 2 Only B. 2 Only
C. 3 Only D. 2 and 3 Only
Explanation : C
DIKSHA Platform:

● It is a national platform for school education, an initiative of the National Council for
Education Research and Training (NCERT), Ministry of Education.
● It provides e-content for schools via an online portal and a mobile application.
● It was developed based on the core principles of open architecture, open access, open
licensing, choice and autonomy.
● The core building blocks of DIKSHA comprise majority of National Digital Education
Architecture (NDEAR) building blocks, having enabled some successful use-cases of

AMIGOS IAS 31
NDEAR such as energised textbooks, online courses, content authoring, content sourcing,
interactive quizzes, question banks, chatbot, analytics and dashboard.

55. Consider the following statements regarding a rice variety recently in the news
1. It is known as the ‘Prince of Rice’ and is an aromatic variety of rice.
2. The ancestors of the present tribal communities of Koraput district have domesticated
the rice in the region for thousands of years.
3. The rice is grown in Koraput district’s Tolla, Patraput, Pujariput, Baliguda and Mohuli
areas.
Choose the correct answer from the below given Codes :
A. Sahabhagidhan Rice B. Ulikan Rice
C. Hazaridhan Rice D. Kalajeera Rice
Explanation : D
● Recently, Koraput Kalajeera Rice,’ an aromatic rice, has got Geographical Indications
status.
Koraput Kalajeera Rice:
● The farmers of Koraput district in Odisha have domesticated Kalajeera rice over
generations.
● It is known as the ‘Prince of Rice’ and is an aromatic variety of rice.
● The ancestors of the present tribal communities of Koraput district have domesticated
the rice in the region for thousands of years, contributing to the conservation of the crop.
● It is popular among rice consumers for its black color, good aroma, taste and texture.
● The ancient text explains that Kala Jeera rice improves memory and controls diabetes.
● It is believed to increase hemoglobin levels and the body's metabolism.

56. Consider the following statements regarding a Lake in India


1. It is one of the biggest artificial lakes in Meghalaya, spread over an area of 10 square km.
2. It is encircled by lush green East Khasi hills
3. It is formed by the confluence of the two streams, Umkhrah and Umshvrpi.
Choose the correct answer from the below given Codes:
A. Ward's Lake B. Umiam Lake
C. Sajar Nangli Thadlaskein Lake D. Thadlaskein Lake
Explanation : B
● The Meghalaya government recently adopted AI-enabled robotic technology to keep its
tourist hotspot, Umiam Lake, free of pollutants.
Umiam Lake:

AMIGOS IAS 32
● Umiam Lake, also known as Barapani Lake, is a lake in the state of Meghalaya 15 km (9.3
miles) north of Shillong, the capital of Meghalaya.
● It is encircled by lush green East Khasi hills.
● It is one of the biggest artificial lakes in Meghalaya, spread over an area of 10 square km,
● The lake was formed after a dam was constructed to generate hydroelectric power in
1965.
● The dam was a part of northeast India’s first hydel power project called the Umiam Umtru
Hydroelectric Power Project.
● The confluence of the two streams, Umkhrah and Umshvrpi, form the Wah Ro-ro stream
in the northwest of the town and joins River Umiam, the main source of water for the
lake.

57. Consider the following statements regarding the Urban Infrastructure Development Fund (UIDF).
1. It is established through the use of priority sector lending shortfall.
2. The Fund will be used by public agencies to create urban infrastructure in tier -1 cities
3. The fund is managed by the Reserve Bank of India
4. The interest rate on UIDF loans has been kept at Bank Rate minus 1.5 percent.
How many of the above given Statements is/are correct ?
A. Only one B. Only two
C. Only three D. All four
Explanation : B
Urban Infrastructure Development Fund (UIDF):
● UIDF is established through the use of priority sector lending shortfall.
● The Fund will be used by public agencies to create urban infrastructure in tier-2 and tier-
3 cities.
● Focus areas :
○ Basic services like sewerage and Solid Waste Management
○ water supply and sanitation
○ construction and improvement of drains/stormwater drains and
○ impact-oriented projects will be prioritised.
● It is managed by the National Housing Bank.
● The initial corpus for this Fund is ₹10,000 crore.
● It is established on the lines of the Rural Infrastructure Development Fund (RIDF).
58. ‘AG 365S’ has been in the news recently. It is a/an ?
A. Habitable planet
B. New genetic variety of wheat
C. Agricultural Drone

AMIGOS IAS 33
D. Bio Pesticide
Explanation : C
AG 365S:
● It is a multi-utility agricultural drone in the small category (less than 25kg).
● It has been designed and developed by Marut Drones; an Indian drone technology
company based in Hyderabad.
● The made-in-India kisan drone – AG 365 has been developed particularly for agricultural
purposes to reduce crop loss, lower agrochemical usage, and better yield and profits to
the farmers.
● It is the first multi-utility agriculture small-category drone to receive the DGCA-approved
type certificate in India.

59. Consider the following statements regarding the ULLAS- Nav Bharat Saksharta Karyakram.
1. It is a Centrally Sponsored Scheme implemented during FYs 2022-27 .
2. The slogan for the scheme is ‘Jan Jan Sakshar’
3. The scheme will cover non-literates and literates of the age group of 18 years and above
in some selected state/UTs in the country.
4. The scheme does not cover Critical Life Skills in its scope.
How many of the above given Statements is/are correct ?
A. Only one B. Only two
C. Only three D. All four
Explanation : B

ULLAS- Nav Bharat Saksharta Karyakram on Education for All


● It is a Centrally Sponsored Scheme (jointly funded by the Centre and states) and was
implemented during FYs 2022-27 in alignment with National Education Policy (NEP) 2020.
● The Scheme has five components, namely
○ Foundational Literacy and Numeracy
○ Critical Life Skills
○ Basic Education
○ Vocational Skills
○ Continuing Education
● The slogan for the scheme is ‘Jan Jan Sakshar’.
● The scheme will cover non-literates of the age group of 15 years and above in all state/UTs
in the country.

AMIGOS IAS 34
60. Consider the following statements regarding the Organised Crime and Corruption Reporting
Project (OCCRP).
1. It is a global network of investigative journalists.
2. It is spread across Europe, Africa, Asia and Latin America.
3. The entity was involved in the coverage of Pegasus spyware as well as the Panama Papers
leak.
Which of the above given Statements is/are correct ?
A. 1 and 2 Only B. 2 and 3 Only
C. 1 and 3 Only D. 1,2 and 3
Explanation : D
OCCRP
● The Organized Crime and Corruption Reporting Project is a global network of investigative
journalists. It is spread across Europe, Africa, Asia and Latin America.
● The organization was founded by Drew Sullivan and Paul Radu in 2006.
● It is a member of the Global Investigative Journalism Network.
● The group aims to develop and equip a global network of investigative journalists and
publish their stories. With this, OCCRP exposes crime and corruption so the public can
hold power to account.
● Its vision is “A world where lives, livelihoods, and democracy are not threatened by crime
and corruption.

61. Consider the following statements regarding PSLV-XL Variant.


1. The PSLV-XL is the ‘full configuration’ PSLV, fitted with 6 strap-on motors.
2. Aditya-L1 marks the 25th flight of PSLV-XL.
3. AstroSat and Mangalyaan missions were carried out through this vehicle
How many of the above given Statements is/are correct ?
A. Only one B. Only two
C. All three D. None of them
Explanation : C
● Aditya-L1 will be launched with the PSLV-XL Variant (PSLV-C57) Launch Vehicle.
● PSLV-XL - The PSLV-XL is the ‘full configuration’ PSLV, fitted with 6 strap-on motors, the
maximum for this expendable launch vehicle.
● Aditya-L1 mission aims to study the Sun's Corona, Chromosphere, and Photosphere and
marks the 25th flight of PSLV-XL.
Various important missions of PSLV XL Variants
1. Chandrayaan–1 2. Mangalyaan
3. AstroSat 4. EOS-06 satellite, and 8 nano-satellites

AMIGOS IAS 35
5. TeLEOS-2 6. DS-SAR
7. Aditya L1 mission

62. Consider the following Statements regarding the Status of National Council of Educational
Research and Training (NCERT).
1. It has been granted the deemed-to-be-university status by the Ministry of Higher
Education.
2. It is an implementation agency for bilateral cultural exchange programmes with other
countries.
3. Jadui Pitara, a play-based learning-teaching material is a work of NCERT.
Which of the above given Statements is/are correct ?
A. 1 and 2 Only B. 2 and 3 Only
C. 1 and 3 Only D. 1,2 and 3
Explanation : D
● The National Council of Educational Research and Training (NCERT) has been granted the
deemed-to-be-university status by the Ministry of Higher Education recently.
○ It is a recognition to higher education institutions that excel in specific areas of study.
○ It will now be able to award its own graduate, postgraduate and doctoral degrees.
National Council of Educational Research and Training (NCERT)
● The National Council of Educational Research and Training (NCERT) is an autonomous
organization.
● It was set up in 1961 by the Government of India.
● NCERT is an implementation agency for bilateral cultural exchange programmes with
other countries in the field of school education.
● Jadui Pitara – It is a play-based learning-teaching material tailored for children between
the age group of 3 and 8 years.

63. Consider the following statements with reference to Polar bears.


1. They are the only bear species to be considered marine mammals.
2. Polar bears can not swim for long distances to get from one piece of ice to another.
3. The oil and gas industry in the arctic is the reason behind habitat destruction of polar
bears.
Which of the above given Statements is/are correct ?
A. 1 and 2 Only B. 3 Only
C. 2 Only D. 1 and 3 Only
Explanation : D
● An innovative new technique developed by WWF and DNA specialist firm SPYGEN allows
scientists to isolate DNA from a polar bear’s footprint in the snow.

AMIGOS IAS 36
● Scientists for the first time are able to directly quantify the impact of human-caused
greenhouse gas emissions from specific sources on polar bear cub survival.
Polar Bears
● They are the only bear species to be considered marine mammals.
● Polar bears can swim for long distances and steadily for many hours to get from one piece
of ice to another.
● The oil and gas industry in the arctic comes with the potential risks of habitat destruction
of polar bears from oil exploration work.
● Polar bears can also be exposed to toxic chemicals such as pesticides through their prey,
which can affect a bear's biological functioning and ability to reproduce.
64. Consider the following statements regarding Adopt a Heritage 2.0 Scheme.
1. It seeks to invite corporate stakeholders to enhance amenities at monuments by utilising
their CSR funds.
2. It doesn't clearly define the amenities sought for different monuments as per the AMASR
Act 1958.
3. Public sector Entities are not allowed under this program
Which of the above given Statements is/are correct ?
A. 1 and 2 Only B. 1 Only
C. 1 and 3 Only D. 1, 2 and 3
Explanation : B
Adopt a Heritage 2.0 Scheme:
● It seeks to invite corporate stakeholders to enhance amenities at monuments by utilising
their CSR funds.
● It is a revamped version of the earlier scheme launched in 2017. It clearly defines the
amenities sought for different monuments as per the Ancient Monuments and
Archaeological Sites and Remains Act (AMASR Act) 1958.
● Interested stakeholders will be able to adopt a monument or specific amenities through
a dedicated web portal at indian heritage.gov.in.
● Adopt a Heritage Scheme is an initiative of the Ministry of Tourism in collaboration with
the Ministry of Culture and the Archaeological Survey of India.
● It was launched in September 2017 on World Tourism Day.
65. Consider the following statements regarding the Kokborok Language.
1. It is the language spoken by the Kuki people belonging to the State of Manipur.
2. It is an Aryan language and can be traced back to at least the 1st century AD
3. It is one of the state languages of Manipur
How many of the above given Statements is/are incorrect ?
A. Only one B. Only two
C. All three D. None of them

AMIGOS IAS 37
Explanation : C
● Recently, over 260 people were detained in Tripura during a 12-hour statewide strike
called by the Twipra Students’ Federation (TSF) to press for the introduction of Roman
script for Kokborok.
Kokborok Language:
● Kokborok is the language spoken by the Borok people belonging to the State of Tripura.
● The term kok means "verbal", and borok means "people" or "human".
● It is a Sino-Tibetan language and can be traced back to at least the 1st century AD when
the historical record of Tripuri kings started to be written down in a book called the
Rajratnakar.
● Kokborok got the written form in the year 1897 as Doulot Ahammad, a Muslim scholar,
wrote the first Kokborok Grammar named “ KOKBOROMA ANG TRIPURA – VYAKARAN
GRAMMAR.”
● It is one of the state languages of Tripura, notified on January 19, 1979.

66. Consider the following statements regarding RAMBHA-LP Payload of Chandrayaan-3.


1. It is a device used for characterizing a plasma.
2. It explores the changes occurring in the near-surface plasma environment throughout the
lunar day.
3. It was led by Space Physics Laboratory (SPL), Vikram Sarabhai Space Centre (VSSC),
Thiruvananthapuram.
How many of the above given Statements is/are correct ?
A. Only one B. Only two
C. All three D. None of them
Explanation : C
● Recently, the RAMBHA-LP payload on board Chandrayaan-3’s lander Vikram has
completed the first in-situ measurements of the surface-bound lunar plasma
environment over the south polar region.
RAMBHA-LP Payload:
● The Radio Anatomy of Moon Bound Hypersensitive ionosphere and Atmosphere -
Langmuir Probe, named after American chemist and physicist Irving Langmuir, who was
awarded the Nobel Prize in Chemistry.
● It is a device used for characterising a plasma.
● The probe operates without interruption, aiming to explore the changes occurring in the
near-surface plasma environment throughout the lunar day.
● It was led by Space Physics Laboratory (SPL), Vikram Sarabhai Space Centre (VSSC),
Thiruvananthapuram.
67. Consider the following statements regarding Superfoods.

AMIGOS IAS 38
1. They are nutrient-rich foods packed with vitamins, minerals, and antioxidants beneficial
for health.
2. They do not include Non vegetarian foods like Poultry and Fish products

Which of the above given Statements is/are correct ?


A. 1 Only B. 2 Only
C. Both 1 and 2 D. Neither 1 nor 2

Explanation : A
Superfoods
● Superfoods are nutrient-rich foods, primarily plant-based but including some fish and
dairy, packed with vitamins, minerals, and antioxidants beneficial for health.
● According to the American Heart Association, superfood lacks scientific recognition, with
no established criteria to define it.
● Scientists claim that the use of the term superfood is largely a marketing tool, with no
root in academic research.

68. Consider the following statements with respect to Asiatic Wild Dog (Dhole).
1. The Western and Eastern Ghats is a stronghold region for dholes.
2. They endemic to India
3. They inhabit dense jungles, steppes, mountains, scrub forests, and pine forests.
4. They are protected under Schedule I of the both Wildlife Protection Act 1972 and CITES
How many of the above given Statements is/are correct ?
A. Only one B. Only two
C. Only three D. All four
Explanation : B
● It is a wild canid found in the forests of central, south, and southeast Asia.
● Other Names: Indian wild dog, whistling dog, red wolf, red dog and mountain wolf.
● They are found throughout Eastern and Southeastern Asia.
● They can be seen as far north as Siberia, as far south as some Malaysian islands, and as
far west as the Indian peninsula.
● They are found in three clusters across India, namely the Western and Eastern Ghats, the
central Indian landscape and North East India. The Western and Eastern Ghats is a
stronghold region for dholes.
● Dholes are animals that inhabit dense jungles, steppes, mountains, scrub forests, and pine
forests.
● Conservation status:

AMIGOS IAS 39
○ IUCN Red List: Endangered
○ The Wildlife Protection Act 1972: Schedule II
○ CITES: Appendix II
69. Consider the following statements regarding Marine Heatwaves.
1. They are extended periods of unusually warm ocean temperatures in specific regions.
2. They occur worldwide in coastal and open ocean areas.
3. The long-term consequences of the Marine Heatwaves are more severe compared to
short term impacts

Which of the above given Statements is/are correct ?


A. 1 and 2 Only B. 2 and 3 Only
C. 1 and 3 Only D. 1, 2 and 3
Explanation : A
Marine heatwaves
● Marine heatwaves are extended periods of unusually warm ocean temperatures in
specific regions.
● They can result from factors like climate change, El Niño events, and natural oceanic
conditions.
● Marine heatwaves can harm ecosystems, causing coral and sponge decay, seabird deaths,
harmful algal blooms, seaweed depletion, and increased marine mammal strandings.
● Global Distribution: They occur worldwide in coastal and open ocean areas.
● Local vs. Long-Term Effects: While they can cause short-term local changes, long-term
consequences are often less severe than anticipated.
● Recent research indicates that fish populations may be surprisingly resilient to marine
heatwaves, with effects varying and often masked by natural ecosystem variability.

70. Consider the following statements regarding Micronesia.


1. It is a subregion of Oceania, encompassing around 2,000 small islands in the
Northwestern Pacific Ocean.
2. Major Archipelagos in Micronesia include Tristan da Cunha, Fernando De Noronha, The
Faroe Islands etc.
Which of the above given Statements is/are correct ?
A. 1 Only B. 2 Only
C. Both 1 and 2 D. Neither 1 nor 2
Explanation : A
Micronesia
● Recently, the United States signed agreements with Micronesia to extend economic
assistance to the island state.

AMIGOS IAS 40
● Micronesia is a subregion of Oceania, encompassing around 2,000 small islands in the
Northwestern Pacific Ocean.
● Major archipelagos in Micronesia include Caroline Islands, Gilbert Islands, Mariana
Islands, and Marshall Islands.

71. Consider the following statements regarding ENTAZIA.


1. It is a bio fungicide crop protection product formulated with Bacillus subtilis.
2. It is a sustainable tool to protect crops from fungi while maintaining environmental
integrity.
3. It leverages the natural capabilities of Bacillus subtilis to control bacterial leaf blight.
Which of the above given Statements is/are correct ?
A. 1 and 2 Only B. 2 and 3 Only
C. 1 Only D. 1,2 and 3
Explanation : D

ENTAZIA:
● It is a bio fungicide crop protection product formulated with Bacillus subtilis.
● It was developed by FMC India.
● It will provide farmers with a powerful and sustainable tool to protect their crops from
fungi while maintaining environmental integrity.
● It leverages the natural capabilities of Bacillus subtilis to control bacterial leaf blight, one
of the most serious diseases of rice.
● By activating the crop’s defence system against plant pathogens, it acts to prevent and
control bacterial leaf blight while staying harmless to natural predators and parasites.
72. Recently, India’s First AI School came up in which of the following States/Union Territories ?
A. Andhra Pradesh B. Karnataka
C. Delhi D. Kerala
Explanation : D
India’s First AI School:
● It was launched by Santhigiri Vidhyabhavan in Thiruvananthapuram, Kerala.
● The establishment of this AI school is the result of a collaboration between iLearning
Engines (ILE) USA and Vedhik eSchool.
● The school’s curriculum aligns with the National School Accreditation Standards, which
are based on the New National Education Policy (NEP 2020).
● The school's unique approach centres on deepening students' understanding of Artificial
Intelligence (AI) and advanced technologies, enhancing their learning journey.

73. Consider the following statements regarding a protected area in India.

AMIGOS IAS 41
1. It is situated in the Kawardha district of Chhattisgarh.
2. It is named after the famous 11th-century Temple situated near the Sanctuary.
3. It shares a border with Kanha National Park in Madhya Pradesh.
4. The Chhattisgarh High Court recently dismissed a public interest litigation (PIL) that aimed
to designate it as a tiger reserve.
Choose the correct answer from the below given Codes :
A. Gomarda Wildlife Sanctuary B. Sitanadi Wildlife Sanctuary
C. Achanakmar Wildlife Sanctuary D. Bhoramdeo Wildlife Sanctuary
Explanation : D
• The Chhattisgarh High Court recently dismissed a public interest litigation (PIL) that aimed to
designate the Bhoramdeo Wildlife Sanctuary (BWS) as a tiger reserve.
Bhoramdeo Wildlife Sanctuary:
● It is situated in the Kawardha district of Chhattisgarh.
● It is named after the famous 11th-century Bhoramdeo Temple situated near the
Sanctuary.
● It shares a border with Kanha National Park in Madhya Pradesh, making it an important
tiger habitat in central India.
● The sanctuary covers an area of approximately 325 square kilometres (125 square miles).
● This wildlife sanctuary is the origin of the Fen and Sankari rivers.

74. Who among following Prominent Indians have won the Ramon Magsaysay Award award.
1. Vinoba Bhave 2. Mother Teresa
3. Satyajit Ray 4. Arvind Kejriwal
5. Ravi Kannan 6. Bezwada Wilson
Choose the correct answer:
A. 1, 2, 3 and 4 Only B. 2, 3, 4 and 6 Only
C. 2, 3, 4 and 6 Only D. 1 ,2 ,3 ,4, 5 and 6
Explanation : D
• Recently, Oncologist Ravi Kannan has been named one of the four winners of the 2023 Ramon
Magsaysay Award.
Ramon Magsaysay Award:
● It is Asia’s premier prize and the highest honour celebrates the greatness of spirit and
transformative leadership in Asia.
● The Ramon Magsaysay Awardees, annually selected by the RMAF board of trustees, are
presented with a certificate and a medallion with an embossed image of Ramon
Magsaysay facing right in profile.

AMIGOS IAS 42
Indian winners on the list
● Prominent Indians who have won the award include Vinoba Bhave in 1958, Mother Teresa
in 1962, Kamaladevi Chattopadhyay in 1966, Satyajit Ray in 1967, and Mahasweta Devi in
1997.
● In recent years, Arvind Kejriwal (2006), Anshu Gupta of Goonj (2015), human rights
activist Bezwada Wilson (2016), and journalist Ravish Kumar (2019) have won the award.

75. Consider the following statements regarding Project TAMARA.


1. It is a smart aeration system enhanced with sensors and IoT-based technology to manage
water quality.
2. This project is in line with other successful initiatives of the Govt. like Namami Gange and
Jal Shakti Abhiyan that focus on revitalizing and protecting India's waterbodies.
Which of the above given Statements is/are correct ?
A. 1 Only B. 2 Only
C. Both 1 and 2 D. Neither 1 nor 2
Explanation : C
● The Technology Development Board (TDB) of the Ministry of Science & Technology
supports the Waterbody Management Project "TAMARA" with 89 Lakhs out of 150 lakhs.
● Development and Commercialization of Intelligent Water Body Management System
(IWMS)-TAMARA.
● It is a smart aeration system enhanced with sensors and IoT-based technology to manage
water quality.
● This modern approach not only improves existing methods of treating water and
wastewater but also ensures that water bodies and aquaculture ponds stay clean and
healthy for everyone.
● Agency developing it - M/s Bariflo Labs Private Limited, Odisha.
● This project is in line with other successful initiatives of the Govt. like Namami Gange and
Jal Shakti Abhiyan that focus on revitalizing and protecting India's waterbodies.

76. The Radio Anatomy of Moon Bound Hypersensitive Ionosphere and Atmosphere - Langmuir
Probe (Rambha-LP) payload onboard Chandrayaan-3 is an instrument that measures ?
A. Amount of Hydrogen B. Plasma
C. Sulphur content D. Seismic movements
Explanation : B
● RAMBHA-LP payload on board Chandrayaan-3’s lander Vikram has completed the first in-
situ measurements of surface-bound lunar plasma environment recently.
● The RAMBHA Langmuir probe is an instrument that measures properties of plasmas, a
state of matter.

AMIGOS IAS 43
● Developed by - Space Physics Laboratory (SPL) at ISRO’s Vikram Sarabhai Space Centre
(VSSC).
● It is one of the 3 Indian payloads on the Chandrayaan-3 lander (Vikram).
77. Consider the following statements regarding The Global Fund.
1. It is a worldwide movement to defeat HIV, TB and Malaria
2. It was set up in 2022.
3. 100% of total funding comes from the donor governments.
4. It aims to accelerate impact towards the 2030 horizon as set out in Sustainable
Development Goal 3.
How many of the above given Statements is/are correct ?
A. Only one B. Only three
C. Only two D. All
Explanation : B
The Global Fund recently announced a deal with generic pharmaceutical manufacturers to
significantly slash the price of a cutting-edge HIV drug.
Global Fund
● It is a worldwide movement to defeat HIV, TB and Malaria and ensure a healthier, safer,
more equitable future for all.
● Partners- Governments, civil society, technical agencies, the private sector and people
affected by the diseases.
● Year - It was set up in 2002.
● Funding - The Fund pools the world’s resources to invest strategically in programs to end
AIDS, TB and malaria as public health threats.
● The financing is primarily from the public sector, with 92% of total funding coming from
donor governments.
● Strategy – To accelerate impact towards the 2030 horizon as set out in Sustainable
Development Goal 3.

78. Consider the following statements regarding the Special Session of Parliament.

1. The power to convene a session of Parliament rests with the government.


2. As per the Constitution Parliament meets for three sessions in a year.
3. The summoning of Parliament is specified in Article 80 of the Constitution.

Which of the above given Statements is/are correct ?


A. 2 and 3 Only B. 2 Only
C. 1 Only D. 1 and 2 Only

AMIGOS IAS 44
Explanation : C
● The Union government recently announced a Special Session of Parliament from
September 18 to 22.
● The power to convene a session of Parliament rests with the government.
● The decision is taken by the Cabinet Committee on Parliamentary Affairs, and is
formalised by the President, in whose name MPs are summoned to meet for a session.
● India does not have a fixed parliamentary calendar. By convention, Parliament meets for
three sessions in a year.
● The summoning of Parliament is specified in Article 85 of the Constitution. Like many
other articles, it is based on a provision of The Government of India Act, 1935.
● Article 85(1) says, the President shall from time to time summon each House of
Parliament to meet at such time and place as he thinks fit, but six months shall not
intervene between its last sitting in one session and the date appointed for its first sitting
in the next session.

79. Consider the following statements with respect to General Comment No. 26.
1. It is a document which clarifies the content of the rights set out in a treaty in question.
2. It provides a legal framework to address the adverse effects of environmental
degradation and climate change on the enjoyment of children’s rights.
3. It does not outline any potential violations of those rights.
Which of the above given Statements is/are correct ?
A. 1 and 2 Only B. 2 and 3 Only
C. 1 and 3 Only D. 1,2 and 3
Explanation : A
The United Nations has recognised a new guidance on children’s rights with a special focus on
climate change.
General Comment
● General Comment is a document which clarifies the content of the rights set out in a
treaty in question.
● It sometimes outlines potential violations of those rights and offers advice to states
parties on how best to comply with their obligations under the treaty.
General Comment No. 26
● It provides a legal framework to address the adverse effects of environmental
degradation and climate change on the enjoyment of children’s rights.
● It specifies that States are responsible not only for protecting children’s rights from
immediate harm, but also for foreseeable violations of their rights in the future due to
States’ acts — or failure to act — today.
● Furthermore, it underlines that States can be held accountable not only for environmental
harm occurring within their borders, but also for the harmful impacts of environmental
damage and climate change beyond their borders.

AMIGOS IAS 45
80. Consider the following statements with reference to Leaf Insects.
1. They are tropical insects that resemble leaves to avoid predators.
2. They are closely related to stick insects and are known for their remarkable camouflage.
3. Leaf insects are not found anywhere in India.
Which of the above given Statements is/are correct ?
A. 1 and 2 Only B. 2 and 3 Only
C. 1 and 3 Only D. 1,2 and 3
Explanation : A
Leaf Insects:
● Leaf insects are tropical insects that resemble leaves to avoid predators. They are closely
related to stick insects and are known for their remarkable camouflage.
● Leaf insects are found in the forests of Asia, Papua New Guinea, Australia, and the islands
of the Indian Ocean.
● These insects belong to the stick and leaf insect order, which are known for their
remarkable camouflage that makes them appear like parts of plants such as twigs, bark,
or leaves.
● This disguise offers protection from predators and poses challenges for researchers.
● Genetic analysis played a key role in identifying these “cryptic species” that cannot be
differentiated based solely on their external appearance.

81. This country is located at the Equator in West-Central Africa, bordering the Atlantic Ocean.A
former French colony, it retains strong ties to France and to the French language and culture.
Recently, mutinous soldiers claimed to have seized power hours after saying they had placed the
country’s newly re-elected president under house arrest.
The country being discussed in the above paragraph is ?
A. Nigeria B. Congo
C. Gabon D. Tanzania
Explanation : C
● Recently, mutinous soldiers claimed to have seized power in Gabon hours after saying
they had placed the country’s newly re-elected president under house arrest.
Gabon:
● The Republic of Gabon is located at the Equator in West-Central Africa, bordering the
Atlantic Ocean.
● It is bordered by Equatorial Guinea and Cameroon to the north and the Republic of the
Congo to the east and south.
● A former French colony, Gabon retains strong ties to France and to the French language
and culture.

AMIGOS IAS 46
● Gabon gained independence from France on August 17, 1960.

82. Consider the following statements regarding CE-20 cryogenic engine.


1. It has been designed and developed by the Liquid Propulsion Systems Centre (LPSC), ISRO.
2. It will power the Cryogenic Upper Stage of the LVM3 launch vehicle.
3. ISRO will use it for its ‘Mission Gaganyaan’.
4. It is the first Indian cryogenic engine to feature a gas-generator cycle.

How many of the above given Statements is/are correct ?


A. Only one B. Only two
C. Only three D. All four
Explanation : D
• Indian Space Research Organisation’s Liquid Propulsion Research Centre (IPRC) in Mahendragiri
has successfully tested the cryogenic rocket engine to be used in its ‘Mission Gaganyaan’.
CE-20 cryogenic engine:
● It has been designed and developed by the Liquid Propulsion Systems Centre (LPSC), a
subsidiary of ISRO.
● It will power the Cryogenic Upper Stage of the LVM3 launch vehicle.
● ISRO will use it for its ‘Mission Gaganyaan’ for sending man to space in 2024.
● It is the first Indian cryogenic engine to feature a gas-generator cycle.
● It is one of the most powerful upper-stage cryogenic engines in the world.
● This engine develops a nominal thrust of 186.36 kN in vacuum.

83. Consider the following statements with respect to the Intergovernmental Science-Policy Platform
on Biodiversity and Ecosystem Services (IPBES).
1. It is an independent intergovernmental body established in 2012.
2. The United Nations Environment Programme (UNEP) can provide secretariat services to
IPBES.
3. India is still not a member country of this organisation.
Which of the above given Statements is/are incorrect ?
A. 1 and 2 Only B. 1 and 3 Only
C. 3 Only D. 1 Only
Explanation : D
• The Intergovernmental Science-Policy Platform on Biodiversity and Ecosystem Services (IPBES)
has gathered at Bonn, Germany, for the body’s 10th plenary.
IPBES:
● It is an independent intergovernmental body established in 2012.

AMIGOS IAS 47
● It provides policymakers with objective scientific assessments about the state of
knowledge regarding the planet’s biodiversity, ecosystems and the benefits they provide
to people, as well as the tools and methods to protect and sustainably use these vital
natural assets.
● This independent body was inspired by the Intergovernmental Panel on Climate Change
(IPCC) and the Millennium Ecosystem Assessment.
● It is not a United Nations body. However, at the request of the IPBES Plenary and with
the authorisation of the UNEP Governing Council in 2013, the United Nations
Environment Programme (UNEP) provides secretariat services to IPBES.
● India is a member country of this organisation.

84. Consider the following statements regarding India’s Rice Production and Exports.
1. Globally, the top rice-producing country is India, followed by China.
2. Andhra Pradesh is the largest rice producer in India.
3. India is the largest rice exporter globally.
Which of the above given Statements is/are correct ?
A. 1 and 2 Only B. 1 Only
C. 2 Only D. 3 Only

Explanation : D
● Recently, government has levied certain restrictions on rice export to check the domestic
rise in prices and to ensure domestic food security.
● Globally, the top rice-producing country is China, followed by India.
● As per Second Advance Estimates, the estimated production of Rice for 2022-23 is
1308.37 Lakh Tonnes.
● West Bengal is the largest rice producer in India.
● India is the largest rice exporter globally with a 45% share in the world rice market.
● The Government has prohibited the export of white rice and broken rice.
● It levied a 20% duty on Parboiled non-basmati rice till October 15.
● It permitted the export of Basmati rice for contracts with value of 1,200 dollars a tonne
or above.
● Rice is the 3rd major agricultural commodity in India to face restrictions on overseas sales
in 2022-23.

85. Consider the following statements regarding Air Quality Life Index (AQLI) 2023.
1. It is produced by the World Health Organization
2. It converts air pollution concentrations & their impact on life expectancy.

AMIGOS IAS 48
3. Not a single nation across the globe achieved the air quality benchmark set by WHO as
per the report.
Which of the above given Statements is/are correct ?
A. 1 and 2 Only B. 2 and 3 Only
C. 1 and 3 Only D. 1,2 and 3
Explanation : B
● Annual update of the Air Quality Life Index (AQLI) 2023 has been released recently.
● AQLI, converts air pollution concentrations & their impact on life expectancy.
● It is produced by the Energy Policy Institute at the University of Chicago (EPIC).
● The AQLI was built to help solve some of these challenges by providing local information
on air quality and its health consequences.
● Not a single nation across the globe achieved the air quality benchmark set by the World
Health Organization, which stands at 5 micrograms of pollutants per cubic meter of air
(μg/m³) in the year 2021.

86. Consider the following statements regarding Renewable Energy Technology Action Platform.
1. It is launched solely by the Ministry of New and Renewable Energy (MNRE).
2. It is intended to advance new and emerging renewable technologies for deployment and
scaling.
3. Its only focus will be on green/clean hydrogen.
Which of the above given Statements is/are correct ?
A. 1 and 2 Only B. 2 Only
C. 2 and 3 Only D. 1,2 and 3
Explanation : B
• US Department of Energy (DOE) and Ministry of New and Renewable Energy (MNRE) have
together launched US-India Renewable Energy Technology Action Platform (RETAP) under the
Strategic Clean Energy Partnership.
Renewable Energy Technology Action Platform
● RETAP was announced by the Prime Minister of India and President of the USA during PM
Modi’s visit to the USA in 2023 for expanding collaboration on new and emerging
technologies to accelerate the clean energy transition.
● RETAP is intended to advance new and emerging renewable technologies for deployment
and scaling up in result oriented, time bound technology focus orientation.
● The initial focus will be on green/clean hydrogen, wind energy, long duration energy
storage, exploration of geothermal energy, ocean/tidal energy and other emerging
technologies as mutually determined in the future.

87. Consider the following statements regarding Red Sand Boa.

AMIGOS IAS 49
1. It is a Venomous species of Snake.
2. It is Endemic to India.
3. It primarily lives in semi-desert areas and foothills
4. It is protected by Schedule I of the Wildlife (Protection) Act, 1972.
How many of the above given Statements is/are incorrect ?
A. Only one B. Only two
C. Only three D. All four
Explanation : C
• A report by the Wildlife Conservation Society (WCS)-India has pointed out 172 incidents of
seizures of red sand boa between the years 2016-2021.
Red Sand Boa
● Scientific Name : Eryx johnii
● It is a non-venomous snake species. Nocturnal and spends the majority of its time under
the ground.
● It primarily lives in semi-desert areas and foothills.
● Endemic to India, Iran, and Pakistan.
● The red sand boa is now acknowledged as one of the most traded reptile species in the
illegal trade market, due to its demand in the pet trade, as well as for use in black magic.
● Protection Status :
○ IUCN Red List Status : ‘Near Threatened’
○ In India, it is protected by Schedule IV of the Wildlife (Protection) Act, 1972.
● It is listed in Appendix II of CITES.

88. Consider the following statements regarding Pacific Decadal Oscillation(PDO).


1. It is a 20–30-year cycle that occurs in the Pacific Ocean, north of 20°N.
2. During its positive phase, the west Pacific becomes warmer and part of the eastern ocean
cools.
3. The PDO has been linked to faster global warming.
How many of the above given Statements is/are correct ?
A. Only one B. Only two
C. All three D. None of them
Explanation : A
● The PDO is a 20–30-year cycle that occurs in the Pacific Ocean, north of 20°N.
● The PDO is a pattern of ocean-atmosphere climate variability.
● During a “warm”, or “positive”, phase, the west Pacific becomes cooler and part of the
eastern ocean warms. During a “cool”, or “negative”, phase, the opposite pattern occurs.

AMIGOS IAS 50
● The PDO has been linked to slower global warming because cold phases of the PDO
increase the mixing of colder, deep ocean waters with warmer surface waters.
● This temporarily reduces the rate of global warming caused by increasing greenhouse gas
emissions.

89. Consider the following statements regarding Liquid Apogee Motor (LAM).
1. It refers to a type of chemical rocket engine typically used as the main engine in a
spacecraft.
2. They are used for orbital adjustment manoeuvres of satellites/spacecraft in orbit.
3. It is developed by the Liquid Propulsion Systems Centre (LPSC), ISRO.
How many of the above given Statements is/are correct ?
A. Only one B. Only two
C. All three D. None of them

Explanation : C
● The smooth operation of Liquid Apogee Motor (LAM) engine critical to Aditya-L1 success.
● A liquid apogee engine (LAE), or apogee engine, refers to a type of chemical rocket engine
typically used as the main engine in a spacecraft.
● In simple terms, LAM engines are used for orbital adjustment manoeuvres of
satellites/spacecraft in orbit.
● The name apogee engine derives from the type of manoeuvre for which the engine is
typically used, i.e. an in-space delta-v change made at the apogee of an elliptical orbit in
order to circularize it.
● The successful operation of LAM, short for Liquid Apogee Motor, is vital to ISRO’s plans
to place the Aditya spacecraft in a halo orbit at Lagrangian point L1.
● It is developed by the Liquid Propulsion Systems Centre (LPSC), the ISRO centre for liquid
and cryogenic propulsion in Thiruvananthapuram.

90. Consider the following statements with reference to Funga.


1. Funga refers to the fungal diversity of a given place.
2. It is the Fauna and Flora equivalent to the kingdom of Fungi.
3. Fungi has been already included in the conservation strategies besides rare and
endangered plants and animals
How many of the above given Statements is/are correct ?
A. Only one B. Only two
C. All three D. None
Explanation : B

AMIGOS IAS 51
● United Nations Biodiversity has urged people globally to use the word ‘funga’ whenever
they say ‘flora and fauna’, in order to highlight the importance of fungi.
● Funga refers to the fungal diversity of a given place.
● It is the Fauna and Flora equivalent to the kingdom of Fungi.
● Fungi, along with Animalia (animals), Plantae (plants), Protista, Archaea/Archaebacteria,
and Bacteria or Eubacteria form the six ‘kingdoms’ of biology.
● The Species Survival Commission of the International Union for Conservation of Nature
announced that it would use mycologically inclusive language in its internal and public-
facing communications.
● This is to incorporate fungi in conservation strategies with rare and endangered plants
and animals.
91. Consider the following statements regarding Laser-Induced Breakdown Spectroscopy (LIBS)
Technique.
1. It is a scientific technique that analyses the composition of materials by exposing them to
intense laser pulses.
2. It uses a high-energy pulsar to generate plasma from rocks or soil.
3. Using the technique, chandrayaan-3' made the first-ever in-situ measurements on the
elemental composition of the lunar surface near the South Pole.
Which of the above given Statements is/are correct ?
A. 1 and 2 Only B. 3 Only
C. 2 and 3 Only D. 1,2 and 3
Explanation : D
● Pragyan rover confirms Sulphur at moon’s South Pole.
● The LIBS is a scientific technique that analyses the composition of materials by exposing
them to intense laser pulses.
● A high-energy laser pulse is focused onto the surface of a material, such as a rock or soil,
and generates an extremely hot and localized plasma.
● The LIBS instrument onboard the Chandrayaan-3’s rover has made the first-ever in-situ
measurements on the elemental composition of the lunar surface near the South Pole.
● The LIBS instrument on the rover, developed by ISRO’s Laboratory for Electro-Optics
Systems (LEOS), uses a high-energy pulsar to generate plasma from rocks or soil.
● The other instrument on the rover, called Alpha Particle X-Ray Spectrometer, is also
meant to study the elemental composition of the lunar surface.

92. Consider the following statements regarding School Health and Wellness Programme (SHWP).
1. It was initiated under the Swacch Bharat campaign by Prime Minister Narendra Modi.
2. Its primary objective is to impart health education, prevent diseases, and enhance access
to health services through wellness centres within educational institutions.

AMIGOS IAS 52
3. It is an initiative undertaken by the Ministry of Education and the Ministry of Health and
Family Welfare.
Which of the above given Statements is/are correct ?
A. 1 and 2 Only B. 3 Only
C. 2 and 3 Only D. 1,2 and 3
Explanation : C
● The Union Education and Skill Development & Entrepreneurship Minister launched a
novel comic book titled "Let’s Move Forward.
● The comic book imparts life skills essential pertaining to the 11 thematic components of
the School Health and Wellness Programme (SHWP) for fostering responsible and health-
conscious behaviour among adolescents.
School Health and Wellness Programme (SHWP)
● The SHWP was initiated under the Ayushman Bharat campaign in 2018 by Prime Minister
Narendra Modi.
● Its primary objective is to impart health education, prevent diseases, and enhance access
to health services through wellness centres within educational institutions.
● It is an initiative undertaken by the Ministry of Education and the Ministry of Health and
Family Welfare.

93. Consider the following statements with reference to Spamouflage.


1. It is a Chinese influence campaign that has been active on over 50 platforms and forums
2. The accounts in the campaign have a tendency to intersperse political posts
3. The campaign has pushed positive narratives about the United States and negative
commentary about China.
How many of the above given Statements is/are correct ?
A. Only one B. Only two
C. All three D. None
Explanation : B
Spamouflage:
● Spamouflage is a Chinese influence campaign that has been active on over 50 platforms
and forums, including Facebook, Instagram, TikTok, YouTube, and X (formerly Twitter).
● The accounts in the campaign have a tendency to intersperse political posts.
● The campaign has pushed positive narratives about China and negative commentary
about the United States, Western foreign policies, and critics of the Chinese government.
94. Consider the following statements regarding Hollongapar Gibbon Sanctuary.
1. It is an isolated protected area of evergreen forest located in the state of Manipur.
2. The sanctuary has a rich biodiversity and is home to the only apes in India, the western
Hoolock Gibbon

AMIGOS IAS 53
3. It is also home to the only nocturnal primate found in the northeast Indian states.
How many of the above given Statements is/are correct ?
A. Only one B. Only two
C. All three D. None
Explanation : B
● Recently, scientists suggested rerouting the railway track running through the
Hollongapar Gibbon Sanctuary.
Hollongapar Gibbon Sanctuary:
● The Hoollongapar Gibbon Sanctuary was renamed on 25 May 2004, formerly known as
the Gibbon Wildlife Sanctuary or Hollongapar Reserve Forest.
● It is an isolated protected area of evergreen forest located in the Jorhat district of Assam.
● The upper canopy of the forest is dominated by the Hollong tree, while the Nahar
dominates the middle canopy. The lower canopy consists of evergreen shrubs and herbs.
● The sanctuary has a rich biodiversity and is home to the only apes in India, the western
Hoolock, as well as the only nocturnal primate found in the northeast Indian states, the
Bengal slow loris.
● Also it is home to Stump-tailed macaque, northern pig-tailed macaque, eastern Assamese
macaque, rhesus macaque, and capped langur etc

95. Consider the following statements regarding Coking Coal.


1. It is a naturally occurring sedimentary rock found within the earth’s crust.
2. It typically contains less carbon, more ash and more moisture than thermal coal.
3. It is an essential ingredient in the production of steel.
4. Domestic raw Coking Coal production is projected to reach 140 MT by 2030,

How many of the above given Statements is/are correct ?


A. Only one B. Only two
C. Only three D. All four
Explanation : C

● Collaborative efforts between the Ministry of Steel and Ministry of Coal have yielded
positive outcomes to enhance domestic Coking Coal availability and minimize import
reliance.
● Metallurgical coal is also known as met coal and coking col
● It is a naturally occurring sedimentary rock found within the earth’s crust.
● Met coal typically contains more carbon, less ash and less moisture than thermal coal,
which is used for electricity generation.

AMIGOS IAS 54
● Metallurgical coal is an essential ingredient in the production of steel, making it one of
the most widely used building materials on earth.
● Metallurgical coal comes mainly from the United States, Canada and Australia.
● Domestic raw Coking Coal production is projected to reach 140 MT by 2030, yielding
about 48 MT of usable Coking Coal after washing. There is growing domestic demand for
Coking Coal as projected in the National Steel Policy 2017

96. Consider the following statements regarding the GREAT Scheme.


1. It gives impetus for the development of the Electric Vehicles startup ecosystem in India
2. Under this scheme, a grant-in-aid of up to Rs 50 lakh for up to a period of 18 months will
be provided.
3. It focuses on supporting individuals and companies to translate prototypes to
technologies & products, including commercialisation.
How many of the above given Statements is/are incorrect ?
A. Only one B. Only two
C. All three D. None
Explanation : A

GREAT Scheme:
● Grant for Research and Entrepreneurship across Aspiring Innovators in Technical Textiles
(GREAT) Scheme focuses on supporting individuals and companies to translate prototypes
to technologies & products, including commercialisation.
● It aims to provide much-needed impetus for the development of the technical textiles
startup ecosystem in India, especially in niche sub-segments such as bio-degradable and
sustainable textiles, high-performance and speciality fibres, and smart textiles.
● Under this scheme, a grant-in-aid of up to Rs 50 lakh for up to a period of 18 months will
be provided.
● The textile ministry will additionally provide 10 per cent of the total grant-in-aid to
incubators.
● Only a minimum of 10 per cent contribution has to be made by the incubatee.

97. Consider the following statements regarding Electrified Flex fuel vehicles.
1. An Electrified Flex Fuel Vehicle has both a Flex Fuel engine and an electric powertrain.
2. They do not have an internal combustion engine.
3. They are not capable of operating on gasoline.
4. Flex fuel engines are already popular in countries like Brazil, United States, European
Union etc..

AMIGOS IAS 55
How many of the above given Statements is/are correct ?
A. Only one B. Only two
C. Only three D. All four
Explanation : B
Electrified Flex fuel vehicle:
● It is the World’s first BS-6 Stage-II Electrified Flex fuel vehicle.
● An Electrified Flex Fuel Vehicle has both a Flexi Fuel engine and an electric powertrain.
● Flexible fuel vehicles (FFVs) have an internal combustion engine and are capable of
operating on gasoline and any blend of gasoline and ethanol up to 83%.
● In flex fuels, the ratio of ethanol to petrol can be adjusted, but the most commonly used
flex fuel uses 85 per cent ethanol and 15 per cent petrol.
● Bio-ethanol contains less energy per litre than petrol, but the calorific value (energy
contained in the fuel) of bio-ethanol will become on par with petrol with the use of
advanced technology.
● Since an FFV is capable of running on either petrol or ethanol, it will be the first of its kind
100 per cent dual fuel vehicle to be running on Indian roads.
● Flex fuel engines are already popular in Brazil, the United States, the European Union and
China, among many others.

98. Consider the following statements regarding Promotion of Research & Innovation in the Pharma-
MedTech sector (PRIP).
1. The aim of the scheme is to promote industry-academia linkage for R&D.
2. Rs 55,000 crore will be spent in ten years (2024-34) under the PRIP scheme
3. Antimicrobial Resistance has also been added in the scope of the scheme.
Which of the above given Statements is/are correct ?
A. 1 and 2 Only B. 2 and 3 Only
C. 1 and 3 Only D. 1, 2 and 3
Explanation : C
● The Union Cabinet recently approved the Promotion of Research & Innovation in Pharma-
MedTech sector (PRIP) scheme with an outlay of ₹5,000 crore for five years
PRIP Scheme:
● The objective of the PRIP scheme is to transform the Indian Pharma MedTech sector from
cost-based competitiveness to innovation-based growth by strengthening the research
infrastructure in the country.
● The aim of the scheme is to promote industry-academia linkage for R&D in priority areas
to inculcate a culture of quality research and nurture our pool of scientists.
● Rs 5,000 crore will be spent in five years (2024-28) under the PRIP scheme, and companies
will be given incentives for research.

AMIGOS IAS 56
● The six focus areas of the scheme are:
○ new chemical entities, including biological and phytopharmaceuticals;
○ complex generics and biosimilars;
○ precision medicines such as gene therapy and stem cells;
○ medical devices using artificial intelligence and machine learning;
○ orphan drugs;
○ anti-microbial resistance;

99. India’s defense minister recently signed an MoU with which of the following nations defense
minister for capacity building and collaboration in ship design and construction ?
A. Australia B. South Africa
C. Indonesia D. Kenya
Explanation : D
● On a visit to India, the Kenyan Cabinet Secretary for Defence signed an MoU with India’s
defence minister for capacity building and collaboration in ship design and construction.
● Kenya is located in East Africa between the Indian Ocean and Lake Victoria.
● Kenya is significant for India in the geopolitics of Western Indian Ocean. It is also an active
member of the African Union with whom India has a long-established connection.

100. Recently, African environment ministers have come to an agreement to establish national and
regional strategies aimed at minimizing the environmental impacts associated with the extraction
and processing of critical mineral resources. This decision stems from the 19th African Ministerial
Conference on the Environment (AMCEN) . The declaration being discussed here is ?
A. Washington Declaration B. Brussels Declaration
C. Ahmedabad Declaration D. Addis Ababa Declaration
Explanation : D
ADDIS ABABA DECLARATION
● African environment ministers have come to an agreement to establish national and
regional strategies aimed at minimizing the environmental impacts associated with the
extraction and processing of critical mineral resources.
● This decision stems from the 19th African Ministerial Conference on the Environment
(AMCEN) held in Addis Ababa, Ethiopia.
● The conference, themed “Seizing Opportunities and Enhancing Collaboration to Address
Environmental Challenges in Africa,” resulted in the Addis Ababa declaration, which
acknowledges pressing environmental challenges like land degradation, desertification,
and drought.
● The declaration emphasizes actions to combat climate change, plastic pollution, marine
protection, biodiversity conservation, and natural capital, contributing to global goals of
sustainable development and climate mitigation.

AMIGOS IAS 57
101. Consider the following statements with reference to Kampala Ministerial Declaration on
Migration, Environment and Climate Change (KDMECC)
1. KDMECC was originally signed and agreed upon by 15 African states in Kampala, Uganda
in July 2022.
2. It is the first comprehensive framework led by Member States to address climate-induced
mobility in a practical manner.
3. All African countries have recently agreed to adopt the Declaration to address the nexus
of human mobility and climate change in the continent.

Which of the above given Statements is/are correct ?


A. 1 and 2 Only B. 2 and 3 Only
C. 1 Only D. 2 Only

Explanation : A

KDMECC
● KDMECC was originally signed and agreed upon by 15 African states in Kampala, Uganda
in July 2022.
● The Declaration is the first comprehensive, action-oriented framework led by Member
States to address climate-induced mobility in a practical and effective manner.
● The KDMECC-AFRICA is expected to be signed by Member States during the Africa Climate
Summit in Nairobi on September 4, 2023.
● A total of 48 African countries have now agreed to adopt the Kampala Ministerial
Declaration on Migration, Environment and Climate Change (KDMECC) to address the
nexus of human mobility and climate change in the continent.

102. Consider the following statements with reference to IBSA World Games.
1. It is the world’s biggest sport event for athletes with visual impairments
2. The multi-sport competition is held every four years.
3. Powerlifting, Showdown and Ten pin are also part of the games in 2023.
4. Indian women's blind cricket team has won the Bronze at the IBSA World Games

How many of the above given Statements is/are incorrect ?


A. Only one B. Only two
C. Only three D. All four
Explanation : C
IBSA world games

AMIGOS IAS 58
● IBSA world games started in 1998 in Madrid, Spain and is the world’s biggest sport event
for athletes with visual impairments.
● IBSA is in charge of a wide range of sports for athletes who are blind or partially sighted.
● IBSA’s status as a legal entity was formalised in 1985 with the adoption of its first
constitution during the General Assembly held in Norway.
● With various names over the years including the ‘World Blind Sports Championships’ and
‘IBSA World Championships and Games’, the multi-sport competition is held every four
years.
● 2023 games hosted three paralympic and seven non-paralympic sports; Archery, Chess,
Cricket, Men’s Blind Football, Partially Sighted Football, Women’s Blind Football, Goalball
(Male/Female), Judo, Powerlifting, Showdown, Ten Pin, Bowling and Tennis.
● Indian women's blind cricket team has won the Gold at the IBSA World Games

103. Consider the following statements with reference to the Amitabh Kant Committee.
1. Its task is to suggest ways to revive stalled food processing projects.
2. The committee recommended Government's direct support to the real estate developers
3. The report suggested all stakeholders take “haircuts” to make the stalled projects
financially viable.

Which of the above given Statements is/are correct ?


A. 1 Only B. 2 and 3 Only
C. 2 Only D. 3 Only
Explanation : D
Amitabh Kant committee
● The Amitabh Kant committee was tasked by the Government with suggesting ways to
revive stalled real estate projects.
● The 14-member committee chaired by former NITI Aayog CEO Amitabh Kant, submitted
its report to Housing and Urban Affairs Minister Hardeep Singh Puri recently
● The report by the committee acknowledged the financial stress of real estate developers
but recommended no direct support.
● The report has essentially suggested that all stakeholders — developers, financiers and
land authorities — take “haircuts” to make the stalled projects financially viable.

104. Consider the following statements regarding the World Athletics Championships.
1. The World Athletics Championships 2023 took place in Budapest, Hungary recently.
2. It is an annual athletics competition organized by World Athletics.
3. Neeraj Chopra became the first Indian athlete to win World Athletics Championships ‘gold
medal in men's javelin throw.

AMIGOS IAS 59
Which of the above given Statements is/are correct ?
A. 1 and 2 Only B. 3 Only
C. 1 and 3 Only D. 1 Only

Explanation : C
World Athletics Championships:
● The World Athletics Championships 2023 took place in Budapest, Hungary, from 19–27
August 2023.
● The World Athletics Championships are a biennial athletics competition organized by
World Athletics.
● Alongside the Olympic Games, they represent the highest level championships of senior
international outdoor athletics competition for track and field athletics globally, including
marathon running and race walking etc.
● Neeraj Chopra achieved another historic feat by becoming the first Indian athlete to win
World Athletics Championships ‘gold medal’ during the men's javelin throw final event in
Budapest.
● From India till now the other two Indian javelin throwers were in contention – Kishore
Jena and DP Manu – who also produced impressive performances even as they couldn't
secure podium spots.

105. Consider the following statements regarding Shinon Meeras.


1. It is a museum to highlight the Dard-Shina tribes at the Gurez Valley in Jammu and
Kashmir.
2. Dard Community is a Hindu group inhabiting the villages in and around the Jhelum River.
3. They are culturally and linguistically similar to other tribes in region
Which of the above given Statements is/are correct ?
A. 1 and 2 Only B. 3 Only
C. 2 and 3 Only D. 1 Only
Explanation : D
● The Dard community in Jammu & Kashmir gets its own museum Shinon Meeras in Gurez
Valley recently.
● Shinon Meeras, a centre to highlight the Dard-Shina tribes was thrown open at the Gurez
Valley in north Kashmir’s Bandipora district.
● The center is a unique tribute to preserve and promote the glorious artistic heritage of
the Dard-Shin tribal community.
● Dard Community are a Buddhist group inhabited in the villages of Dha, Hanu, Garkone
and Darchik on both sides of the Indus River. They speak Shina language.

AMIGOS IAS 60
● The Dard or Dard Aryans or Dard-Shina tribes are the inhabitants of Jammu & Kashmir’s
Ladakh region.
● They are culturally and linguistically different from other tribes in region.
● Researchers believe that the ‘Aryans of Ladakh’ or the ‘Brokpas’ are descendants of
Alexander’s army and had come to the region over 2,000 years ago.
106. Consider the following statements regarding Caste Census in India.
1. The first caste-based census took place in 1881 during the colonial period.
2. The 1931 census was the last exercise to enumerate caste data.
3. India has counted and published caste data from 1951 to 2011 of the Scheduled Castes
and Scheduled Tribes only.
How many of the above given Statements is/are correct ?
A. Only one B. Only two
C. All three D. None
Explanation : C
Caste Census in India
● Caste census means inclusion of caste-wise tabulation of India’s population in the Census
exercise. India has counted and published caste data — from 1951 to 2011 — of the
Scheduled Castes and Scheduled Tribes only.
● In India, there have been caste-based censuses conducted in the past. The first caste-
based census took place in 1881 during the colonial period.
● The 1931 census was the last exercise to enumerate caste data. The population of OBCs
was 52% as last recorded in the 1931 caste census.
● The UPA government conducted Socio-Economic and Caste Census (SECC) in 2011. It was
the first caste-based census since 1931. The data was, however, not published.

107. Consider the following statements regarding Mahananda Wildlife Sanctuary.


1. It is located on the foothills of the Himalayas and in between the Teesta and the
Mahananda rivers.
2. It is situated in the Darjeeling district of West Bengal.
3. In 1959, it got the status of a sanctuary mainly to protect the Indian Bison and Royal
Bengal Tiger.
Which of the above given Statements is/are correct ?
A. 1 and 2 Only B. 2 Only
C. 2 and 3 Only D. 1,2 and 3
Explanation : D
Mahananda Wildlife Sanctuary:
● It is situated in the Darjeeling district of West Bengal.

AMIGOS IAS 61
● It is located on the foothills of the Himalayas and in between the Teesta and the
Mahananda rivers.
● It was started as a game sanctuary for children in 1955. In 1959, it got the status of a
sanctuary mainly to protect the Indian Bison and Royal Bengal Tiger.
● The Sanctuary reaches up to a high elevation at Latpanchar in Kurseong Hill.
● Terrain: The terrain is undulating, with moderate to steep slopes and high ridges towards
the north.
● Vegetation: It varies mainly from riverine forests to dense mixed-wet forests due to the
difference in altitude.

108. Consider the following statements :


Statement-I :
Astronomers recently conducted an autopsy of a star that got destroyed after wandering too close
to a supermassive black hole in what is known as a tidal disruption event (TDE).
Statement-II :
TDE is a phenomenon in astronomy that occurs when a star passes too close to a supermassive
black hole or a massive stellar-mass black hole.
Which one of the following is correct in respect of the above statements?
A. Both Statement-I and Statement-II are correct and Statement-II is the correct explanation
for Statement-I
B. Both Statement-I and Statement-II are correct and Statement-II is not the correct
explanation for Statement-I
C. Statement-I is correct but Statement-II is incorrect
D. Statement-I is incorrect but Statement-II is correct
Explanation : A
Tidal Disruption Event (TDE):
● TDE is a phenomenon in astronomy that occurs when a star passes too close to a
supermassive black hole or a massive stellar-mass black hole.
● The intense gravitational forces from the black hole cause tidal forces that can tear the
star apart. This process is also known as "tidal disruption."
109. Consider the following statements regarding Article 35A of the Indian Constitution
1. It was incorporated into the Constitution of India in 1954 by a Presidential Order,
following the 1952 Delhi Agreement
2. Article 35A gives the J&K Legislature complete freedom to decide the ‘permanent
residents’ of the State.
3. Article 35A appears in the main body of the Constitution.
Which of the above given Statements is/are correct ?
A. 1 and 2 Only B. 2 and 3 Only

AMIGOS IAS 62
C. 1 and 3 Only D. 1,2 and 3
Explanation : A
● Recently, Chief Justice of India D.Y. Chandrachud said Article 35A denied fundamental
rights to others.
● It empowered the Jammu and Kashmir Legislature to define “permanent residents” of the
State and provide them special privileges.
● It was introduced through the Constitution (Application to Jammu and Kashmir) Order,
1954 which was issued by the President under Article 370.
● Article 35A was incorporated into the Constitution in 1954 by an order of the then
President Rajendra Prasad on the advice of the Jawaharlal Nehru Cabinet.
● It was incorporated into the Constitution of India in 1954 by a Presidential Order,
following the 1952 Delhi Agreement [between the then central govt, and the then PM of
J&K Sheikh Abdullah].
● The Delhi Agreement extended Indian citizenship to the ‘State subjects’ of J&K.
● Article 35A is unique in the sense that -
○ It does not appear in the main body of the Constitution.
○ It by passed the parliamentary route of lawmaking. Article 368 of the Constitution
empowers only Parliament to amend the Constitution.

110. Consider the following statements with respect to Goldilocks economy.


1. It describes the worst state for an economy whereby the economy is expanding by too
much.
2. A Goldilocks economy has steady economic growth, preventing a recession, but not so
much growth that inflation rises by too much.
3. The Goldilock scenario is bad for investors as companies perform bad and stocks go down
How many of the above given Statements is/are correct ?
A. Only one B. Only two
C. All three D. None
Explanation : A
● Goldilocks position' for Indian equities is getting questioned with crude rallying whereas
China gaining some traction, rising consumer inflation and yields move up.
Goldilocks economy
● A Goldilocks economy describes an ideal state for an economy whereby the economy is
not expanding or contracting by too much.
● A Goldilocks economy has steady economic growth, preventing a recession, but not so
much growth that inflation rises by too much.
● In this state of the economy, there are certain features attached. For one, the
unemployment rate in the economy is really low.

AMIGOS IAS 63
● There is steady growth in the Gross Domestic Product (GDP) numbers and companies
report better earnings.
● The retail inflation and the interest rates are relatively low.
● The Goldilock scenario is good for investors as companies perform well and stocks rally.

111. Consider the following statements regarding Nabhmitra device.


1. It is a satellite-based communication system developed for the safety of fishermen.
2. It was developed by the Defence Research and Development Organization ( DRDO)
3. It enables two-way messaging services from and to the sea.
4. The device will also help in identifying fishing fields.

How many of the above given Statements is/are correct ?


A. Only one B. Only two
C. Only three D. All four

Explanation : C
Nabhmitra device:
● It is a satellite-based communication system developed for the safety of fishermen.
● It enables two-way messaging services from and to the sea.
● The weather and cyclone warnings will be communicated in the local language, the boats
can also send distress messages to the authorities.
● In emergency situations like capsizing and fire, fishers can press a button on the device
and get in touch with the control centre.
● While the control centre will receive the alert, including the location of the boat, the crew
on the boat will get a response message from the control centre.
● Apart from providing information about shipping channels and maritime boundaries, the
device will also help to identify fishing fields.
● It was developed by the Indian Space Research Organisation (ISRO).

112. Consider the following statements regarding Chokuwa rice.


1. It is also known as Magic rice cultivated in Manipur.
2. Chokuwa rice recently earned a GI (Geographical Indication) tag for its exquisiteness.
3. The sticky and glutinous variety is categorised as Bora and Chokuwa based on their
amylose concentration
Which of the above given Statements is/are correct ?
A. 2 Only B. 1 and 3 Only
C. 3 Only D. 2 and 3 Only

AMIGOS IAS 64
Explanation : D
• Chokuwa rice recently earned a GI (Geographical Indication) tag for its exquisiteness.
Chokuwa rice:
● It is also known as Magic rice cultivated in Assam.
● It is a part of Assam's culinary heritage; this unique rice has been a staple of the troops of
the mighty Ahom dynasty.
● This unique and healthy rice is cultivated around the Brahmaputra River area.(In several
parts of Assam like Tinsukia, Dhemaji, Dibrugarh, etc.)
● It is basically semi-glutinous winter rice, known as Sali rice.
● The sticky and glutinous variety is categorised as Bora and Chokuwa based on their
amylose concentration.
● The low amylase Chokuwa rice variants are used to make soft rice, which is known as
Komal Chaul or soft rice.

113. Consider the following statements with reference to Exercise Bright Star-23.
1. It is an annual multilateral tri-service exercise.
2. This is the second time that IAF is participating in Ex BRIGHT STAR-23.
3. This exercise was launched in 1980 as part of the US-brokered peace treaty between India
and Pakistan.
Which of the above given Statements is/are incorrect ?
A. 1 and 3 Only B. 2 Only
C. 3 Only D. 1, 2 and 3
Explanation : D
• Recently, an Indian Air Force (IAF) contingent departed to participate in Exercise BRIGHT STAR-
23, scheduled to be held at Cairo (West) Air Base, Egypt, from 27 August to 16 September 2023.
EXERCISE BRIGHT STAR-23:
● It is a biennial multilateral tri-service exercise.
● This multinational exercise was launched in 1980 as part of the US-brokered peace treaty
between Egypt and Israel.
● This is the first time that IAF is participating in Ex BRIGHT STAR-23.
● Participating countries: United States of America, Saudi Arabia, Greece and Qatar.

114. Consider the following statements with reference to Smishing.


1. It involves sending fraudulent text messages to individuals with the aim of tricking them
into divulging sensitive personal information
2. It is a type of Phishing attack
3. Smishing simply uses text messages instead of email.

AMIGOS IAS 65
4. Smishing messages typically contain urgent or enticing content to persuade recipients to
take immediate action
How many of the above given Statements is/are correct ?
A. Only one B. Only two
C. Only three D. All four
Explanation : D
● The term "smishing" is a combination of "SMS" (Short Message Service) and "phishing."
● When cybercriminals "phish," they send fraudulent emails that seek to trick the recipient
into clicking on a malicious link.
● Smishing simply uses text messages instead of email.
● Smishing messages typically contain urgent or enticing content to persuade recipients to
take immediate action, such as clicking on a malicious link, calling a phone number, or
providing sensitive information.

115. Consider the following statements regarding Seethakali folk art.


1. It is a unique centuries-old folk art form that has its origins in the Kollam district of Kerala
2. The characters of Rama and Laxmana appear in green in the art form
3. The instruments used during performances are all made of natural materials.
4. It is based on certain episodes taken from the Indian epic Mahabharata.
How many of the above given Statements is/are correct ?
A. Only one B. Only two
C. Only three D. All four
Explanation : C

Seethakali:
● It is a unique centuries-old folk art form that is believed to have originated at Perinad in
the Kollam district of Kerala.
● This art form was first performed some 150 years back by the people of Vedar and Pulayar
communities.
● It is based on certain episodes taken from the Indian epic Ramayana.
● In the early times, Seethakali was performed as part of the harvest festival Onam.
● The props and instruments used during performances are all made of natural materials
like bamboo and palm leaves.
● The costumes and the make-up are loud and eye-catching.
● The characters of Rama and Laxmana appear in green since the colour is used to represent
gods and goddesses in Kathakali.

AMIGOS IAS 66
116. Consider the following statements with reference to the 6th Census Report on Minor Irrigation
Schemes.
1. There is an increase of 6.6% in the number of minor irrigation schemes in the 6th Minor
Irrigation Census
2. Medium tube wells have the highest share in MI schemes followed by Dug wells and
shallow tube wells, and deep tube wells.
3. Maharashtra has the largest number of minor irrigation schemes followed by Uttar
Pradesh
4. 96.6% of minor irrigation schemes are under public ownership.
How many of the above given Statements is/are correct ?
A. Only one B. Only two
C. Only three D. All four
Explanation : A
● The Government of India has been conducting a census of minor irrigation schemes to
create sound and reliable data for minor irrigation schemes for effective policy and
planning in this sector.
● Till now 5 minor irrigation censuses have been done 1986-87, 1993-94, 2000-01, 2006-07
and 2013-14 respectively.
● The 6th minor irrigation census with reference year 2017-18 was completed in 32 States/
UTs.
Findings of the report
● There are 23.14 million minor irrigation schemes in India spread over 695 districts.
● Groundwater accounts for a major share (94.8%) of all minor irrigation schemes in India.
● UP has the largest number of minor irrigation schemes (17.2%) followed by Maharashtra
(15.4%), MP (9.9%) and Tamil Nadu (9.1%).
● 96.6% of minor irrigation schemes are under private ownership whereas 3.4% are under
public ownership.
● Own saving of farmers is the major source of financing for installation of minor irrigation
schemes

117. Consider the following statements regarding the Terai-Arc Landscape( TAL).
1. It is spread across the states of Uttarakhand and Uttar Pradesh Only
2. About 50% of the wild tiger population in India is found across the TAL.
3. This landscape has very less mammalian diversity
4. FAO has identified TAL as the “World Restoration Flagship” for ecosystem restoration
How many of the above given Statements is/are incorrect ?
A. Only one B. Only two
C. Only three D. All four

AMIGOS IAS 67
Explanation : C
Terai Arc Landscape (TAL)
● It is a stretch between the river Yamuna in the west and the river Bhagmati in the east,
comprising the Shivalik hills, the adjoining bhabhar areas and the Terai flood plains.
● It is spread across the Indian states of Uttarakhand, Uttar Pradesh and Bihar, and the low
lying hills of Nepal.
● It has been shortlisted by FAO as the “World Restoration Flagship” as a part of the ongoing
UN Decade for Ecosystem Restoration Campaign (2021-2030).
● About 22% of the wild tiger population in India is found across the TAL, living amidst some
of the highest human and livestock densities on the subcontinent.
● The landscape also has a high mammalian diversity with herbivores including gaur, nilgai,
sambar, northern swamp deer, wild pig, chital, goral, and some primates

118. Direction: The question consists of two statements, one labeled 'Assertion (A)' and the other
labeled 'Reason (R)'. Go through these two statements carefully and select the answer to this
question using the codes given below :
Assertion(A) :
Scientists have known of somatic variants for many years, but recently there has been an
explosion in the amount of data.
Reason(R) :
Scientists have been able to study genes and the functional diversity of the human body’s cells
using advanced microfluidics and high-throughput sequencers which can sequence tens of
thousands of cells from a tissue at the same time.
Options :
A. Both A and R are true and R is the correct explanation of A.
B. Both A and R are true but R is not the correct explanation of A.
C. A is true but R is false.
D. A is false but R is true.
Explanation : A

Somatic Genetic Variants:


● Somatic genetic variants, also known as somatic mutations or somatic changes, refer to
alterations in the DNA sequence that occur in the cells of an individual’s body after
conception.
● Somatic mutations can occur for various reasons, such as errors during DNA replication,
exposure to environmental factors (like radiation or chemicals), or simply as a natural
consequence of cellular processes.
● Somatic mutations can have different effects depending on where they occur in the
genome and which genes are affected.

AMIGOS IAS 68
● Some somatic mutations are harmless and have no discernible impact on the cell or
individual.
● However, others can lead to the development of diseases, including cancer.
Reason for explosion of Data
● Although scientists have known about somatic variations for long, recent advancements
in sequencing genetic matter in individual cells have contributed to a better
understanding.
● Scientists have been able to study genes and the functional diversity of the human body’s
cells using advanced microfluidics and high-throughput sequencers which can sequence
tens of thousands of cells from a tissue at the same time.

119. Consider the following statements with reference to “State Of India’s Birds, 2023” Report.
1. Population of Species like the Indian Peafowl, Rock Pigeon, Asian Koel, and House Crow
has considerably decreased
2. Raptors, migratory shorebirds, and ducks have experienced the most significant Increase
in numbers.
3. Specialist bird species, which inhabit specific habitats like wetlands and rainforests, are
facing rapid declines.
4. Birds with diets focused on vertebrates and carrion have seen notable increase in number.
How many of the above given Statements is/are correct ?
A. Only one B. Only two
C. Only three D. All four
Explanation : A
• The “State of India’s Birds, 2023” report reveals a widespread decline in most bird species
across the country, with some currently declining and others projected to decline in the future.
Findings of the report:
● Raptors, migratory shorebirds, and ducks have experienced the most significant declines.
● However, several species like the Indian Peafowl, Rock Pigeon, Asian Koel, and House
Crow are thriving and increasing in both abundance and distribution.
● Specialist bird species, which inhabit specific habitats like wetlands and rainforests, are
facing rapid declines, while generalist species capable of adapting to various habitats are
faring better.
● Long-distance migratory birds, such as those from Eurasia and the Arctic, have suffered
significant declines, while resident species remain more stable.
● Birds with diets focused on vertebrates and carrion have seen notable declines,
potentially due to harmful pollutants present in these food resources.
● The report emphasizes the decline of species endemic to the Western Ghats and Sri Lanka
biodiversity hotspots.

AMIGOS IAS 69
120. Which of the below given strategies can be considered effective in minimizing Heat Waves in
Urban India ?
1. Having Low-rise areas with sparse vegetation
2. Enhancing Effective vegetation cover (EVC)
3. Having lower Floor Space Index
4. Having more North-south-oriented streets
Select the correct answer from the below given codes :
A. 1,2 and 3 Only B. 3 and 4 Only
C. 2 Only D. 4 Only
Explanation : C
• The study conducted by the Centre for Science and Environment (CSE) explores the impact of
different urban forms on heat resilience.
CSE recommendations include:
1. Morphology Influence:
● Areas with open highrise, open midrise, and compact mid rise urban morphologies show
lower land surface temperatures (LST), indicating better heat resilience.
○ Low-rise areas with sparse vegetation tend to have higher LST.
2. Blue/Green Infrastructure:
● Greenery, like trees, shrubs, and grass, enhances microclimates. Effective vegetation
cover (EVC) reduces LST; trees with thick foliage are more effective.
3. Floor Space Index (FSI):
● Higher FSI correlates with lower LST. Increased FSI results in decreased temperatures.
4. Street Orientation:
● North-south-oriented streets have higher LST due to sun exposure, offering guide-lines
for urban planning.
5. Cooling Solutions:
● Different urban forms can have distinct zoning regulations for heat resilience, such as
shaded walkways, cool roofs, and high EVC.
6. Adapting Urban Plans:
● Cities should amend building by-laws and master plans based on factors influencing heat
gain.

7. Impact on Energy Consumption:


● A 1°C temperature decrease can lead to a 2% reduction in the city’s power consumption.
121. Consider the following statements regarding United Nations Commission on International Trade
Law (UNCITRAL).

AMIGOS IAS 70
1. The UNCITRAL was established by the UN General Assembly (UNGA) in 1966.
2. It has 160 members who are chosen to serve terms of two years.
3. India is a founding member of UNCITRAL
4. India is the only country which has been a member of UNCITRAL since its inception.
How many of the above given Statements is/are correct ?
A. Only one B. Only two
C. Only three D. All four
Explanation : C
• India and Iran drop foreign arbitration clauses in the Chabahar port issue.
UNCITRAL
● The UNCITRAL was established by the UN General Assembly (UNGA) in 1966.
● The UNCITRAL is the core legal body of the United Nations system in the field of
international trade law.
● It works with a mandate to further the progressive harmonization and unification of the
law of international trade.
● UNCITRAL has 60 members who are chosen to serve terms of six years.
● The members are 14 from Asia, 14 from Africa, 10 from Latin America and the Caribbean,
8 from Eastern Europe, and 14 from Western Europe and other countries.
● India – India is a founding member of UNCITRAL and is the only country which have been
a member of UNCITRAL since its inception.

122. Consider the following statements with reference to Aditya L1 Mission.


1. Aditya L1 is the first space-based Indian mission to study the Sun.
2. It will be launched by the PSLV-XL launch vehicle.
3. The spacecraft shall be placed in a halo orbit around the Lagrange point 1 (L1) of the Sun-
Earth system,
4. The spacecraft carries seven payloads to observe the photosphere, chromosphere and
the outermost layers of the Sun (the corona)
How many of the above given Statements is/are correct ?
A. Only two B. Only one
C. Only three D. All four
Explanation : D
Aditya L1
● Aditya L1 is the first space-based Indian mission to study the Sun.
● It will be launched by the PSLV-XL launch vehicle.
● The spacecraft shall be placed in a halo orbit around the Lagrange point 1 (L1) of the Sun-
Earth system, which is about 1.5 million km from the Earth.

AMIGOS IAS 71
● A satellite placed in the halo orbit around the L1 point has the major advantage of
continuously viewing the Sun without any occultation/eclipses.
● This will provide a greater advantage in observing solar activities and their effect on space
weather in real-time.
● The spacecraft carries seven payloads to observe the photosphere, chromosphere and
the outermost layers of the Sun (the corona) using electromagnetic and particle and
magnetic field detectors.

123. Consider the following statements regarding the Near side and Far sides of the Moon.
1. The near side is always the same side that is visible from Earth
2. The ‘full moon’ is the time when the other ‘far side’ of the moon is bathed in sunlight.
3. Astronauts aboard the Apollo 8 mission of 1968 were the first humans to see the far side
of the moon.
4. The near side is relatively smoother and has many more ‘maria’ or large volcanic plains
compared to the far side
How many of the above given Statements is/are correct ?
A. Only one B. Only two
C. Only three D. All four
Explanation : C
● The near side refers to the portion of the moon (about 60%) that is visible to us.
● The ‘dark side’ is always faced away from the Earth.Thus it is dark only in the sense that
it is mysterious and its various topographical features are hidden.
● The ‘new moon’ or when the moon is invisible from Earth is the time when the other ‘far
side’ of the moon is bathed in sunlight and continues to receive light for nearly a fortnight.
● Soviet spacecraft Luna 3 in 1959 photographed it and the Soviet Academy of Sciences
released an atlas of these images.
● Astronauts aboard the Apollo 8 mission of 1968 were the first humans to see the far side
of the moon.
● The major difference between the two sides is that the near side is relatively smoother
and has many more ‘maria’ or large volcanic plains compared to the far side.
● On the far side, there are huge craters, thousands of kilometers wide, which have likely
resulted from collisions with asteroids.
124. Consider the following statements with reference to the G. Padmanabhan Committee.
1. It's aim is to identify measures to encourage new Fintechs to have a global outlook
2. The committee is formed by the GIFT city- IFSC
3. The committee also studies the reasons for shifting of Indian startups outside India.
Which of the above given Statements is/are correct ?
A. 1 and 2 Only B. 2 and 3 Only

AMIGOS IAS 72
C. 1,2 and 3 D. 2 Only
Explanation : C
● International Financial Services Centre Authority (IFSCA) forms a committee to ‘Onshoring
the Indian Innovation to GIFT IFSC’.
● It is chaired by G. Padmanabhan, former executive director, Reserve Bank of India,
focusing on the nuances from regulatory, tax, legal and other perspectives.
○ To identify measures to encourage new Fintechs to have a global outlook to set up their
commercial presence in GIFT IFSC.
Objectives of the Committee:
1. Understanding the reasons for shifting of Indian startups outside India;
2. To avoid externalisation of startups in the future;
3. To persuade startups, currently externalised, to redomicile back;
4. To examine the provisions of the RBI directions pertaining to transfer of stressed loans by
lending institutions to entities in IFSCA;

125. Consider the following statements regarding the NAMASTE Scheme.


1. NAMASTE stands for National Action for Mechanised Sanitation Ecosystem
2. It is the sole initiative of the Ministry of Social Justice and Empowerment (MoSJE).
3. Niti Aayog would be the implementing agency under the scheme.
4. It will be implemented for the period 2022-26
How many of the above given Statements is/are correct ?
A. Only one B. Only two
C. Only three D. All four
Explanation : B

What is NAMASTE Scheme?


● NAMASTE stands for National Action for Mechanised Sanitation Ecosystem.
● It has been launched with an objective to ensure safety and dignity of sanitation workers
in urban India as well as providing sustainable livelihood to these workers.
● Ministries involved: It is launched as a joint initiative of the Ministry of Social Justice and
Empowerment (MoSJE) and the Ministry of Housing and Urban Affairs (MoHUA).
● Implementing agency - National Safai Karamchari Financial Development Corporation
(NSKFDC) would be implementing agency for NAMASTE.
● Coverage: Five hundred cities (converging with AMRUT cities) will be taken up under this
phase of NAMASTE. It will be implemented for the period 2022-26.

126. Consider the following statements regarding The Global Biodiversity Framework Fund (GBFF).

AMIGOS IAS 73
1. It is established by the Seventh Global Environment Facility (GEF) Assembly
2. It will Support the implementation of the Kunming-Montreal Global Biodiversity
Framework.
3. The fund does not recognise the efforts of Indigenous Peoples and local communities in
Biodiversity protection
Which of the above given Statements is/are correct ?
A. 1 Only B. 2 and 3 Only
C. 2 Only D. 1 and 2 Only
Explanation : D
Global Biodiversity Framework Fund (GBF-Fund):
● The Global Biodiversity Framework Fund (GBFF) was ratified and launched at the Seventh
Assembly of the Global Environment Facility (GEF) in Vancouver, Canada.
● The Seventh Global Environment Facility (GEF) Assembly established a Global Biodiversity
Framework Fund (GBF-Fund).
● GBF-Fund is a special trust fund under the GEF, to support the implementation of the
Kunming-Montreal Global Biodiversity Framework agreed at COP15 last year.
● As part of the Kunming-Montreal Global Biodiversity Framework agreement, the GEF was
requested to establish the Global Biodiversity Framework Fund, a special trust fund to
support the implementation of the Agreement.
● The creation of this fund and its commitment to supporting Indigenous Peoples and local
communities is an important and clear recognition of the fundamental role they have had
for generations protecting biodiversity.

127. Consider the following statements regarding Bidriware.


1. It is a form of metal handicraft that has Persian influences
2. The origin of Bidriware as a craft is attributed mostly to the Bahamani Sultans
3. It was first brought to India by the noted Sufi Khwaja Moinuddin Hasan Chisti in the form
of utensils.
Which of the above given Statements is/are correct?
A. 1 and 2 Only B. 3 Only
C. 2 and 3 Only D. 1,2 and 3
Explanation : D
• Indian Prime Minister recently gifted Bidri work pair of 'Surahi' from Telangana to South African
President Cyril Ramaphosa.
Bidriware:
● It is a form of metal handicraft that has Persian influences and has been made for
centuries by artisans from Karnataka’s Bidar district.
● It is renowned for its intricate, handcrafted designs.

AMIGOS IAS 74
● The origin of Bidriware as a craft is attributed mostly to the Bahamani Sultans who ruled
the region during the 14th and 15th centuries.
● It was first brought to India by the noted Sufi Khwaja Moinuddin Hasan Chisti in the form
of utensils.
● The art form developed in the kingdom was a mix of Turkish, Persian and Arabic
influences, which were intermingled with the local styles, and thus a unique style of its
own was born.

128. Consider the following statements regarding Sodium Ion Battery.


1. It can generate electricity through a chemical reaction
2. In a sodium-ion battery, lithium ions are replaced with sodium ions in the battery’s
cathode
3. Sodium batteries ensure high standards of safety as cells based on this chemical element
are neither flammable nor susceptible
Which of the following statements is/are correct ?
A. 1 and 2 Only B. 2 and 3 Only
C. 1 and 3 Only D. 1,2 and 3
Explanation : D
• Recently, AR4 Tech has partnered Sodion Energy of Singapore to make sodium-ion battery
packs for domestic and export markets.
Sodium-Ion Battery:
● These types of batteries generate electricity through a chemical reaction.
● These are made up of an anode, cathode, separator and electrolyte.
● In a sodium-ion battery, lithium ions are replaced with sodium ions in the battery’s
cathode, and lithium salts are swapped for sodium salts in the electrolyte.
● When the battery is being charged, Na atoms in the cathode release electrons to the
external circuit and become ions, which migrate through the electrolyte toward the
anode, where they combine with electrons from the external circuit while reacting with
the layered anode material.
● This process is reversed during discharge.
129. Consider the following statements with reference to Gond Painting.
1. It is a famous folk art of the Gond tribal community of central India.
2. Gond paintings are highly interlinked with nature.
3. The artist uses distinctive styles to make a complete picture, such as dots, fine lines,
curved lines, dashes, fish scales, etc.
Which of the above given Statements is/are correct ?
A. 1 and 2 Only B. 2 and 3 Only
C. 1 and 3 Only D. 1,2 and 3

AMIGOS IAS 75
Explanation : D
• Recently, during the BRICS meeting, the Prime Minister of India gifted a Gond painting to the
Brazilian president Lula da Silva.
Gond Painting:
● It is a famous folk art of the Gond tribal community of central India.
● It is done to preserve and communicate the culture of the Gond tribal community.
● Gond tribes are highly interlinked with nature, and this appears in their paintings too.
They include animals, the mahua tree, mythological stories, Hindu gods, local deities and
folktales, etc.
● The artist uses his distinctive pattern and style to fill the images. These style signatures
are used in collages to make a complete picture, such as dots, fine lines, curved lines,
dashes, fish scales, etc.

130. Consider the following statements regarding AUSINDEX-23.


1. It is a major biennial maritime exercise, being conducted since 2015.
2. This is the first edition of the biennial AUSINDEX maritime exercise between the Indian
Navy and the Royal Australian Navy (RAN)
3. This year’s edition was held in India.
Which of the above given Statements is/are correct ?
A. 1 and 2 Only B. 2 Only
C. 2 and 3 Only D. 1 Only
Explanation : D
• Recently, the fifth edition of the biennial AUSINDEX maritime exercise between the Indian Navy
and the Royal Australian Navy (RAN) was conducted from 22-25 August 23 in Sydney.
AUSINDEX-23:
● It is a major biennial maritime exercise, being conducted since 2015.
● The exercise will provide an opportunity for both Navies to further bolster inter-
operability, gain from best practices and develop a common understanding of procedures
for Maritime Security Operations.
● This year’s edition was held in Australia.
● INS Sahyadri and INS Kolkata participated in the exercise along with HMAS Choules and
HMAS Brisbane from RAN.
● Besides ships and their integral helicopters, the exercise also witnessed the participation
of fighter aircraft and maritime patrol aircraft.

131. Consider the following statements :


Statement-I :

AMIGOS IAS 76
A joint report by the World Health Organization (WHO) and the Indian Council for Research on
International Economic Relations (ICRIER) has drawn attention to a substantial and troubling
surge in the sales of ultra-processed foods in India.
Statement-II :
Ultra-processed foods typically contain maximum natural ingredients and are characterized by
their inconvenience, short shelf life, and often addictive taste profiles
Which one of the following is correct in respect of the above statements?
A. Both Statement-I and Statement-II are correct and Statement-II is the correct explanation
for Statement-I
B. Both Statement-I and Statement-II are correct and Statement-II is not the correct
explanation for Statement-I
C. Statement-I is correct but Statement-II is incorrect
D. Statement-I is incorrect but Statement-II is correct
Explanation : C
• A report jointly released by the World Health Organisation (WHO) and the Indian Council for
Research on International Economic Relations (ICRIER) highlights a significant and concerning
increase in the sales of ultra-processed foods in India over the past decade.
Statement-I is correct .
Ultra-processed foods:
● Ultra-processed foods are a category of food products that have undergone multiple
stages of processing, often involving the addition of various artificial ingredients, such as
preservatives, colours, flavours, and additives.
● These foods typically contain minimal whole or natural ingredients and are characterized
by their convenience, long shelf life, and often addictive taste profiles.
Therefore, Statement-II is incorrect .

132. Consider the following statements regarding the protection of Tigers in India.
1. India has seen an increase in tiger population, with an annual growth rate of 6% between
2018-22.
2. Madhya Pradesh has the highest number of tigers followed by Karnataka.
3. India has established its 54 th tiger reserve in the Karauli and Dholpur districts of
Rajasthan recently.

How many of the above given Statements is/are incorrect ?


A. Only one B. Only two
C. All three D. None of them
Explanation : D
● Over recent years, India has seen an increase in tiger population, rising from 2,967 in 2018
to 3,682 in 2022, signifying an annual growth rate of 6%. Therefore statement 1 is correct.

AMIGOS IAS 77
● Notably, Madhya Pradesh has the highest number of tigers (785), followed by Karnataka
(563), Uttarakhand (560), and Maharashtra (444). Statement 2 is correct.
● India has established its 54th tiger reserve in the Karauli and Dholpur districts of
Rajasthan, following approval from the National Tiger Conservation Authority (NTCA).
Hence, statement 3 is correct.
● This new reserve marks Rajasthan’s fifth, joining the existing reserves of Ranthambore,
Sariska, Mukundra Hills, and Ramgarh Vishdhari.
133. The term refers to a chronic fear or worry about environmental issues and the potential negative
impacts of climate change. This emotional response can lead to feelings of helplessness, sadness,
and fear about the Earth’s future. The term that has been in the news recently is ?
A. Green Stress B. Nature-Induced Stress
C. Eco-Concern D. Eco- Anxiety
Explanation : D
Eco-anxiety
● Eco-anxiety refers to a chronic fear or worry about environmental issues and the potential
negative impacts of climate change.
● It is characterized by a sense of distress and anxiety related to the state of the planet’s
environment, including concerns about the future well-being of ecosystems, wildlife, and
human populations due to factors like pollution, climate change, and natural disasters.
● This emotional response can lead to feelings of helplessness, sadness, and fear about the
Earth’s future.

134. Consider the following statements with reference to Minimum Export Price.
1. The government fixes MEP for the selected commodities to arrest domestic price rises
and augment domestic supply.
2. MEP was first implemented on basmati rice in FY23 to deter exports
3. MEP does not have any legal backing in India.
How many of the above given Statements is/are incorrect ?
A. All One B. Only three
C. Only two D. None of them
Explanation : C
• The Central government of India may be considering a minimum export price (MEP) for a host
of speciality rice, including the basmati variety, to ensure that unscrupulous traders don’t ship
excess quantities abroad.
Minimum Export Price:
● It is the price below which an exporter is not allowed to export the commodity from India.
● It is imposed in view of the rising domestic retail/wholesale price or production
disruptions in the country.

AMIGOS IAS 78
● The government fixes MEP for the selected commodities to arrest domestic price rises
and augment domestic supply. Statement 1 is correct.
● This is intended to be imposed for short durations and is removed when situations
change.
● MEP was first implemented on basmati rice in FY11 to deter exports and is typically
implemented to contain surging domestic prices because of production disruptions. So,
statement 2 is incorrect.
● As per section 5 of the Foreign Trade (Development And Regulation) Act, 1992, the
Central Government may, from time to time, formulate and announce, by notification in
the Official Gazette, the export and import policy and may also, in the like manner, amend
that policy. Hence statement 3 is incorrect.

135. Consider the following statements with respect to the


Provisions of the Motor Vehicle Rules.
1. As per the rules “Stickers and adhesive labels” in registration number plates are
permitted.
2. The number plate ‘shall be a solid unit made of 1.0 mm aluminum’ and ‘should bear the
letters “IND” in blue color on the extreme left center’.
3. If the number plate is not as per the rules,Section 192 of the Act, has the provision of a
fine of up to Rs 5,000 for the first offense.
How many of the above given Statements is/are incorrect ?
A. Only one
B. Only two
C. All three
D. None of them
Explanation : A
● The Noida and Ghaziabad Police, over the past few days, issued challans to people for
putting caste and religious stickers on their cars.
○ The orders are issued as per The Motor Vehicle Act 1988 and Motor Vehicle Rules 1989.
● According to the Motor Vehicle Rules, “stickers and adhesive labels” in registration
number plates are not permitted.
● Specifications of the number plate: According to the rule, the number plate ‘shall be a
solid unit made of 1.0 mm aluminum’ and ‘should bear the letters “IND” in blue color on
the extreme left center’.
● If the number plate is not as per the rules,Section 192 of the Act, has the provision of a
fine of up to Rs 5,000 for the first offense. For subsequent offenses, they can be
imprisoned for up to 1 year and fined up to rupees 10,000.

136. Consider the following statements regarding Intended Actions for Chandrayaan-3 after its
Landing.

AMIGOS IAS 79
1. It is expected to operate for at least ten lunar days on the lunar surface.
2. The Pragyan rover will move around the landing site within a radius of 5 kilometres
3. The Vikram lander will relay the data and images to the orbiter, which will then transmit
them to Earth.
Which of the above given Statements is/are incorrect ?
A. 1 and 3 Only B. 2 Only
C. 1 Only D. 1 and 2 Only
Explanation : D
● Chandrayaan-3 is expected to operate for at least one lunar day (14 Earth days) on the
lunar surface.
● The Pragyan rover will move around the landing site within a radius of 500 meters,
conducting experiments and sending data and images to the lander.
● The Vikram lander will relay the data and images to the orbiter, which will then transmit
them to Earth.

137. Consider the following regarding the key features of the National Curriculum Framework (NCF).
1. The NCF introduces changes in language learning, subject structure, evaluation strategies,
reshaping the educational landscape for Grades 3 to 12 under CBSE.
2. Students in Classes 9 and 10 learn three languages, with at least two being native Indian
languages
3. Students are allowed to take Board exams on at least two occasions in a school year.

How many of the above given Statements is/are correct ?


A. Only one
B. Only two
C. All three
D. None of them
Explanation : C
● The NCF introduces changes in language learning, subject structure, evaluation strategies,
and environmental education, reshaping the educational landscape for Grades 3 to 12
under CBSE.
● Students in Classes 9 and 10 learn three languages, with at least two being native Indian
languages.
● In Classes 11 and 12, students will study two languages, including one of Indian origin.
● Aim to achieve a "literary level" of linguistic capacity in at least one Indian language.
● Students are allowed to take Board exams on at least two occasions in a school year.
○ Only the best score among attempts will be retained.

AMIGOS IAS 80
138. Consider the following statements regarding the ASTRA Missile.
1. It is an indigenous fire and forget Surface-to-air missile
2. It will be able to destroy highly manoeuvring supersonic aerial targets.
3. It is designed and developed by the Defence Research and Development Laboratory
(DRDL), Research Centre Imarat (RCI) etc.
Which of the above given Statements is/are correct ?
A. 1 and 2 Only
B. 2 and 3 Only
C. 1 and 3 Only
D. 1,2 and 3
Explanation : B
• Recently, Tejas, Light Combat Aircraft (LCA) LSP-7 successfully fired the ASTRA off the coast of
Goa on August 23, 2023.
ASTRA Missile:
● It is an indigenous Beyond Visual Range (BVR) air-to-air missile.
● It is to engage and destroy highly manoeuvring supersonic aerial targets.
● It is designed and developed by the Defence Research and Development Laboratory
(DRDL), Research Centre Imarat (RCI) and other laboratories of DRDO.
● The indigenous Astra BVR firing from homegrown Tejas fighters is a major step towards
‘Aatmanirbhar Bharat’.

139. “Microsites project” that has been in the news recently is related to which of the following?
A. Education B. Cyber Security
C. Healthcare D. Biodiversity Conservation
Explanation : C
• Recently, the National Health Authority (NHA) announced 100 Microsites projects for
accelerated adoption of the Ayushman Bharat Digital Mission (ABDM) across the country.
Microsites project:
● These are defined geographical regions where focused outreach efforts would be made
to onboard small and medium-scale private healthcare providers.
● Types of facilities to be covered under a Microsite:
○ Private facilities such as standalone clinics, polyclinics, nursing homes, small hospitals
(preferably <10 beds), labs, pharmacies, and any other healthcare facilities wherein
health records are generated.
○ Health Facilities and Health Professionals from all the systems of medicine.

140. This is a holistic approach to problems that recognises the interconnections between the health
of humans, animals, plants, and their shared environment. An early articulation of this idea can

AMIGOS IAS 81
be found in the writings of Hippocrates (460-367 BC), who contemplated the relationships
between public health and clean environments.
The concept being discussed in the above paragraph is ?
A. Unified Health Approach B. Comprehensive Health & Wellness
C. Whole Health D. One Health
Explanation : D
• India is currently preparing for a ‘National One Health Mission’ which can help India respond
better to health crises.

One Health
● One Health is a holistic approach to problems that recognises the interconnections
between the health of humans, animals, plants, and their shared environment.
● An early articulation can be found in the writings of Hippocrates (460-367 BC), who
contemplated the relationships between public health and clean environments.

141. Consider the following statements regarding ASEAN-India Trade in Goods Agreement (AITIGA).
1. It was signed by ASEAN and India in Bangkok, Thailand in 2009.
2. The Agreement has led to steadily increasing trade between ASEAN and India since its
signing.
3. Recently, India and ASEAN agreed to review the FTA by 2025.
Which of the above given Statements is/are correct ?
A. 1 and 2 Only B. 3 Only
C. 1 Only D. 1,2 and 3
Explanation : D
AITIGA
● The ASEAN-India Trade in Goods Agreement (the Agreement) is a trade deal between the
ten member states of ASEAN and India.
● ASEAN and India signed the Agreement at the 7th ASEAN Economic Ministers-India
Consultations in Bangkok, Thailand in 2009.
● The Agreement, which came into effect in 2010, is sometimes referred to as the ASEAN-
India Free Trade Agreement.
● The Agreement has led to steadily increasing trade between ASEAN and India since its
signing.
● In 2019-20, trade between India and ASEAN was worth US$86 billion.
● Though this represented a decline from US$97 billion in 2018-19 because of the COVID-
19 pandemic, it was an increase from US$81.3 billion in the 2017-18 financial year.
142. Which of the following is correct with reference to HD 45166 ?

AMIGOS IAS 82
1. It is a giant helium-rich star that is several times larger than the sun and is located 3,000
light years away from Earth.
2. It has the most powerful magnetic field ever detected in a massive star, 43,000 gauss.
3. The star is also part of a binary system
Choose the correct answer from the below given Codes :
A. 1 and 2 Only B. 2 and 3 Only
C. 1 and 3 Only D. 1,2 and 3
Explanation : D
HD 45166
● A Star, HD 45166, that could become the strongest magnet in the universe has been
discovered by astronomers.
● HD 45166, is a giant helium-rich star that is several times larger than the sun and is located
3,000 light years away from Earth.
● It was spotted from several telescopes from all over the planet.
● HD 45166 has a remarkable feature: it has the most powerful magnetic field ever detected
in a massive star, 43,000 gauss, to be precise.
● The star is also part of a binary system, meaning that it has another star orbiting around
it.

143. Consider the following statements:

Statement-I :
The Hubble Space Telescope recently captured an image of the irregular galaxy ESO 300-16,
located at a distance of 28.7 million lightyears in the southern constellation of Eridanus.

Statement-II :
Irregular galaxies born from galaxy interactions or collisions typically host a Only younger stars.

Which one of the following is correct in respect of the above statements?


A. Both Statement-I and Statement-II are correct and Statement-II is the correct explanation
for Statement-I
B. Both Statement-I and Statement-II are correct and Statement-II is not the correct
explanation for Statement-I
C. Statement-I is correct but Statement-II is incorrect
D. Statement-I is incorrect but Statement-II is correct

AMIGOS IAS 83
Explanation : C
Irregular Galaxy:

● An irregular galaxy is a galaxy that does not have a distinct regular shape, like a spiral or
an elliptical galaxy.
● They range from dwarf irregular galaxies with 100 million times the Sun’s mass to large
ones weighing 10 billion solar masses.
● They also contain abundant amounts of gas and dust.
● There are many ways in which an irregular galaxy can be formed.
○ For example, one can result from a collision between galaxies. When this happens,
gravitational forces between separate galaxies interact, which causes an irregular type of
rotation.
● A young galaxy can also take an irregular form, suggesting that it has not yet reached a
symmetrical rotation.
● Irregular galaxies born from galaxy interactions or collisions typically host a mix of older
and younger stars.

144. Consider the following statements with reference to North East Special Infrastructure
Development Scheme (NESIDS).
1. It was approved by the Government of India as a Central Sector Scheme in 2017.
2. Under the scheme guidelines of NESIDS, 100% of the central funding is provided to the
State Governments of North Eastern Region
3. The Scheme covers water supply, power, and connectivity, enhancing tourism, and Social
infrastructure etc.
Which of the above given Statements is/are correct ?
A. 1 Only B. 3 Only
C. 2 Only D. 1,2 and 3
Explanation : D
• Recently, the cabinet gave approval for the continuation of the North East Special Infrastructure
Development Scheme (NESIDS) with an approved outlay of Rs. 8139.50 crore.
North East Special Infrastructure Development Scheme (NESIDS):
● It was approved by the Government of India as a Central Sector Scheme in 2017.
● Under the scheme guidelines of NESIDS, 100% of the central funding is provided to the
State Governments of the North Eastern Region.
● The funding is provided for projects of physical infrastructure relating to water supply,
power, and connectivity, enhancing tourism, and Social infrastructure relating to primary
and secondary sectors of education and health.
● For the period from 2022-23 to 2025-26, Cabinet approved two components, , NESIDS-
Road and NESIDS-Other Than Road Infrastructure (OTRI).

AMIGOS IAS 84
● The scheme is a central sector scheme with 100% central funding.

145. Consider the following statements with reference to Onion Production in India.
1. India is the largest onion-growing country in the world.
2. Onions are more commonly associated with the Kharif season.
3. The crop requires well-drained soil and is often cultivated in sandy or loamy soils.
4. Karnataka ranks first in Onion production.
How many of the above given Statements is/are correct ?
A. Only one B. Only two
C. Only three D. All four
Explanation : A
• After a long slump, onion prices are again on the rise, causing concern for the central
government. As a result, the Ministry of Finance imposed a 40 per cent export duty on onions.

Onion Production in India


● India is the second-largest onion-growing country in the world.
● Onions are more commonly associated with the Rabi season.
● Onions are primarily grown through seeds, although bulb-to-bulb planting is also
practiced.
● The crop requires well-drained soil and is often cultivated in sandy or loamy soils.
● Adequate irrigation is essential, and farmers may use drip irrigation systems to conserve
water.
● The Major Onion producing states are Maharashtra, Karnataka, Madhya Pradesh, Gujarat,
Bihar, Andhra Pradesh, Rajasthan, Haryana and Telangana.
● Maharashtra ranks first in Onion production

146. Consider the following statements with reference to the Maritime State Development Council
(MSDC).
1. It is an apex advisory body constituted in 1997 to develop the Maritime Sector.
2. It aims to ensure the integrated development of Major and other notified ports.
Which of the above given Statements is/are correct ?
A. 1 Only B. 2 Only
C. Both 1 and 2 D. Neither 1 nor 2
Explanation : C
• The Union Minister for Ports, Shipping and Waterways (MoPSW), unveiled a vision for India's
maritime sector, outlining key initiatives that promise a transformative impact at the 19th
Maritime States Development Council meeting.

AMIGOS IAS 85
Maritime State Development Council (MSDC)
● It is an apex advisory body constituted in 1997 to develop the Maritime Sector.
● It aims to ensure the integrated development of Major and other notified ports.

147. Consider the following statements with reference to World Water Week 2023.
1. It is a global event organised by the Stockholm International Water Institute every year
since 1991.
2. Current theme of the event is "Seeds of Change: Innovative Solutions for a Water-Wise
World".
Which of the above given Statements is/are incorrect ?
A. 1 Only B. 2 Only
C. Both 1 and 2 D. Neither 1 nor 2
Explanation : D
• An online session in World Water Week 2023 recently organised on the theme ‘Peer
Networking for Integrated River Basin Planning and Management’ with an extensive discussion
on the Namami Gange Programme.
About World Water Week
● It is a global event organised by the Stockholm International Water Institute every year
since 1991.
● It serves as a platform for experts, policymakers, and organizations to discuss and
collaborate on global water issues.
● The 2023 edition of the world water week is held in the Waterfront Congress centre,
Stockholm with the theme, "Seeds of Change: Innovative Solutions for a Water-Wise
World".
148. Which of the following statements is/are correct regarding the RBI Study on Investment Trends in
India for FY 2022-23.
1. Madhya Pradesh got the highest share of 16.2% in the total cost of projects sanctioned
by banks and financial institutions.
2. The Automobile sector played a pivotal role, accounting for 60% of the total project cost
in 2022-23.
3. Kerala has the lowest score receiving just 0.9% of the total investment plans.
How many of the above given Statements is/are correct ?
A. Only one B. Only two
C. All three D. None of them
Explanation : A

AMIGOS IAS 86
State-wise Share in Total Project Costs:
● Uttar Pradesh emerges as the frontrunner, accounting for the highest share of 16.2% in
the total cost of projects sanctioned by banks and financial institutions.
● Following closely are Gujarat (14%), Odisha (11.8%), Maharashtra (7.9%), and Karnataka
(7.3%), showcasing a dynamic distribution of investments.
● Kerala, Goa, and Assam secured the lowest shares, with Kerala receiving just 0.9% of the
total investment plans.
● Haryana and West Bengal also fell within the 1% bracket of the total investment projects.
● The infrastructure sector played a pivotal role, accounting for 60% of the total project
cost in 2022-23.
● Notably, road and bridge projects within the infrastructure sector garnered significant
attention, benefiting from the "Bharatmala" initiative.

149. Consider the following statements with reference to Chess World Cup 2023.
1. The World Cup 2023 is organised by the International Chess Federation (FIDE), the
governing body of chess in the world.
2. Rameshbabu Praggnanandhaa won over World No. 3 Fabiano Caruana in the tie-breaker
round of the World Cup 2023, chess tournament
3. Tajikistan is hosting the current world cup in rotation.
How many of the above given Statements is/are correct ?
A. 1 and 2 Only B. 3 Only
C. 2 and 3 Only D. 2 Only
Explanation : A
● Grandmaster Rameshbabu Praggnanandhaa wins over World No. 3 Fabiano Caruana in
the tie-breaker round of the World Cup 2023, chess tournament at Baku, Azerbaijan.
● The World Cup 2023 is organised by the International Chess Federation (FIDE), the
governing body of chess in the world.
○ It's constituted as a non-governmental institution.
● Praggnanandhaa, the youngest Indian grandmaster and the youngest international
master is among the most promising talents in the world of chess.

150. Consider the following statements regarding the Green Hydrogen standard of India.
1. Green Hydrogen should have a well-to-gate emission of not exceeding 2 kg carbon dioxide
(CO2) equivalent per kg Hydrogen(H2), taken as an average over the last 12-month
period.
2. The well-to-gate emission does not include water treatment, electrolysis, gas purification,
drying and compression of hydrogen.

AMIGOS IAS 87
3. The Bureau of Energy Efficiency (BEE), Ministry of Power, will serve as the Nodal Authority
for accrediting agencies overseeing monitoring, verification, and certification of green
hydrogen production projects.
How many of the above given Statements is/are correct ?
A. Only one B. Only two
C. All three D. None of them
Explanation : B
● The Ministry of New and Renewable Energy (MNRE) recently defined a clear Green
Hydrogen Standard, which establishes emission thresholds for hydrogen production
categorized as 'green'.
● The MNRE has determined that Green Hydrogen should have a well-to-gate emission of
not exceeding 2 kg carbon dioxide (CO2) equivalent per kg Hydrogen(H2), taken as an
average over the last 12-month period.
● The well-to-gate emission includes water treatment, electrolysis, gas purification, drying
and compression of hydrogen.
● The MNRE will specify a detailed methodology for measuring, reporting, monitoring, on-
site verification, and certification of green hydrogen and its derivatives.
● The Bureau of Energy Efficiency (BEE), Ministry of Power, will serve as the Nodal Authority
for accrediting agencies overseeing monitoring, verification, and certification of green
hydrogen production projects.

151. Consider the following statements with reference to Palm cockatoo.


1. It is native to the Indian Subcontinent
2. They occur in rainforests, paperbark woodlands, monsoon woodlands etc.
3. It is under threat by habitat loss through logging and seasonal fires.
4. It's Conservation status as per IUCN red list is Endangered
How many of the above given Statements is/are correct ?
A. Only one B. Only two
C. Only three D. None of them
Explanation : B
● Recently, six Palm cockatoos (Probosciger aterrimus) birds– were rescued from Assam’s
Cachar district by the police.
Palm cockatoo:
● It is also known as the goliath cockatoo or great black cockatoo, is a large smoky-grey or
black parrot of the cockatoo family.
● It has a very large black beak and prominent red cheek patches.
● It is native to New Guinea, Aru Islands, and Cape York Peninsula.
● They occur in rainforests, such as gallery forests, forest edges, eucalypt and paperbark
woodlands, monsoon woodlands, dense savannas and partly cleared areas.

AMIGOS IAS 88
● They choose big trees for roosting and nesting.
● It is under threat by habitat loss through logging and seasonal fires, which each year
destroy their nest trees in significant numbers.
● Conservation status - IUCN: Least Concern

152. Consider the following statements regarding TAPAS UAV.


1. It is a long-endurance unmanned aerial vehicle.
2. It was previously referred to as Indrajaal-II.
3. It has a range of over 100 hours and altitude capabilities of up to 48,000 feet.
4. It is being developed in India by Aeronautical Development Establishment.
How many of the above given Statements is/are correct ?
A. Only one B. Only two
C. Only three D. All four
Explanation : B
• Recently, a Tapas unmanned aerial vehicle (UAV) of the Defence Research and Development
Organisation (DRDO) crashed in agriculture fields near a village in Karnataka's Chitradurga.

TAPAS UAV:
● The Tactical Airborne Platform for Aerial Surveillance-Beyond Horizon-201 or Tapas BH-
201 is a long-endurance unmanned aerial vehicle.
● It was previously used as Rustom-II.
● It has a range of over 18 hours and altitude capabilities of up to 28,000 feet.
● It is being developed in India by Aeronautical Development Establishment.
● Its compact, lightweight design makes it easily transportable and deployable to remote
locations.

153. Consider the following statements with respect to Kashmir stag.


1. It is a subspecies of Central Asian red deer endemic to Kashmir and surrounding areas.
2. Till today, its population has shown signs of decreasing consistency over the last two
decades.
3. In Kashmir, it is found primarily in the Dachigam National Park Only
4. It is a Vulnerable species as per the IUCN Red list
How many of the above given Statements is/are incorrect ?
A. Only one B. Only two
C. Only three D. All four
Explanation : A

AMIGOS IAS 89
• After two years, the unique Kashmir stag (Hangul) the state animal of Jammu and Kashmir, has
again shown a marginal increase in its population.
Kashmir stag:
● It is also called Hangul which is a subspecies of Central Asian red deer endemic to Kashmir
and surrounding areas.
● It is found in dense riverine forests in the high valleys and mountains of Jammu and
Kashmir and northern Himachal Pradesh.
● In Kashmir, it is found primarily in the Dachigam National Park where it receives
protection.
● A small population has also been witnessed in Overa-Aru Wildlife Sanctuary in south
Kashmir. Conservation status IUCN: Critically Endangered
○ CITES : Appendix I
154. Which among the following can be considered as important sources of Mercury Pollution ?
1. Erosion of rocks
2. Artisanal gold mining operations
3. Cement manufacturing
4. Chlorine production
5. Proper disposal of e-waste
Select the correct answer from the below given codes :
A. 1, 2, 4 and 5 Only B. 2, 3, 4 and 5 only
C. 1,2 and 3 Only D. 1,2,3 and 4 Only
Explanation : D
● The sixth anniversary of the Minamata Convention on Mercury is a reminder of global
efforts to combat the toxic effects of mercury.
Sources of Mercury Pollution:
Natural Sources:
● Volcanic eruptions release small amounts of mercury.
● Erosion of rocks and soil can release mercury into water bodies.
Anthropogenic Sources:
● Artisanal gold mining operations are responsible for 37% of global mercury pollution.
● Various industries, such as chlorine production, cement manufacturing, and waste
incineration, emit mercury.
○ The cement industry is responsible for around 11% of global anthropogenic mercury
emissions.
● Improper disposal of e-waste products containing mercury, such as fluorescent bulbs and
batteries, leads to mercury leaching into the environment.

155. Consider the following statements with respect to Acoustic Side Channel Attacks (ASCA).

AMIGOS IAS 90
1. In an ASCA, the sound of clicks generated by a keyboard is used to analyse keystrokes and
interpret what is being typed.
2. ASCA attacks are new to the world , a reason why it is taking time to get over.
3. Using touch-based typing can not also solve side channel attacks.
Which of the above given Statements is/are incorrect ?
A. 1 and 2 Only B. 3 Only
C. 1 Only D. 2 and 3 Only
Explanation : D

ASCA
● In an ASCA, the sound of clicks generated by a keyboard is used to analyse keystrokes and
interpret what is being typed to leak sensitive information.
● These attacks are particularly dangerous as the acoustic sounds from a keyboard are not
only readily available but also because their misuse is underestimated by users.
● While there is no explicit means of defence against ASCAs, simple changes to typing could
reduce the chances of attacks.
● ASCA attacks are not new and have been around since 1950 when acoustic emanations
of encryption devices were used to crack their security.
● Using touch-based typing can also reduce the chances of successful keystroke recognition
from 64% to 40%, making it more difficult for threat actors to leak sensitive information.

156. The term “Thoppikkallu” has been in the news recently. It is related to ?
A. Buddhist viharas B. Jain Basadis
C. Megalithic hat stones D. Paleolithic Caves
Explanation : C
● A large number of megalithic hat stones were found from a single site during a recent
archaeological salvage excavation conducted by the Kerala State Archaeology
Department at Nagaparamba in Kuttippuram village, near Tirunavaya.
● Hat stones, popularly called Thoppikkallu in Malayalam, are hemispherical laterite stones
used as lid on burial urns during the megalithic period are fond here.
● The findings may throw light on the life and culture of people who lived in those parts
more than 2,000 years ago.

157. Consider the following statements regarding the Global Initiative on Digital Health (GIDH).
1. It is a World Health Organization managed network.
2. The GIDH will aim to align efforts to support the Global Strategy on Digital Health 2020–
2025

AMIGOS IAS 91
3. The WHO and India’s G20 presidency collaboratively introduced the ‘Global Initiative on
Digital Health’ (GIDH).
Which of the above given Statements is/are correct ?
A. 1 and 2 Only B. 3 Only
C. 2 and 3 Only D. 1, 2 and 3
Explanation : D
● The WHO and India’s G20 presidency collaboratively introduced the ‘Global Initiative on
Digital Health’ (GIDH).
● It is a World Health Organization (WHO)-managed network, with the aim of consolidating
the evidence and amplifying the gains in global digital health.
● The GIDH will promote equitable access to digital health by addressing challenges such as
duplication of efforts and “products-focused” digital health transformation.
● The GIDH will aim to align efforts to support the Global Strategy on Digital Health 2020–
2025 (an ongoing WHO project).

158. This lotus has 108 petals and was discovered several years ago in Manipur and kept at the
institute as part of its collection of flowers and plants, on which the institute conducts research.
However, it was not until four years ago that one of the scientists discovered that it had 108
petals. This is the only lotus variety in India to have had its genome sequenced. Which of the
following is that lotus variety.
A. Green Maiden B. Blue Star
C. Curious Lotus D. Namoh 108
Explanation : D
● The Science Minister unveiled a variety of lotus called ‘Namoh 108’.
● The lotus has 108 petals and was discovered several years ago in Manipur and kept at the
institute as part of its collection of flowers and plants, on which the institute conducts
research.
● However, it was not until four years ago that one of the scientists discovered that it had
108 petals.
● This is the only lotus variety in India to have had its genome sequenced.
● It was only after the number of petals was discovered that the National Botanical
Research Institute (NBRI) cloned and worked on improving its germplasm and modifying
its characteristics.
● The name of the variety was given by an internal committee of the NBRI.

159. Consider the following statements regarding the status of the horticulture sector in India?
1. Fruits and vegetables alone account for almost 90% of the total horticulture production
in the country.
2. More than 50% of the floriculture products are produced in Karnataka, Andhra Pradesh,
Tamil Nadu and Madhya Pradesh.

AMIGOS IAS 92
3. Horticulture contributes 30.4% of the agriculture Gross Domestic Product (GDP).
4. The global horticulture market of India is at mere 10 %.
How many of the above given Statements is/are correct ?
A. Only one B. Only two
C. Only three D. All four
Explanation : C
• Though the production and productivity of fruits and vegetables quadrupled during the last
three decades, the same has not been translated into comparable increases in the incomes of
the cultivators.
• Fruits and vegetables alone account for almost 90% of the total horticulture production in the
country.
• According to FAO 2021, India leads in the production of certain vegetables (ginger and okra)
and fruits (banana, mangoes and papaya).
• More than 50% of the floriculture products are produced in Karnataka, Andhra Pradesh and
Tamil Nadu and Madhya Pradesh.
• Horticulture contributes 30.4% of the agriculture Gross Domestic Product (GDP) using only
13.1% of gross cropped area.
• India is ranked 14th in vegetables and 23rd in fruits.
• However, the global horticulture market of India is at mere 1%.
160. Consider the following statements regarding the provision of e-services in the Education Sector of
India.
1. Karnataka provides the maximum e-services in Education sector.
2. Madhya Pradesh provides the maximum types of e-services in education.
3. The Government of Andhra Pradesh is the first State in the country to use online system
for allotment of 100% seats through their Right to Education (RTE) portal
4. G-Shala portal is developed by the Delhi State Government
How many of the above given Statements is/are incorrect ?
A. Only one B. Only two
C. Only three D. All four
Explanation : D
● Department of Administrative Reforms & Public Grievances (DARPG) has released the
fifth edition of ‘National e-Governance Service Delivery Assessment (NeSDA) – Way
Forward Monthly Report for States/UTs’, which provides a detailed overview of status of
e-service delivery across States/UTs.
● Madhya Pradesh provides the maximum e-services in Education sector (122), which is
followed by Karnataka (113), Kerala (92), Gujarat (69), and Himachal Pradesh (55)
● Out of 25 types of distinct e-services identified under Education sector, Karnataka (21)
provides the maximum types of e-services. This is followed by Kerala (17) and Madhya
Pradesh (16)

AMIGOS IAS 93
● The Government of Gujarat has developed G-Shala (Gujarat Students’ Holistic Adaptive
Learning App) which is a Learning Management System (LMS) with e-Content for Grade I-
XII
● The Government of Madhya Pradesh is the first State in the country to have used online
and centralized system for allotment of 100% seats through their Right to Education (RTE)
portal.

161. Consider the following statements with reference to Infrastructure Debt Fund - Non-Banking
Financial Company (IDF - NBFC).
1. It is a deposit taking NBFC permitted to Refinance infrastructure projects that have
completed at least 10 years of commercial operations
2. It is not allowed to Finance Toll-Operate-Transfer (TOT) projects as a direct lender.
Which of the above given Statements is/are incorrect?
A. 1 Only
B. 2 Only
C. Both 1 and 2
D. Neither 1 nor 2
Explanation : C
Infrastructure Debt Fund - Non-Banking Financial Company (IDF - NBFC)
● RBI revised the regulatory framework for IDF-NBFCs.
● IDF-NBFCs will not need sponsors but will be required to have net-owned funds of at least
₹300 crore; capital-to-risk weighted assets ratio (CRAR) of 15% .
● They will be allowed to raise funds through rupee or dollar-denominated bonds with at
least a five-year maturityIDF-NBFC is a non-deposit taking NBFC permitted to:
○ Refinance infrastructure projects that have completed at least one year of commercial
operations
○ Finance Toll-Operate-Transfer (TOT) projects as direct lender.

162. Consider the following statements with reference to Magnetar.


1. A magnetar is an exotic type of neutron star its defining feature that it has an ultra-
powerful magnetic field.
2. Apart from ultra-powerful magnetic fields, magnetars also release vast amounts of energy
in the form of flares, X-rays, and gamma-ray bursts.
Which of the above given Statements is/are correct ?
A. 1 Only B. 2 Only
C. Both 1 and 2 D. Neither 1 nor 2
Explanation : C
Magnetar:

AMIGOS IAS 94
● A magnetar is an exotic type of neutron star, its defining feature that it has an ultra-
powerful magnetic field.
● The field is about 1,000 times stronger than a normal neutron star and about a trillion
times stronger than the Earth’s.
● Apart from ultra-powerful magnetic fields, magnetars also release vast amounts of energy
in the form of flares, X-rays, and gamma-ray bursts.
● They are therefore associated with extreme events in the universe, making them perhaps
the most bizarre objects in the cosmos next to black holes.
● The magnetic field of a magnetar may be caused by a neutron star’s interior – thought to
be made up of neutrons, quarks and exotic states of matter such as Bose-Einstein
Condensates – becoming a superconducting fluid.
● Thus, when the star rotates, it would behave like a huge dynamo, generating an immense
magnetic field.

163. Consider the following statements regarding National Automated Fingerprint Identification
System (NAFIS).
1. It is a nationwide database managed by NCRB
2. It swiftly stores crime-related fingerprints and assigns a unique 10-digit National
Fingerprint Number (NFN)
3. NFN remains with the offender for life, connecting various crimes under different FIRs to
the same ID.
Which of the above given statements is/are correct?
A. 1 and 2 Only B. 3 Only
C. 2 and 3 Only D. 1,2 and 3
Explanation : D
● The team of National Automated Fingerprint Identification System (NAFIS) of National
Crime Records Bureau (NCRB) for winning the Gold Award under the Excellence in
Government Process Reengineering for Digital Transformation Category-1 of Department
of Administrative Reforms and Public Grievances (DARPG).
● NAFIS, a nationwide database managed by NCRB, swiftly stores crime-related fingerprints
and assigns a unique 10-digit National Fingerprint Number (NFN) to apprehended
criminals within 24 hours.
● NFN remains with the offender for life, connecting various crimes under different FIRs to
the same ID.
● Operates from New Delhi's Central Fingerprint Bureau(CFPB).

164. Consider the following statements regarding Global Initiative on Digital Health.
1. It is the first such global initiative aimed at data convergence

AMIGOS IAS 95
2. India, in collaboration with the World Health Organization (WHO) will launch this under
G 20 platform
3. The summit is aimed to be an interim medical countermeasure (MCM) before the next
health emergency hits us.
Which of the above given Statements is/are correct ?
A. 2 Only B. 3 Only
C. 1 and 2 Only D. 1,2 and 3
Explanation : D
• India, in collaboration with the World Health Organization (WHO), will launch the Global
Initiative on Digital Health as part of the ongoing G-20 summit in Gandhinagar.
Global Initiative on Digital Health
● It is the first such global initiative aimed at data convergence, the interface of health
platforms and investments in the digital health space around the globe.
● The summit is aimed to be an interim medical countermeasure (MCM) before the next
health emergency hits us.
● The interim medical countermeasure (MCM) is based on a ‘network of networks
approach.
● India, in collaboration with WHO, is leading the advocacy.

165. Consider the following statements regarding Technology Development Fund (TDF) scheme.
1. It was established with the aim of promoting self-reliance in the nation’s defence
technology
2. It is executed by the Defence Research and Development Organisation.
3. It was Launched in the Budget 2023-24
Which of the above given Statements is/are correct ?
A. 1 and 2 Only B. 2 and 3 Only
C. 1 and 3 Only D. 1, 2 and 3
Explanation : A
• The Defence Ministry’s Technology Development Fund (TDF) scheme, has so far supported 41
MSMEs and 20 startups.
Technology Development Fund (TDF)
● Technology Development Fund (TDF) program was initiated by the Ministry of Defence,
India and executed by the Defence Research and Development Organisation.
● Launched in 2016 it was established with the aim of promoting self-reliance in the nation’s
defence technology, it is a part of the popular Make in India.
● The Scheme aims to provide a major fillip to the defence manufacturing sector by
encouraging the industry to innovate on defence technologies.
166. Consider the following with reference to a personality in news recently :

AMIGOS IAS 96
1. He was executed at the age of 24 for his involvement in the assassination of British official
Curzon Wyllie.
2. He became a part of Vinayak Savarkar’s Abhinav Bharat Mandal.
3. A memorial is being inaugurated in Amritsar’s Golbagh area to honor his sacrifice
Select the correct answer from the below given Codes :
A. Udham Singh B. Kartar Singh Sarabha
C. Saifuddin Kitchlew D. Madan Lal Dhingra
Explanation : D
● A memorial is being officially inaugurated to honor Madan Lal Dhingra on the 114th
anniversary of his execution.
● A memorial is being inaugurated in Amritsar’s Golbagh area to honor Dhingra’s sacrifice
and dedication to India’s independence. This memorial is a testament to his legacy.
Madan Lal Dhingra
● Madan Lal Dhingra, an Indian revolutionary, was executed at the age of 24 for his
involvement in the assassination of British official Curzon Wyllie.
● He became a part of Vinayak Savarkar’s Abhinav Bharat Mandal and actively participated
in discussions held at India House, a prominent hub for Indian nationalists residing in
London.
● Being expelled from college due to his participation in protests against the use of British-
imported cloth.
● While in London, Dhingra was associated with various Indian student and political
organizations, which he used as platforms to spread awareness about the oppressive
nature of British rule in India.

167. Consider the following statements regarding the livestock sector in India.
1. The sector has consistently growing at a high Compound Annual Growth Rate (CAGR) of
7.67% from 2014-15 to 2021-22
2. The livestock sector contributed approximately 30.19% to the total Gross Value Added
(GVA) of the agriculture and allied sectors in the year 2021-22.
Which of the above given Statements is/are correct ?
A. 1 Only B. 2 Only
C. Both 1 and 2 D. Neither 1 nor 2
Explanation : C
● A Regional Review Meeting chaired by the Union Secretary of the Department of Animal
Husbandry and Dairying focused on the progress and implementation of various programs
and schemes related to the Animal Husbandry and Dairy sector in the Northern States.
● The livestock sector contributed approximately 30.19% to the total Gross Value Added
(GVA) of the agriculture and allied sectors in the year 2021-22 (at constant prices).

AMIGOS IAS 97
● The meeting highlighted the growth of the livestock sector, which has been consistently
growing at a high Compound Annual Growth Rate (CAGR) of 7.67% from 2014-15 to 2021-
22 (at constant prices).
● This growth is attributed to various parameters within the Livestock Sector, including
Dairy, Bovine, Poultry, Goatery, and Piggery.

168. Consider the following statements regarding Bio-trace Minerals Project.


1. The project aims to revolutionize the manufacturing of bio-trace minerals for animal feed.
2. It uses an innovative technology, known as 'Accelerated Natural Bio Transformation'
(ANBioT)
Which of the above given Statements is/are correct ?
A. 1 Only B. 2 Only
C. Both 1 and 2 D. Neither 1 nor 2
Explanation : C
● This project aligns with the goals of the National Livestock Mission and focuses on
enhancing livestock productivity, optimizing feed and fodder resources, and incorporating
technology into livestock management.
● The project aims to revolutionize the manufacturing of bio-trace minerals for animal feed,
with a particular focus on the livestock and poultry/dairy sectors.
● The use of innovative technology, known as 'Accelerated Natural Bio Transformation'
(ANBioT), along with the strategic utilization of pupa proteins, forms the foundation of
this endeavour.

169. Recently, in order to boost the local economy and contribute significantly to this Union Territory's
growth the auction of Sapphire mines is being considered. They are being auctioned in ?
A. Ladakh B. Jammu and Kashmir
C. Daman and Diu D. Chandigarh
Explanation : B
● The efforts and initiatives of Lieutenant Governor Manoj Sinha in the Union Territory of
Jammu and Kashmir, specifically focusing on the district of Kishtwar.
● Lieutenant Governor Manoj Sinha announced plans to auction Sapphire mines in Jammu
and Kashmir.
● The objective is to boost the local economy and contribute significantly to the Union
Territory's growth.

170. Consider the following statements regarding the Murmansk Port.


1. It is situated about 2,000 km northwest of Moscow
2. It serves as a crucial gateway for trade and transportation in Russia's Arctic region.

AMIGOS IAS 98
3. Petroleum is the primary commodity shipped through Murmansk Port, with a significant
portion destined for India's eastern coast.
Which of the above given Statements is/are correct ?
A. 1 and 2 Only B. 3 Only
C. 2 and 3 Only D. 1 Only
Explanation : C
● India's increasing engagement with Russia's Arctic region, particularly focusing on the
strengthening trade relationship and exploration of the Northern Sea Route (NSR).
● India-bound goods comprise the largest share of cargo handled by Murmansk, Russia's
main northern gateway.
● India and Russia consider a sea-corridor proposal linking Chennai and Vladivostok for
transit container traffic.
Murmansk Port
● Murmansk Port serves as a crucial gateway for trade and transportation in Russia's Arctic
region.
● Murmansk is situated about 2,000 km northwest of Moscow, making it a vital maritime
link between Europe and the Arctic region.
● The port's proximity to the Arctic Ocean grants it access to the NSR, a shorter shipping
route that offers potential advantages over traditional routes like the Suez Canal.
● Coal is a primary commodity shipped through Murmansk Port, with a significant portion
destined for India's eastern coast.

171. Consider the following statements regarding the UDGAM Portal.


1. It has been developed by the State Bank of India.
2. It can be used to search individuals' unclaimed deposits across multiple banks at one
place.
3. As per the RBI, "Unclaimed Deposits" refers to funds held in savings or current accounts
that have remained inactive for duration of 1 year.
How many of the above given Statements is/are correct ?
A. Only one B. Only two
C. All three D. None of them
Explanation : B
• The Reserve Bank of India (RBI) recently launched a Centralised Web Portal, UDGAM
(Unclaimed Deposits – Gateway to Access inforMation), for the public to search for their
unclaimed deposits across multiple banks at one place.

UDGAM Portal:

AMIGOS IAS 99
● It has been developed by RBI for use by members of the public to facilitate and make it
easier for them to search their unclaimed deposits across multiple banks at one place.
● The portal will enable users to either claim the deposit amount or make their deposit
accounts operative at their respective banks.
● According to RBI, "Unclaimed Deposits" refers to funds held in savings or current accounts
that have remained inactive for a duration of 10 years, or in the case of fixed deposits
(FDs), have not been withdrawn within 10 years from the maturity date.

172. Consider the following statements regarding the Agnibaan SubOrbital Technological
Demonstrator (SOrTeD).
1. It is a three-stage launch vehicle powered by ISRO'S patented Agnilet engine.
2. It is an entirely 3D-printed, single-piece, 6 kilonewton (kN) semi-cryogenic engine.
3. Agnibaan SOrTeD will lift off vertically and follow a predetermined trajectory to perform
a precisely orchestrated set of manoeuvres
4. It is capable of carrying payloads up to 5000 kg to an altitude of 7000 km in five different
configurations
How many of the above given Statements is/are correct ?
A. Only one B. Only two
C. Only three D. All four
Explanation : B
• Chennai-based start-up AgniKul Cosmos recently commenced the process of integrating its
cutting-edge Agnibaan SubOrbital Technological Demonstrator (SOrTeD) at its private
launchpad at the Satish Dhawan Space Centre (SDSC) SHAR in Sriharikota.
Agnibaan SubOrbital Technological Demonstrator (SOrTeD):
● Agnibaan SOrTeD is a single-stage launch vehicle powered by AgniKul’s patented Agnilet
engine, which is an entirely 3D-printed, single-piece, 6 kilonewton (kN) semi-cryogenic
engine.
● Unlike traditional sounding rockets that launch from guide rails, Agnibaan SOrTeD will lift
off vertically and follow a predetermined trajectory to perform a precisely orchestrated
set of manoeuvres during flight.
● It is a customisable launch vehicle that could be launched in one or two stages.
● The rocket stands 18 metres tall and has a mass of 14,000 kg.
● It is capable of carrying payloads up to 100 kg to an altitude of 700 km in five different
configurations.

173. Consider the following statements with respect to Matti banana.


1. They are indigenous to Thiruvananthapuram, where it thrives in the unique climate and
soil.
2. The fruit will be without the sweet fragrance and honey-like taste.

AMIGOS IAS 100


3. Its low total soluble solids content (TSSC) recommends it as a baby food.

Which of the above given Statements is/are incorrect ?


A. 3 Only B. 2 and 3 Only
C. 1 and 2 Only D. 1 Only

Explanation : D
● The Matti banana variety, native to Kanniyakumari district was recently granted the
Geographical Indication (GI) tag.

Matti banana:
● There are six known types of the Matti banana and they are indigenous to Kanniyakumari,
where it thrives in the unique climate and soil.
● They are known as ‘Baby Banana’ which flourishes mainly in Kalkulam and Vilavancode
taluks.
● Even if it takes root and yields in other areas, the fruit will be without the sweet fragrance
and honey-like taste unique to the Matti bananas grown in Kanyakumari.
● Unlike typical banana bunches that grow straight, the Matti’s fingers exhibit a distinct
wind-blown appearance.
● Its low total soluble solids content (TSSC) recommends it as a baby food

174. Consider the following statements with reference to Herons.


1. These are any of about 60 species of long-legged wading birds.
2. They are widely distributed across the world equally in tropical and temperate regions.
3. They usually feed while wading quietly in the shallow waters of pools, marshes, and
swamps, catching frogs, fishes, and other aquatic animals.
4. Monitoring their population can indicate the health of the aquatic ecosystem, freshwater
as well as brackish water.
How many of the above given Statements is/are correct ?
A. Only one B. Only two
C. Only three D. All four
Explanation : C
• The annual heronry count jointly organised by WWF-India in association with the Forest
department in Thiruvananthapuram district has recorded more heronries that in the past.
Heron:
● These are any of about 60 species of long-legged wading birds.
● These are classified in the family Ardeidae (order Ciconiiformes) and generally including
several species usually called egrets.

AMIGOS IAS 101


● The Ardeidae also include the bitterns (subfamily Botaurinae).
● These are widely distributed over the world but are most common in the tropics.
● They usually feed while wading quietly in the shallow waters of pools, marshes, and
swamps, catching frogs, fishes, and other aquatic animals.
● They nest in rough platforms of sticks constructed in bushes or trees near water; the nests
usually are grouped in colonies called heronries.
● Being one of the top predators in the aquatic food chain, monitoring their population can
indicate the health of the aquatic ecosystem, freshwater as well as brackish water.

175. Which of the following can be included as World Heritage Railway Lines of India ?
1. Darjeeling Himalayan Railways (DHR)
2. Nilgiri Mountain Railways (NMR)
3. Kalka Shimla Railway (KSR)
4. Chhatrapati Shivaji Maharaj Terminus, Mumbai (CSMT)
Which of the above given Statements is/are correct ?
A. 1,2 and 3 Only B. 2,3 and 4 Only
C. 1,3 and 4 Only D. 1, 2, 3 and 4
Explanation : D
● Heavy rain and floods in Himachal Pradesh have battered the 94-km-long Kalka-Shimla
railway line, a UNESCO World Heritage Site.
● These are various areas or objects inscribed on the United Nations Educational, Scientific
and Cultural Organization (UNESCO) World Heritage List.
● The sites are designated as having “outstanding universal value” under the Convention
Concerning the Protection of the World Cultural and Natural Heritage. This document was
adopted by UNESCO in 1972 and formally took effect in 1975.
World Heritage Railway Lines of India
● Following are the UNESCO World Heritage Sites of Indian Railways
A. Mountain Railways of India:
(i) Darjeeling Himalayan Railways (DHR) - inscribed by UNESCO in 1999
(ii) Nilgiri Mountain Railways (NMR) inscribed in 2005.
(iii) Kalka Shimla Railway (KSR) inscribed in 2008.
B. Chhatrapati Shivaji Maharaj Terminus, Mumbai (CSMT) inscribed in 2004.

176. The concept is a more holistic system than Food Security. It recognizes that control over the food
system needs to remain in the hands of farmers, for whom farming is both a way of life and a
means of producing food. It also recognizes the contribution of indigenous peoples, pastoralists,
forest dwellers, workers and fishers to the food system. It ensures that food is produced in a

AMIGOS IAS 102


culturally acceptable manner and in harmony with the ecosystem in which it is produced. This is
how traditional food production systems have regenerated their soils, water, biodiversity and
climatic conditions, for generations. The concept here is ?
A. Food group B. Farm weight
C. Food pattern equivalent D. Food Sovereignty
Explanation : D
Food sovereignty
● Food sovereignty is the right of peoples to healthy and culturally appropriate food
produced through ecologically sound and sustainable methods, and their right to define
their own food and agriculture systems.
● It puts the aspirations and needs of those who produce, distribute and consume food at
the heart of food systems and policies rather than the demands of markets and
corporations.
● The idea of Food Sovereignty developed as a response to the crises facing the world’s
farmers and food systems.

177. Consider the following statements regarding the Global Centre for Traditional Medicine.
1. It is an initiative of World Health Organization
2. It acts as a hub for fostering collaboration among countries and experts in the field of
traditional medic
3. The Global Centre for Traditional Medicine is headquartered in Jamnagar, Gujarat, India.
Which of the above given Statements is/are correct ?
A. 1 and 2 Only B. 3 Only
C. 2 Only D. 1,2 and 3
Explanation : D
• Recently, World Health Organisation (WHO) Director-General Tedros Adhanom Ghebreyesus
called on nations globally to tap into the potential of traditional medicine.
WHO Global Centre for Traditional Medicine:
● The Global Centre for Traditional Medicine is headquartered in Jamnagar, Gujarat, India.
● The center's primary goal is to advance and promote traditional medicine on a global
scale.
● It acts as a hub for fostering collaboration among countries and experts in the field of
traditional medicine.
● It supports initiatives to integrate traditional medicine into national health systems,
contributing to holistic healthcare approaches.
● Through partnerships and collaborations, the centre works to address health challenges
using a combination of modern and traditional approaches.
● The centre's efforts align with WHO's broader global health strategy, which includes
recognizing the value of traditional medicine in healthcare systems.

AMIGOS IAS 103


● The Global Centre for Traditional Medicine hosted the first global summit on traditional
medicine during the G-20 Health Ministers' meeting.
178. Consider the following statements :
Statement-I :
Recently, Gabon announced a $500 million debt-for-nature swap which is the largest such deal
signed by any country.
Statement-II:
Debt-for-nature swap allows heavily indebted developing countries to seek help from financial
institutions in the developed world with paying off their debt if they agree to spend on
conservation of natural resources.
Which one of the following is correct in respect of the above statements?
A. Both Statement-I and Statement-II are correct and Statement-II is the correct explanation
for Statement-I
B. Both Statement-I and Statement-II are correct and Statement-II is not the correct
explanation for Statement-I
C. Statement-I is correct but Statement-II is incorrect
D. Statement-I is incorrect but Statement-II is correct
Explanation : A
Under the debt-for-nature swap, Gabon has agreed to a deal with the Bank of America, the US
International Development Finance Corporation (USDFC) and The Nature Conservancy (TNC), to
refinance $500 million in national debt toward marine conservation efforts in the country.
Debt-for-nature swap:
● It allows heavily indebted developing countries to seek help from financial institutions in
the developed world with paying off their debt if they agree to spend on conservation of
natural resources.
● The notion of debt-for-nature swaps was first mooted in 1984 by Thomas Lovejoy, the
former vice-president for science at the World Wildlife Fund-US, in response to the Latin
American debt crisis.
● The first debt-for-nature swap was a third-party deal facilitated by Conservation
International. Finalised in 1987, it involved foreign creditors agreeing to forgive USD
650,000 of Bolivia’s debt in exchange for the country setting aside 1.5 million hectares in
the Amazon Basin for conservation efforts.

179. Consider the following statements with reference to a protected area in India :
1. It is a major wildlife sanctuary in Odisha which was established in 1984.
2. It is spread across the Chota Nagpur Plateau region
3. It is connected with Simlipal Reserve via Nato and Sukhupada Hill ranges.
4. It contains mixed deciduous forest.
Select the correct answer from the below given codes :

AMIGOS IAS 104


A. Sunabeda Wildlife Sanctuary
B. Lakhari Valley Wildlife Sanctuary
C. Balukhand - Konark Wildlife Sanctuary
D. Kuldiha Wildlife Sanctuary
Explanation : D
• Recently, the National Green Tribunal (NGT) directed a notice to be issued, related to the
complaint of mining activities taking place in the eco-sensitive zone of Kuldiha Wildlife
Sanctuary, Balasore district, Odisha.

Kuldiha Wildlife Sanctuary:
● It is a major wildlife sanctuary in Odisha which was established in 1984.
● It is spread across the Chota Nagpur Plateau region.
● It is connected with Simlipal Reserve via Nato and Sukhupada Hill ranges.
● It contains mixed deciduous forest.
● It consists of a lot of wild animals like the tigers, elephants, leopards, bison, gaur, giant
squirrels and sambar etc.

180. Consider the following statements with reference to Sanchar Saathi portal.
1. The portal aims to provide various reforms and services related to mobile connections
and telecommunications.
2. It has been developed by C-DoT under the Department of Tele-communications (DoT).
3. The portal allows mobile phone users to Check the connections registered on their names.
Which of the above given Statements is/are correct ?
A. 1 Only B. 3 Only
C. 2 Only D. 1 ,2 and 3
Explanation : D
● In order to reduce cyber frauds performed through fraudulently acquired SIM cards, the
Department of Telecommunications (DoT) will now require police verification for SIM
dealers, the enforcement move follows the launch of the Sanchar Saathi portal.
Sanchar Saathi portal:
● The portal aims to provide various reforms and services related to mobile connections
and telecommunications.
● It has been developed by C-DoT under the Department of Tele-communications (DoT) to
prevent frauds such as identity theft, forged KYC, banking frauds etc.
● The portal allows mobile phone users to:
○ Check the connections registered on their names.
○ Report fraudulent or unrequired connections.

AMIGOS IAS 105


○ Block the mobile phones which are stolen/lost
○ Check IMEI genuineness before buying a mobile phone.
181. Consider the following statements regarding the A-HELP programme.
1. It is part of the Azadi Ka Amrit Mahotsav initiative, Rashtriya Gokul Mission etc for
fostering livestock enhancement.
2. The programme enlists trained women agents for disease control, artificial insemination,
animal tagging, and livestock insurance.
3. It focuses on critical areas of livestock health, disease management, and animal infertility.
Which of the above given Statements is/are correct ?
A. 1 and 2 Only B. 2 and 3 Only
C. 1 and 3 Only D. 1,2 and 3
Explanation : D
A-HELP Programme
● Recently, the Union Minister for Animal Husbandry and Dairying inaugurated the 'A-HELP'
(Accredited Agent for Health and Extension of Livestock Production) programme.
● A-HELP programme is part of the Azadi Ka Amrit Mahotsav initiative, Pashudhan Jagrati
Abhiyaan, and Rashtriya Gokul Mission's goals, fostering livestock enhancement.
● The 'A-HELP' programme enlists trained women agents for disease control, artificial
insemination, animal tagging, and livestock insurance.
● Focuses on critical areas of livestock health, disease management, and animal infertility.
● By empowering women and enhancing livestock, the 'A-HELP' program contributes to the
socio-economic progress of rural communities.

182. Consider the following statements regarding the production of Pulses in India.
1. India is the largest producer, consumer and importer of Pulses in the world.
2. Pulses account for around 12% of the area under foodgrains
3. Kharif pulses contribute more than 60% of the total production.
4. Pulses account for around 7-10% of the total foodgrains production in the country.
How many of the above given Statements is/are correct?
A. Only one B. Only two
C. Only three D. All four
Explanation : B
● Recently, the Union Minister of Agriculture and Farmers Welfare provided valuable
insights in a written reply in Rajya Sabha regarding the comprehensive strategies being
employed to enhance pulses production in India.
India’s Pulses Production:
● India is the largest producer (25% of global production), consumer (27% of world
consumption) and importer (14%) of pulses in the world.

AMIGOS IAS 106


● Pulses account for around 20% of the area under foodgrains and contribute around 7-
10% of the total foodgrains production in the country.
● Though pulses are grown in both Kharif and Rabi seasons, Rabi pulses contribute more
than 60% of the total production.
● Madhya Pradesh, Maharashtra, Rajasthan, Uttar Pradesh and Karnataka are the top five
pulses-producing states.

183. Consider the following statements with reference to Lake Titicaca.


1. It is the largest freshwater lake in Europe
2. It is the highest of the world's largest lakes.
3. Increased evaporation due to rising temperatures have reduced the inflow and volume of
the lake.
Which of the above given Statements is/are correct ?
A. 1 and 2 Only B. 2 and 3 Only
C. 1 and 3 Only D. 1,2 and 3
Explanation : B
● Lake Titicaca is facing a serious threat from climate change and drought. It is the largest
freshwater lake in South America and the highest of the world's largest lakes.
● The lake, which lies on the border between Bolivia and Peru, has seen its water levels
drop to near-record lows.
● The lack of rainfall and the increased evaporation due to rising temperatures have
reduced the inflow and volume of the lake.
● This has resulted in stranded boats, exposed shorelines, and diminished fish populations.
● The lake is also home to more than 500 species of plants and animals, some of which are
endemic and endangered.

184. Which Indian State's 'Koosina Mane' initiative marks a progressive stride towards enhancing
women's labor force participation and addressing gender disparities ?
A. Andhra Pradesh B. Haryana
C. Karnataka D. Madhya Pradesh
Explanation : C
● Karnataka's 'Koosina Mane' initiative, introduced in its 2023-24 Budget, marks a
progressive stride towards enhancing women's labor force participation and addressing
gender disparities.
● This initiative aims to establish childcare centers across 4,000 gram panchayats,
supporting working mothers under the MGNREGA and others nearby.
● It addresses the "triple burden" women face by redistributing childcare responsibilities,
potentially enabling sustained employment and upskilling.

AMIGOS IAS 107


● It will address the issue of the 'motherhood penalty,' which is considered to be one of the
reasons why women drop out of the labor force.
185. Consider the following statements regarding the PM Vishwakarma Yojana.
1. It is a Central Sector Scheme with a financial outlay of Rs.13,000 crore
2. Its aim is to strengthen and nurture the family-based practice of traditional skills by
artisans and craftspeople
3. A credit Support upto 10 lakh rupees with a concessional interest rate of 5% is provided
under the scheme.
Which of the above given Statements is/are correct ?
A. 1 and 2 Only B. 3 Only
C. 1 and 3 Only D. 2 Only
Explanation : A
• Recently, the Cabinet Committee on Economic Affairs chaired by the Prime Minister of India
approved the PM Vishwakarma scheme.
PM Vishwakarma Yojana:
● It is a Central Sector Scheme with a financial outlay of Rs.13,000 crore.
● Time period: Five years (FY 2023-24 to FY 2027-28).
● To strengthen and nurture the Guru-Shishya parampara or family-based practice of
traditional skills by artisans and craftspeople working with their hands and tools.
● The scheme also aims at improving the quality, as well as the reach of products and
services of artisans and craftspeople and to ensure that the Vishwakarmas are integrated
with the domestic and global value chains.
● Under this scheme, the artisans and craftspeople will be provided recognition through
PM Vishwakarma certificate and ID card, Credit Support upto Rs.1 lakh (First Tranche) and
Rs.2 lakh (Second Tranche) with a concessional interest rate of 5%.

186. Consider the following statements regarding Pong Dam.


1. It is an earth-fill embankment dam in the state of Haryana.
2. The purpose of the dam is water storage for irrigation and hydroelectric power
generation.
3. The raised water level of this dam created an artificial lake called the Maharana Pratap
Sagar.
Which of the above given Statements is/are correct ?
A. 1 and 2 Only B. 3 Only
C. 2 and 3 Only D. 1 and 3 Only
Explanation : C
● The Several villages within the Beas River catchment area in Punjab have been submerged
following the release of a substantial amount of water from the Pong Dam.

AMIGOS IAS 108


Pong Dam:
● The Pong Dam, also known as the Beas Dam, is an earth-fill embankment dam on the Beas River
in the state of Himachal Pradesh.
● The purpose of the dam is water storage for irrigation and hydroelectric power generation.
● The construction of the dam began in the year 1961 and was completed in 1974 and at that time
was known as the tallest of its type in the country.
● The raised water level thus invariably created an artificial lake called the Maharana Pratap Sagar,
after the great ruler of Mewar.
● The lake became a renowned bird sanctuary for birds of numerous species including Bar-headed
Goose and the Red-necked Grebe.

187. Recently, which of the following Indian States have become the first Indian state to offer free in
vitro fertilization (IVF) treatment ?
A. Tamilnadu B. Andhra Pradesh
C. Karnataka D. Goa
Explanation : D
● In a first, Goa has become the first Indian state to offer free in vitro fertilization (IVF)
treatment.
In Vitro Fertilization (IVF):
● It is a medical procedure used to assist individuals or couples who are facing fertility
challenges in achieving pregnancy.
● IVF is the most common and effective type of assisted reproductive technology (ART).
● IVF is a complex process that involves retrieving eggs from ovaries and manually
combining them with sperm in a lab for fertilization.
● Several days after fertilization, the fertilized egg (now called an embryo) is placed inside
a uterus.
● Pregnancy occurs when this embryo implants itself into the uterine wall.
● The procedure can be done using a couple's own eggs and sperm. Or IVF may involve eggs,
sperm or embryos from a known or anonymous donor.

188. Which of the following organisations/Institutions/ Ministries/ Departments have recently


launched a Centralised Web Portal UDGAM ?
A. Ministry of MSME
B. Reserve Bank of India
C. Department of Commerce
D. Director General of Foreign Trade
Explanation : B

AMIGOS IAS 109


● Unclaimed Deposits – Gateway to Access inforMation (UDGAM) Portal
○ The Reserve Bank of India (RBI) recently launched a Centralised Web Portal, UDGAM
(Unclaimed Deposits – Gateway to Access inforMation), for the public to search for their
unclaimed deposits across multiple banks at one place.
189. “Gandhinagar Declaration” the WHO South-East Asia Region recently committed to further
accelerate efforts to end ?
A. Cancer B. Diabetes
C. TB D. Cardiovascular diseases
Explanation : C
Gandhinagar Declaration:
● It was adopted at the end of the two-day meeting held in Gandhinagar, Gujarat, to follow
up on the progress made to end tuberculosis (TB) by the countries of the WHO South-East
Asia Region.
● The Declaration calls for establishing a high-level multisectoral commission reporting to
the highest political level in each country for synergy of efforts among various
stakeholders and to monitor progress towards ending TB and other priority diseases.
● These high-level multisectoral commissions on TB could also help build responsive health
systems and advance universal health coverage and health security.
● The declaration calls for ensuring appropriate adoption and use of science and technology
for equitable and human rights-based TB services that are accessible to all, irrespective
of any social, cultural, or demographic divide, through an integrated, primary health care
approach.

190. Consider the following statements regarding the Amazon Cooperation Treaty Organisation
(ACTO).
1. Amazon Cooperation Treaty was signed in 1978 to promote the harmonious development
of Amazonian territories
2. The Permanent Secretariat of ACTO was established in Brasilia in 2002 and permanently
installed in 2003.
3. Argentina, Chile and Peru are some of its members.
Which of the above given Statements is/are correct ?
A. 2 Only B. 3 Only
C. 1 and 2 Only D. 1 Only
Explanation : C
• Amazon Summit under the aegis of the Amazon Cooperation Treaty Organisation (ACTO)
came to the Belem City of Brazil. The Summit produced an outcome document known as
the Belem Declaration. This is only the fourth summit-level meeting under ACTO.
Amazon Cooperation Treaty Organisation (ACTO)

AMIGOS IAS 110


● Amazon Cooperation Treaty Organisation (ACTO) is an intergovernmental organisation
formed by 8 Amazonian countries: Bolivia, Brazil, Colombia, Ecuador, Guyana, Peru,
Suriname and Venezuela (8 countries).
● The above Eight nations decided to create ACTO to strengthen and implement under the
Amazon Cooperation Treaty in 1995 to strengthen and implement the Amazon
Cooperation Treaty objectives.
● The Permanent Secretariat of ACTO was established in Brasilia in 2002 and permanently
installed in 2003.
● Permanent Secretariat of ACTO is to facilitate the exchange, knowledge, cooperation and
joint projection among ACTO Member Countries to fulfil the mandate of Amazon
Cooperation Treaty.

191. Consider the following statements regarding Pibot that has been in the news recently.
1. It is the world’s first humanoid pilot.
2. It can fly an airplane just like a human pilot by manipulating all the single controls in the
cockpit.
3. It is capable of memorizing global flight charts to conduct mistake-free flight missions on
any air route.
Which of the above given Statements is/are correct ?
A. 1 and 2 Only B. 2 and 3 Only
C. 1 and 3 Only D. 1,2 and 3

Explanation : D
● A team of engineers and researchers from the Korea Advanced Institute of Science &
Technology (KAIST) is currently developing a humanoid robot named ‘Pibot’ that can fly
aircraft without needing to modify the cockpit.
Pibot:
● It is the world’s first humanoid pilot.
● It can fly an airplane just like a human pilot by manipulating all the single controls in the
cockpit, which is designed for humans.
● It combines artificial intelligence and robotics technologies, can process the natural
language of the flight manual and control the flight’s operation in real-time.
● It is capable of memorizing global flight charts to conduct mistake-free flight missions on
any air route.
● It can memorise aircraft operation and emergency manuals (QRH, an in-cockpit manual
for the flight crew to refer to in case of in-flight problems) and respond immediately.

192. Recently, Tylototriton zaimeng, a species of salamander, has been in the news. It is found from?
A. Meghalaya B. Manipur

AMIGOS IAS 111


C. Mizoram D. Nagaland

Explanation : B
• Recently, a team of scientists recorded a new species of salamander — the Tylototriton
zaimeng in the Zaimeng Lake of Manipur.
Tylototriton zaimeng:
● This had earlier been mistaken for its close relatives — the T. himalayanus and the T.
verrucosus.
● Phylogenetic analysis revealed that the new salamander is a sister species of the T.
panwaensis and T. houi found in northern Myanmar and southern China.

193. Consider the following statements with reference to Tampara Lake.


1. It is one of the largest freshwater lakes in the State of Assam.
2. It is not yet included in the Wetland Atlas of the Ministry of Environment, Forest and
Climate Change.
3. It is an important habitat for vulnerable species such as Cyprinus carpio, common pochard
and river tern.
How many of the above given Statements is/are correct ?
A. Only one B. Only two
C. All three D. None of them
Explanation : A
• Recently, the National Green Tribunal, Eastern Zone, has directed the Odisha government
not to go ahead with ‘illegal’ construction in and around Tampara Lake.
Tampara Lake:
● It is one of the largest freshwater lakes in the State of Odisha.
● The beautiful lake & the nearby Chilika Lagoon highlight the ecological diversity Odisha is
blessed with.
● It supports at least 60 species of birds, 46 species of fishes, at least 48 species of
phytoplanktons, and more than seven species of terrestrial plants and macrophytes.
● It is an important habitat for vulnerable species such as Cyprinus carpio, common pochard
(Aythya ferina), and river tern (Sterna aurantia).
● It is already placed in the Wetland Atlas prepared by the Ministry of Environment, Forest
and Climate Change in 2010.

194. Consider the following statements with reference to Vindhyagiri frigate.


1. It is named after a mountain range in Gujarat.
2. It is the sixth ship of the Project 17A frigates.

AMIGOS IAS 112


3. 100% of the orders for equipment and systems of Project 17A ships are from indigenous
firms.
How many of the above given Statements is/are correct ?
A. Only one B. Only two
C. All three D. None of them
Explanation : A
● The President of India will launch the Vindhyagiri frigate at Garden Reach Shipbuilders
and Engineers Limited, Kolkata, on August 17.
Vindhyagiri frigate:
● It is named after a mountain range in Karnataka.
● It is the sixth ship of the Project 17A frigates.
● These warships are follow-ons of the Project 17 Class Frigates (Shivalik Class), with
improved stealth features, advanced weapons and sensors and platform management
systems.
● Under the Project 17A programme, four ships by Mumbai-based Mazagaon Dock
Shipbuilders Limited (MDL) and three by Garden Reach Shipbuilders and Engineers
Limited (GRSE) are under construction.
● The project’s first five ships were launched by MDL and GRSE, between 2019-2022.
● These ships have been designed in-house by the Indian Navy’s Warship Design Bureau.
● As much as 75% of the orders for equipment and systems of Project 17A ships are from
indigenous firms, including MSMEs.
195. Consider the following statements regarding the Pradhan Mantri Uchchatar Shiksha Abhiyan
(PM-USHA).
1. It is a Centrally Sponsored Scheme (CSS)
2. It covers Only government institutions of the States and UTs.
3. The scheme also focuses on Accreditation of Non-Accredited Institutions & Improving
Accreditation.
Which of the above given Statements is/are correct ?
A. 1 and 2 Only B. 2 and 3 Only
C. 1 and 3 Only D. 1,2 and 3
Explanation : C
• 14 States and Union Territories which are yet to sign a crucial Memorandum of
Understanding (MoU) with the Union Education Ministry, which mandates the
implementation of the Pradhan Mantri Uchchatar Shiksha Abhiyan (PM-USHA) .
Pradhan Mantri Uchchatar Shiksha Abhiyan:
● In the light of the National Education Policy, Rashtriya Uchchatar Shiksha Abhiyan (RUSA)
scheme has been launched as Pradhan Mantri Uchchatar Shiksha Abhiyan (PM-USHA).
● It covers government and government-aided institutions of the States and UTs.

AMIGOS IAS 113


● It is a Centrally Sponsored Scheme (CSS).
● Accreditation of Non-Accredited Institutions & Improving Accreditation: Accreditation
pushes institutions to meet and maintain higher standards in education, in turn, increases
trust and confidence in them among the public and boosts accountability.
196. Consider the following statements with reference to CarbonLite Metro Travel.
1. It aims to educate passengers about their significant contribution towards reducing
carbon emissions by choosing metro trains.
2. With this initiative, daily commuters will now be able to learn about the average amount
of Carbon Dioxide (CO2) emissions they are decreasing
3. The amount of CO2 will be calculated based on a comparison to road-based motor
vehicles.
Which of the above given Statements is/are correct ?
A. 1 only B. 2 and 3 Only
C. 2 Only D. 1,2 and 3
Explanation : D
• The Delhi Metro Rail Corporation (DMRC) recently unveiled a pioneering initiative called
CarbonLite Metro Travel to educate passengers about their significant contribution
towards reducing carbon emissions by choosing metro trains.
CarbonLite Metro Travel:
● It is a new initiative of the DMRC to help people understand their contribution to reducing
CO2 emissions by opting for metro rail services.
● With this initiative, daily commuters will now be able to understand and learn about the
average amount of Carbon Dioxide (CO2) emissions they are decreasing with their simple
step of selecting the metro as their means of transportation.
● The amount of CO2 will be calculated based on a comparison to road-based motor
vehicles.
● It also aims to persuade commuters to choose an environmentally friendly method of
transportation over motorised alternatives, thereby supporting a cleaner and more
ecologically balanced environment.
● The initiative is in line with the Government of India’s Mission Lifestyle for Environment
(LiFE).

197. Consider the following statements regarding the Nandakini River.


1. Nandakini is one of the five main tributaries of the Ganges River.
2. It is believed to be the abode of Lord Rama.
3. One of the notable tributaries of the river is the Pindar River.
4. The most famous temple along the Nandakini River is the Nandprayag Temple
How many of the above given Statements is/are correct ?
A. Only one B. Only two

AMIGOS IAS 114


C. Only three D. All four
Explanation : C
• The water level of the Nandakini River recently crossed the danger mark in the
Nandanagar area in Uttarakhand’s Chamoli due to incessant rainfall.
Nandakini River:
● Nandakini is one of the five main tributaries of the Ganges River.
● It flows through theChamoli district of Uttarakhand primarily in the Garhwal region,
covering a distance of approximately 105 kilometres before merging with the Alaknanda
River at Nandprayag.
● The river holds great significance in Hindu mythology and is considered sacred. It is
believed to be the abode of Lord Vishnu.
● The region surrounding the river is dotted with ancient temples and shrines.
● The most famous temple along the Nandakini River is the Nandprayag Temple, dedicated
to Lord Vishnu.
● The temple is believed to have been built by Adi Shankaracharya, a renowned philosopher
and saint.
● Several smaller streams and rivers join the Nandakini as it makes its way through the
mountainous terrain. One of the notable tributaries is the Pindar River.

198. Consider the following statements with reference to Floating Rate Loans.
1. They are loans that have an interest rate that changes periodically, depending on a
benchmark rate.
2. Floating-rate loans are also known as variable or adjustable-rate loans
3. Floating rate loans are beneficial to lenders when interest rates are expected to drop in
the future
Which of the above given Statements is/are correct ?
A. 1 Only B. 3 Only
C. 2 and 3 Only D. 1 and 2 Only
Explanation : D
● Recently, the Reserve Bank of India (RBI) will introduce a comprehensive framework to
enhance transparency and establish proper rules for resetting Equated Monthly
Installments (EMIs) for floating rate loans.
● This move aims to address borrower concerns and ensure fair practices by financial
institutions.
● Floating rate loans are loans that have an interest rate that changes periodically,
depending on a benchmark rate or the base rate.

AMIGOS IAS 115


● This base rate, such as the repo rate - rate at which RBI lends money to financial
institutions - is influenced by market forces.
● Floating-rate loans are also known as variable or adjustable-rate loans, as they can vary
over the term of the loan.
● Floating rate loans are common for credit cards, mortgages, and other consumer loans.
● Floating rate loans are beneficial to borrowers when interest rates are expected to drop
in the future.
199. Consider the following statements :
Statement-I :
• Recently, scientists from the Nigerian Centre for Disease Control conducted a study
utilizing metagenomic sequencing for pathogen surveillance.
Statement-II
• Metagenomics is the study of microbes by keeping them in an artificial environment,
which involves the complex chemical processes in which they usually do not exist.
Which one of the following is correct in respect of the above statements?
A. Both Statement-I and Statement-II are correct and Statement-II is the correct explanation
for Statement-I
B. Both Statement-I and Statement-II are correct and Statement-II is not the correct
explanation for Statement-I
C. Statement-I is correct but Statement-II is incorrect
D. Statement-I is incorrect but Statement-II is correct
Explanation : C
● Recently, scientists from the Nigerian Centre for Disease Control conducted a study
utilizing metagenomic sequencing for pathogen surveillance.
● The devastation caused by Covid-19 pandemic led to the rapid development of new
techniques, like metagenomics, and ignited a paradigm shift in how we identify, monitor,
and respond to emerging pathogens.
Metagenomics
● Metagenomics is the study of microbes in their natural living environment, which involves
the complex microbial communities in which they usually exist.
● The study examines the genomic composition of an entire organism, including each of the
microbes that exist within it. It facilitates direct sequencing of patient samples, removing
the need for prior knowledge of the infectious agent.

200. Consider the following statements regarding Building and Other Construction Workers
(Regulation of Employment and Conditions of Service) Act, 1996.
1. It plays a significant role in safeguarding the rights and interests of workers in the
construction sector.

AMIGOS IAS 116


2. It provides for the levy and collection of cess at 1-2% of the cost of construction, as the
Central government may notify.
3. Any worker aged between eighteen and forty five years, engaged in building or
construction activities for a minimum of one year is eligible.
Which of the above given Statements is/are correct ?
A. 1 Only B. 2 Only
C. 3 Only D. 1 and 2 Only
Explanation : D
• Recently, the Minister of Labour and Employment provided valuable insights into the
Building and Other Construction Workers (Regulation of Employment and Conditions of
Service) Act, 1996 during a written reply in the Rajya Sabha.
● The Building and Other Construction Workers (Regulation of Employment and Conditions
of Service) Act, 1996, commonly referred to as the BOCW Act, plays a significant role in
safeguarding the rights and interests of workers in the construction sector.
● It provides for the levy and collection of cess at 1-2% of the cost of construction, as the
Central government may notify.
● As per the provisions, any worker aged between eighteen and sixty years, engaged in
building or construction activities for a minimum of ninety days in the preceding twelve
months, is eligible for registration as a beneficiary with the State Welfare Boards.
Major Provisions:
● The constitution of State Welfare Boards to administer the welfare fund and to provide
various benefits to the registered workers such as accident insurance, medical assistance,
education, housing, pension, etc.
● The fixing of hours of work, wages for overtime work, prohibition of employment of
certain persons in certain building or other construction work, and the provision of
drinking water, latrines, urinals, accommodation, creches, first-aid, canteens, etc.
● The establishment of safety committees and safety officers at every establishment and
the framing of rules for the safety and health of building workers.
● It provides for the levy and collection of cess at 1-2% of the cost of construction, as the
Central government may notify.
201. Consider the following statements regarding the UDGAM Portal.
1. It has been developed by the State Bank of India.
2. It can be used to search individuals' unclaimed deposits across multiple banks at one
place.
3. As per the RBI, "Unclaimed Deposits" refers to funds held in savings or current accounts
that have remained inactive for duration of 1 year.
How many of the above given Statements is/are correct ?
A. Only one B. Only two
C. All three D. None of them
Explanation : B

AMIGOS IAS 117


• The Reserve Bank of India (RBI) recently launched a Centralised Web Portal, UDGAM
(Unclaimed Deposits – Gateway to Access inforMation), for the public to search for their
unclaimed deposits across multiple banks at one place.
UDGAM Portal:
● It has been developed by RBI for use by members of the public to facilitate and make it
easier for them to search their unclaimed deposits across multiple banks at one place.
● The portal will enable users to either claim the deposit amount or make their deposit
accounts operative at their respective banks.
● According to RBI, "Unclaimed Deposits" refers to funds held in savings or current accounts
that have remained inactive for a duration of 10 years, or in the case of fixed deposits
(FDs), have not been withdrawn within 10 years from the maturity date.

202. Consider the following statements regarding the Agnibaan SubOrbital Technological
Demonstrator (SOrTeD).
1. It is a three-stage launch vehicle powered by ISRO'S patented Agnilet engine.
2. It is an entirely 3D-printed, single-piece, 6 kilonewton (kN) semi-cryogenic engine.
3. Agnibaan SOrTeD will lift off vertically and follow a predetermined trajectory to perform
a precisely orchestrated set of manoeuvres
4. It is capable of carrying payloads up to 5000 kg to an altitude of 7000 km in five different
configurations
How many of the above given Statements is/are correct ?
A. Only one B. Only two
C. Only three D. All four
Explanation : B
• Chennai-based start-up AgniKul Cosmos recently commenced the process of integrating
its cutting-edge Agnibaan SubOrbital Technological Demonstrator (SOrTeD) at its private
launchpad at the Satish Dhawan Space Centre (SDSC) SHAR in Sriharikota.
Agnibaan SubOrbital Technological Demonstrator (SOrTeD):

● Agnibaan SOrTeD is a single-stage launch vehicle powered by AgniKul’s patented Agnilet


engine, which is an entirely 3D-printed, single-piece, 6 kilonewton (kN) semi-cryogenic
engine.
● Unlike traditional sounding rockets that launch from guide rails, Agnibaan SOrTeD will lift
off vertically and follow a predetermined trajectory to perform a precisely orchestrated
set of manoeuvres during flight.
● It is a customisable launch vehicle that could be launched in one or two stages.
● The rocket stands 18 metres tall and has a mass of 14,000 kg.
● It is capable of carrying payloads up to 100 kg to an altitude of 700 km in five different
configurations.

AMIGOS IAS 118


203. Consider the following statements with respect to Matti banana.
1. They are indigenous to Thiruvananthapuram, where it thrives in the unique climate and
soil.
2. The fruit will be without the sweet fragrance and honey-like taste.
3. Its low total soluble solids content (TSSC) recommends it as a baby food.
Which of the above given Statements is/are incorrect ?
A. 3 Only B. 2 and 3 Only
C. 1 and 2 Only D. 1 Only
Explanation : D
● The Matti banana variety, native to Kanniyakumari district was recently granted the
Geographical Indication (GI) tag.
Matti banana:
● There are six known types of the Matti banana and they are indigenous to Kanniyakumari,
where it thrives in the unique climate and soil.
● They are known as ‘Baby Banana’ which flourishes mainly in Kalkulam and Vilavancode
taluks.
● Even if it takes root and yields in other areas, the fruit will be without the sweet fragrance
and honey-like taste unique to the Matti bananas grown in Kanyakumari.
● Unlike typical banana bunches that grow straight, the Matti’s fingers exhibit a distinct
wind-blown appearance.
● Its low total soluble solids content (TSSC) recommends it as a baby food

204. Consider the following statements with reference to Herons.


1. These are any of about 60 species of long-legged wading birds.
2. They are widely distributed across the world equally in tropical and temperate regions.
3. They usually feed while wading quietly in the shallow waters of pools, marshes, and
swamps, catching frogs, fishes, and other aquatic animals.
4. Monitoring their population can indicate the health of the aquatic ecosystem, freshwater
as well as brackish water.
How many of the above given Statements is/are correct ?
A. Only one B. Only two
C. Only three D. All four
Explanation : C
• The annual heronry count jointly organised by WWF-India in association with the Forest
department in Thiruvananthapuram district has recorded more heronries that in the past.
Heron:
● These are any of about 60 species of long-legged wading birds.

AMIGOS IAS 119


● These are classified in the family Ardeidae (order Ciconiiformes) and generally including
several species usually called egrets.
● The Ardeidae also include the bitterns (subfamily Botaurinae).
● These are widely distributed over the world but are most common in the tropics.
● They usually feed while wading quietly in the shallow waters of pools, marshes, and
swamps, catching frogs, fishes, and other aquatic animals.
● They nest in rough platforms of sticks constructed in bushes or trees near water; the nests
usually are grouped in colonies called heronries.
● Being one of the top predators in the aquatic food chain, monitoring their population can
indicate the health of the aquatic ecosystem, freshwater as well as brackish water.

205. Which of the following can be included as World Heritage Railway Lines of India ?
1. Darjeeling Himalayan Railways (DHR)
2. Nilgiri Mountain Railways (NMR)
3. Kalka Shimla Railway (KSR)
4. Chhatrapati Shivaji Maharaj Terminus, Mumbai (CSMT)
Which of the above given Statements is/are correct ?
A. 1,2 and 3 Only B. 2,3 and 4 Only
C. 1,3 and 4 Only D. 1, 2, 3 and 4
Explanation : D
● Heavy rain and floods in Himachal Pradesh have battered the 94-km-long Kalka-Shimla
railway line, a UNESCO World Heritage Site.
● These are various areas or objects inscribed on the United Nations Educational, Scientific
and Cultural Organization (UNESCO) World Heritage List.
● The sites are designated as having “outstanding universal value” under the Convention
Concerning the Protection of the World Cultural and Natural Heritage. This document was
adopted by UNESCO in 1972 and formally took effect in 1975.
World Heritage Railway Lines of India
Following are the UNESCO World Heritage Sites of Indian Railways
A. Mountain Railways of India:
(i) Darjeeling Himalayan Railways (DHR) - inscribed by UNESCO in 1999
(ii) Nilgiri Mountain Railways (NMR) inscribed in 2005.
(iii) Kalka Shimla Railway (KSR) inscribed in 2008.
B. Chhatrapati Shivaji Maharaj Terminus, Mumbai (CSMT) inscribed in 2004.

206. The concept is a more holistic system than Food Security. It recognizes that control over the food
system needs to remain in the hands of farmers, for whom farming is both a way of life and a
means of producing food. It also recognizes the contribution of indigenous peoples, pastoralists,

AMIGOS IAS 120


forest dwellers, workers and fishers to the food system. It ensures that food is produced in a
culturally acceptable manner and in harmony with the ecosystem in which it is produced. This is
how traditional food production systems have regenerated their soils, water, biodiversity and
climatic conditions, for generations. The concept here is ?
A. Food group B. Farm weight
C. Food pattern equivalent D. Food Sovereignty
Explanation : D
Food sovereignty
● Food sovereignty is the right of peoples to healthy and culturally appropriate food
produced through ecologically sound and sustainable methods, and their right to define
their own food and agriculture systems.
● It puts the aspirations and needs of those who produce, distribute and consume food at
the heart of food systems and policies rather than the demands of markets and
corporations.
● The idea of Food Sovereignty developed as a response to the crises facing the world’s
farmers and food systems.

207. Consider the following statements regarding the Global Centre for Traditional Medicine.
1. It is an initiative of World Health Organization
2. It acts as a hub for fostering collaboration among countries and experts in the field of
traditional medic
3. The Global Centre for Traditional Medicine is headquartered in Jamnagar, Gujarat, India.
Which of the above given Statements is/are correct ?
A. 1 and 2 Only B. 3 Only
C. 2 Only D. 1,2 and 3
Explanation : D
• Recently, World Health Organisation (WHO) Director-General Tedros Adhanom
Ghebreyesus called on nations globally to tap into the potential of traditional medicine.

WHO Global Centre for Traditional Medicine:


● The Global Centre for Traditional Medicine is headquartered in Jamnagar, Gujarat, India.
● The center's primary goal is to advance and promote traditional medicine on a global
scale.
● It acts as a hub for fostering collaboration among countries and experts in the field of
traditional medicine.
● It supports initiatives to integrate traditional medicine into national health systems,
contributing to holistic healthcare approaches.
● Through partnerships and collaborations, the centre works to address health challenges
using a combination of modern and traditional approaches.

AMIGOS IAS 121


● The centre's efforts align with WHO's broader global health strategy, which includes
recognizing the value of traditional medicine in healthcare systems.
● The Global Centre for Traditional Medicine hosted the first global summit on traditional
medicine during the G-20 Health Ministers' meeting.

208. Consider the following statements :


Statement-I :
Recently, Gabon announced a $500 million debt-for-nature swap which is the largest such deal
signed by any country.
Statement-II:
Debt-for-nature swap allows heavily indebted developing countries to seek help from financial
institutions in the developed world with paying off their debt if they agree to spend on
conservation of natural resources.
Which one of the following is correct in respect of the above statements?
A. Both Statement-I and Statement-II are correct and Statement-II is the correct explanation
for Statement-I
B. Both Statement-I and Statement-II are correct and Statement-II is not the correct
explanation for Statement-I
C. Statement-I is correct but Statement-II is incorrect
D. Statement-I is incorrect but Statement-II is correct
Explanation : A
• Under the debt-for-nature swap, Gabon has agreed to a deal with the Bank of America,
the US International Development Finance Corporation (USDFC) and The Nature
Conservancy (TNC), to refinance $500 million in national debt toward marine conservation
efforts in the country.
Debt-for-nature swap:
● It allows heavily indebted developing countries to seek help from financial institutions in
the developed world with paying off their debt if they agree to spend on conservation of
natural resources.
● The notion of debt-for-nature swaps was first mooted in 1984 by Thomas Lovejoy, the
former vice-president for science at the World Wildlife Fund-US, in response to the Latin
American debt crisis.
● The first debt-for-nature swap was a third-party deal facilitated by Conservation
International. Finalised in 1987, it involved foreign creditors agreeing to forgive USD
650,000 of Bolivia’s debt in exchange for the country setting aside 1.5 million hectares in
the Amazon Basin for conservation efforts.

209. Consider the following statements with reference to a protected area in India :
1. It is a major wildlife sanctuary in Odisha which was established in 1984.
2. It is spread across the Chota Nagpur Plateau region

AMIGOS IAS 122


3. It is connected with Simlipal Reserve via Nato and Sukhupada Hill ranges.
4. It contains mixed deciduous forest.
Select the correct answer from the below given codes :
A. Sunabeda Wildlife Sanctuary
B. Lakhari Valley Wildlife Sanctuary
C. Balukhand - Konark Wildlife Sanctuary
D. Kuldiha Wildlife Sanctuary
Explanation : D
• Recently, the National Green Tribunal (NGT) directed a notice to be issued, related to the
complaint of mining activities taking place in the eco-sensitive zone of Kuldiha Wildlife
Sanctuary, Balasore district, Odisha.
Kuldiha Wildlife Sanctuary:
● It is a major wildlife sanctuary in Odisha which was established in 1984.
● It is spread across the Chota Nagpur Plateau region.
● It is connected with Simlipal Reserve via Nato and Sukhupada Hill ranges.
● It contains mixed deciduous forest.
● It consists of a lot of wild animals like the tigers, elephants, leopards, bison, gaur, giant
squirrels and sambar etc.

210. Consider the following statements with reference to Sanchar Saathi portal.
1. The portal aims to provide various reforms and services related to mobile connections
and telecommunications.
2. It has been developed by C-DoT under the Department of Tele-communications (DoT).
3. The portal allows mobile phone users to Check the connections registered on their names.

Which of the above given Statements is/are correct ?


A. 1 Only B. 3 Only
C. 2 Only D. 1 ,2 and 3
Explanation : D
• In order to reduce cyber frauds performed through fraudulently acquired SIM cards, the
Department of Telecommunications (DoT) will now require police verification for SIM
dealers, the enforcement move follows the launch of the Sanchar Saathi portal.
Sanchar Saathi portal:
● The portal aims to provide various reforms and services related to mobile connections
and telecommunications.
● It has been developed by C-DoT under the Department of Tele-communications (DoT) to
prevent frauds such as identity theft, forged KYC, banking frauds etc.

AMIGOS IAS 123


● The portal allows mobile phone users to:
○ Check the connections registered on their names.
○ Report fraudulent or unrequired connections.
○ Block the mobile phones which are stolen/lost
○ Check IMEI genuineness before buying a mobile phone.

211. Consider the following statements with reference to Infrastructure Debt Fund - Non-Banking
Financial Company (IDF - NBFC).
1. It is a deposit taking NBFC permitted to Refinance infrastructure projects that have
completed at least 10 years of commercial operations
2. It is not allowed to Finance Toll-Operate-Transfer (TOT) projects as a direct lender.
Which of the above given Statements is/are incorrect?
A. 1 Only B. 2 Only
C. Both 1 and 2 D. Neither 1 nor 2
Explanation : C
Infrastructure Debt Fund - Non-Banking Financial Company (IDF - NBFC)
● RBI revised the regulatory framework for IDF-NBFCs.
● IDF-NBFCs will not need sponsors but will be required to have net-owned funds of at least
₹300 crore; capital-to-risk weighted assets ratio (CRAR) of 15% .
● They will be allowed to raise funds through rupee or dollar-denominated bonds with at
least a five-year maturityIDF-NBFC is a non-deposit taking NBFC permitted to:
○ Refinance infrastructure projects that have completed at least one year of commercial
operations
○ Finance Toll-Operate-Transfer (TOT) projects as direct lender.

212. Consider the following statements with reference to Magnetar.


1. A magnetar is an exotic type of neutron star its defining feature that it has an ultra-
powerful magnetic field.
2. Apart from ultra-powerful magnetic fields, magnetars also release vast amounts of energy
in the form of flares, X-rays, and gamma-ray bursts.

Which of the above given Statements is/are correct ?


A. 1 Only B. 2 Only
C. Both 1 and 2 D. Neither 1 nor 2

Explanation : C
Magnetar:

AMIGOS IAS 124


● A magnetar is an exotic type of neutron star, its defining feature that it has an ultra-
powerful magnetic field.
● The field is about 1,000 times stronger than a normal neutron star and about a trillion
times stronger than the Earth’s.
● Apart from ultra-powerful magnetic fields, magnetars also release vast amounts of energy
in the form of flares, X-rays, and gamma-ray bursts.
● They are therefore associated with extreme events in the universe, making them perhaps
the most bizarre objects in the cosmos next to black holes.
● The magnetic field of a magnetar may be caused by a neutron star’s interior – thought to
be made up of neutrons, quarks and exotic states of matter such as Bose-Einstein
Condensates – becoming a superconducting fluid.
● Thus, when the star rotates, it would behave like a huge dynamo, generating an immense
magnetic field.

213. Consider the following statements regarding National Automated Fingerprint Identification
System (NAFIS).
1. It is a nationwide database managed by NCRB
2. It swiftly stores crime-related fingerprints and assigns a unique 10-digit National
Fingerprint Number (NFN)
3. NFN remains with the offender for life, connecting various crimes under different FIRs to
the same ID.
Which of the above given statements is/are correct?
A. 1 and 2 Only B. 3 Only
C. 2 and 3 Only D. 1,2 and 3
Explanation : D
• The team of National Automated Fingerprint Identification System (NAFIS) of National
Crime Records Bureau (NCRB) for winning the Gold Award under the Excellence in
Government Process Reengineering for Digital Transformation Category-1 of Department
of Administrative Reforms and Public Grievances (DARPG).
● NAFIS, a nationwide database managed by NCRB, swiftly stores crime-related fingerprints
and assigns a unique 10-digit National Fingerprint Number (NFN) to apprehended
criminals within 24 hours.
● NFN remains with the offender for life, connecting various crimes under different FIRs to
the same ID.
● Operates from New Delhi's Central Fingerprint Bureau(CFPB).
214. Consider the following statements regarding Global Initiative on Digital Health.
1. It is the first such global initiative aimed at data convergence
2. India, in collaboration with the World Health Organization (WHO) will launch this under
G 20 platform

AMIGOS IAS 125


3. The summit is aimed to be an interim medical countermeasure (MCM) before the next
health emergency hits us.
Which of the above given Statements is/are correct ?
A. 2 Only
B. 3 Only
C. 1 and 2 Only
D. 1,2 and 3
Explanation : D
• India, in collaboration with the World Health Organization (WHO), will launch the Global
Initiative on Digital Health as part of the ongoing G-20 summit in Gandhinagar.
Global Initiative on Digital Health
● It is the first such global initiative aimed at data convergence, the interface of health
platforms and investments in the digital health space around the globe.
● The summit is aimed to be an interim medical countermeasure (MCM) before the next
health emergency hits us.
● The interim medical countermeasure (MCM) is based on a ‘network of networks
approach.
● India, in collaboration with WHO, is leading the advocacy.

215. Consider the following statements regarding Technology Development Fund (TDF) scheme.
1. It was established with the aim of promoting self-reliance in the nation’s defence
technology
2. It is executed by the Defence Research and Development Organisation.
3. It was Launched in the Budget 2023-24
Which of the above given Statements is/are correct ?
A. 1 and 2 Only B. 2 and 3 Only
C. 1 and 3 Only D. 1, 2 and 3
Explanation : A
• The Defence Ministry’s Technology Development Fund (TDF) scheme, has so far
supported 41 MSMEs and 20 startups.

Technology Development Fund (TDF)


● Technology Development Fund (TDF) program was initiated by the Ministry of Defence,
India and executed by the Defence Research and Development Organisation.
● Launched in 2016 it was established with the aim of promoting self-reliance in the nation’s
defence technology, it is a part of the popular Make in India.
● The Scheme aims to provide a major fillip to the defence manufacturing sector by
encouraging the industry to innovate on defence technologies.

AMIGOS IAS 126


216. Consider the following with reference to a personality in news recently :
1. He was executed at the age of 24 for his involvement in the assassination of British official
Curzon Wyllie.
2. He became a part of Vinayak Savarkar’s Abhinav Bharat Mandal.
3. A memorial is being inaugurated in Amritsar’s Golbagh area to honor his sacrifice
Select the correct answer from the below given Codes :
A. Udham Singh B. Kartar Singh Sarabha
C. Saifuddin Kitchlew D. Madan Lal Dhingra
Explanation : D
● A memorial is being officially inaugurated to honor Madan Lal Dhingra on the 114th
anniversary of his execution.
● A memorial is being inaugurated in Amritsar’s Golbagh area to honor Dhingra’s sacrifice
and dedication to India’s independence. This memorial is a testament to his legacy.
Madan Lal Dhingra
● Madan Lal Dhingra, an Indian revolutionary, was executed at the age of 24 for his
involvement in the assassination of British official Curzon Wyllie.
● He became a part of Vinayak Savarkar’s Abhinav Bharat Mandal and actively participated
in discussions held at India House, a prominent hub for Indian nationalists residing in
London.
● Being expelled from college due to his participation in protests against the use of British-
imported cloth.
● While in London, Dhingra was associated with various Indian student and political
organizations, which he used as platforms to spread awareness about the oppressive
nature of British rule in India.

217. Consider the following statements regarding the livestock sector in India.
1. The sector has consistently growing at a high Compound Annual Growth Rate (CAGR) of
7.67% from 2014-15 to 2021-22
2. The livestock sector contributed approximately 30.19% to the total Gross Value Added
(GVA) of the agriculture and allied sectors in the year 2021-22.
Which of the above given Statements is/are correct ?
A. 1 Only B. 2 Only
C. Both 1 and 2 D. Neither 1 nor 2
Explanation : C
• A Regional Review Meeting chaired by the Union Secretary of the Department of Animal
Husbandry and Dairying focused on the progress and implementation of various programs
and schemes related to the Animal Husbandry and Dairy sector in the Northern States.

AMIGOS IAS 127


● The livestock sector contributed approximately 30.19% to the total Gross Value Added
(GVA) of the agriculture and allied sectors in the year 2021-22 (at constant prices).
● The meeting highlighted the growth of the livestock sector, which has been consistently
growing at a high Compound Annual Growth Rate (CAGR) of 7.67% from 2014-15 to 2021-
22 (at constant prices).
● This growth is attributed to various parameters within the Livestock Sector, including
Dairy, Bovine, Poultry, Goatery, and Piggery.

218. Consider the following statements regarding Bio-trace Minerals Project.


1. The project aims to revolutionize the manufacturing of bio-trace minerals for animal feed.
2. It uses an innovative technology, known as 'Accelerated Natural Bio Transformation'
(ANBioT)
Which of the above given Statements is/are correct ?
A. 1 Only B. 2 Only
C. Both 1 and 2 D. Neither 1 nor 2
Explanation : C
● This project aligns with the goals of the National Livestock Mission and focuses on
enhancing livestock productivity, optimizing feed and fodder resources, and incorporating
technology into livestock management.
● The project aims to revolutionize the manufacturing of bio-trace minerals for animal feed,
with a particular focus on the livestock and poultry/dairy sectors.
● The use of innovative technology, known as 'Accelerated Natural Bio Transformation'
(ANBioT), along with the strategic utilization of pupa proteins, forms the foundation of
this endeavour.

219. Recently, in order to boost the local economy and contribute significantly to this Union Territory's
growth the auction of Sapphire mines is being considered. They are being auctioned in ?
A. Ladakh B. Jammu and Kashmir
C. Daman and Diu D. Chandigarh
Explanation : B
● The efforts and initiatives of Lieutenant Governor Manoj Sinha in the Union Territory of
Jammu and Kashmir, specifically focusing on the district of Kishtwar.
● Lieutenant Governor Manoj Sinha announced plans to auction Sapphire mines in Jammu
and Kashmir.
● The objective is to boost the local economy and contribute significantly to the Union
Territory's growth.

AMIGOS IAS 128


220. Consider the following statements regarding the Murmansk Port.
1. It is situated about 2,000 km northwest of Moscow
2. It serves as a crucial gateway for trade and transportation in Russia's Arctic region.
3. Petroleum is the primary commodity shipped through Murmansk Port, with a significant
portion destined for India's eastern coast.
Which of the above given Statements is/are correct ?
A. 1 and 2 Only
B. 3 Only
C. 2 and 3 Only
D. 1 Only
Explanation : C
• India's increasing engagement with Russia's Arctic region, particularly focusing on the
strengthening trade relationship and exploration of the Northern Sea Route (NSR).
● India-bound goods comprise the largest share of cargo handled by Murmansk, Russia's
main northern gateway.
● India and Russia consider a sea-corridor proposal linking Chennai and Vladivostok for
transit container traffic.
Murmansk Port
● Murmansk Port serves as a crucial gateway for trade and transportation in Russia's Arctic
region.
● Murmansk is situated about 2,000 km northwest of Moscow, making it a vital maritime
link between Europe and the Arctic region.
● The port's proximity to the Arctic Ocean grants it access to the NSR, a shorter shipping
route that offers potential advantages over traditional routes like the Suez Canal.
● Coal is a primary commodity shipped through Murmansk Port, with a significant portion
destined for India's eastern coast.

221. Consider the following statements regarding Pibot that has been in the news recently.
1. It is the world’s first humanoid pilot.
2. It can fly an airplane just like a human pilot by manipulating all the single controls in the
cockpit.
3. It is capable of memorizing global flight charts to conduct mistake-free flight missions on
any air route.
Which of the above given Statements is/are correct ?
A. 1 and 2 Only B. 2 and 3 Only
C. 1 and 3 Only D. 1,2 and 3
Explanation : D

AMIGOS IAS 129


● A team of engineers and researchers from the Korea Advanced Institute of Science &
Technology (KAIST) is currently developing a humanoid robot named ‘Pibot’ that can fly
aircraft without needing to modify the cockpit.
Pibot:
● It is the world’s first humanoid pilot.
● It can fly an airplane just like a human pilot by manipulating all the single controls in the
cockpit, which is designed for humans.
● It combines artificial intelligence and robotics technologies, can process the natural
language of the flight manual and control the flight’s operation in real-time.
● It is capable of memorizing global flight charts to conduct mistake-free flight missions on
any air route.
● It can memorise aircraft operation and emergency manuals (QRH, an in-cockpit manual
for the flight crew to refer to in case of in-flight problems) and respond immediately.

222. Recently, Tylototriton zaimeng, a species of salamander , has been in the news. It is found from?
A. Meghalaya B. Manipur
C. Mizoram D. Nagaland

Explanation : B
• Recently, a team of scientists recorded a new species of salamander — the Tylototriton
zaimeng in the Zaimeng Lake of Manipur.

Tylototriton zaimeng:
● This had earlier been mistaken for its close relatives — the T. himalayanus and the T.
verrucosus.
● Phylogenetic analysis revealed that the new salamander is a sister species of the T.
panwaensis and T. houi found in northern Myanmar and southern China.
223. Consider the following statements with reference to Tampara Lake.
1. It is one of the largest freshwater lakes in the State of Assam.
2. It is not yet included in the Wetland Atlas of the Ministry of Environment, Forest and
Climate Change.
3. It is an important habitat for vulnerable species such as Cyprinus carpio, common pochard
and river tern.
How many of the above given Statements is/are correct ?
A. Only one B. Only two
C. All three D. None of them
Explanation : A

AMIGOS IAS 130


• Recently, the National Green Tribunal, Eastern Zone, has directed the Odisha government not
to go ahead with ‘illegal’ construction in and around Tampara Lake.
Tampara Lake:
● It is one of the largest freshwater lakes in the State of Odisha.
● The beautiful lake & the nearby Chilika Lagoon highlight the ecological diversity Odisha is
blessed with.
● It supports at least 60 species of birds, 46 species of fishes, at least 48 species of
phytoplanktons, and more than seven species of terrestrial plants and macrophytes.
● It is an important habitat for vulnerable species such as Cyprinus carpio, common pochard
(Aythya ferina), and river tern (Sterna aurantia).
● It is already placed in the Wetland Atlas prepared by the Ministry of Environment, Forest
and Climate Change in 2010.

224. Consider the following statements with reference to Vindhyagiri frigate.


1. It is named after a mountain range in Gujarat.
2. It is the sixth ship of the Project 17A frigates.
3. 100% of the orders for equipment and systems of Project 17A ships are from indigenous
firms.
How many of the above given Statements is/are correct ?
A. Only one B. Only two
C. All three D. None of them
Explanation : A
● The President of India will launch the Vindhyagiri frigate at Garden Reach Shipbuilders
and Engineers Limited, Kolkata, on August 17.
Vindhyagiri frigate:
● It is named after a mountain range in Karnataka.
● It is the sixth ship of the Project 17A frigates.
● These warships are follow-ons of the Project 17 Class Frigates (Shivalik Class), with
improved stealth features, advanced weapons and sensors and platform management
systems.
● Under the Project 17A programme, four ships by Mumbai-based Mazagaon Dock
Shipbuilders Limited (MDL) and three by Garden Reach Shipbuilders and Engineers
Limited (GRSE) are under construction.
● The project’s first five ships were launched by MDL and GRSE, between 2019-2022.
● These ships have been designed in-house by the Indian Navy’s Warship Design Bureau.
● As much as 75% of the orders for equipment and systems of Project 17A ships are from
indigenous firms, including MSMEs.

AMIGOS IAS 131


225. Consider the following statements regarding the Pradhan Mantri Uchchatar Shiksha Abhiyan
(PM-USHA).
1. It is a Centrally Sponsored Scheme (CSS)
2. It covers Only government institutions of the States and UTs.
3. The scheme also focuses on Accreditation of Non-Accredited Institutions & Improving
Accreditation.
Which of the above given Statements is/are correct ?
A. 1 and 2 Only B. 2 and 3 Only
C. 1 and 3 Only D. 1,2 and 3
Explanation : C
• 14 States and Union Territories which are yet to sign a crucial Memorandum of Understanding
(MoU) with the Union Education Ministry, which mandates the implementation of the
Pradhan Mantri Uchchatar Shiksha Abhiyan (PM-USHA).
Pradhan Mantri Uchchatar Shiksha Abhiyan:
● In the light of the National Education Policy, Rashtriya Uchchatar Shiksha Abhiyan (RUSA)
scheme has been launched as Pradhan Mantri Uchchatar Shiksha Abhiyan (PM-USHA).
● It covers government and government-aided institutions of the States and UTs.
● It is a Centrally Sponsored Scheme (CSS).
● Accreditation of Non-Accredited Institutions & Improving Accreditation: Accreditation
pushes institutions to meet and maintain higher standards in education, in turn, increases
trust and confidence in them among the public and boosts accountability.

226. Consider the following statements with reference to CarbonLite Metro Travel.
1. It aims to educate passengers about their significant contribution towards reducing
carbon emissions by choosing metro trains.
2. With this initiative, daily commuters will now be able to learn about the average amount
of Carbon Dioxide (CO2) emissions they are decreasing
3. The amount of CO2 will be calculated based on a comparison to road-based motor
vehicles.
Which of the above given Statements is/are correct ?
A. 1 only B. 2 and 3 Only
C. 2 Only D. 1,2 and 3
Explanation : D
• The Delhi Metro Rail Corporation (DMRC) recently unveiled a pioneering initiative called
CarbonLite Metro Travel to educate passengers about their significant contribution towards
reducing carbon emissions by choosing metro trains.

AMIGOS IAS 132


CarbonLite Metro Travel:
● It is a new initiative of the DMRC to help people understand their contribution to reducing
CO2 emissions by opting for metro rail services.
● With this initiative, daily commuters will now be able to understand and learn about the
average amount of Carbon Dioxide (CO2) emissions they are decreasing with their simple
step of selecting the metro as their means of transportation.
● The amount of CO2 will be calculated based on a comparison to road-based motor
vehicles.
● It also aims to persuade commuters to choose an environmentally friendly method of
transportation over motorised alternatives, thereby supporting a cleaner and more
ecologically balanced environment.
● The initiative is in line with the Government of India’s Mission Lifestyle for Environment
(LiFE).

227. Consider the following statements regarding the Nandakini River.


1. Nandakini is one of the five main tributaries of the Ganges River.
2. It is believed to be the abode of Lord Rama.
3. One of the notable tributaries of the river is the Pindar River.
4. The most famous temple along the Nandakini River is the Nandprayag Temple
How many of the above given Statements is/are correct ?
A. Only one B. Only two
C. Only three D. All four
Explanation : C
• The water level of the Nandakini River recently crossed the danger mark in the Nandanagar
area in Uttarakhand’s Chamoli due to incessant rainfall.

Nandakini River:
● Nandakini is one of the five main tributaries of the Ganges River.
● It flows through theChamoli district of Uttarakhand primarily in the Garhwal region,
covering a distance of approximately 105 kilometres before merging with the Alaknanda
River at Nandprayag.
● The river holds great significance in Hindu mythology and is considered sacred. It is
believed to be the abode of Lord Vishnu.
● The region surrounding the river is dotted with ancient temples and shrines.
● The most famous temple along the Nandakini River is the Nandprayag Temple, dedicated
to Lord Vishnu.
● The temple is believed to have been built by Adi Shankaracharya, a renowned philosopher
and saint.

AMIGOS IAS 133


● Several smaller streams and rivers join the Nandakini as it makes its way through the
mountainous terrain. One of the notable tributaries is the Pindar River.

228. Consider the following statements with reference to Floating Rate Loans.
1. They are loans that have an interest rate that changes periodically, depending on a
benchmark rate.
2. Floating-rate loans are also known as variable or adjustable-rate loans
3. Floating rate loans are beneficial to lenders when interest rates are expected to drop in
the future
Which of the above given Statements is/are correct ?
A. 1 Only B. 3 Only
C. 2 and 3 Only D. 1 and 2 Only
Explanation : D
● Recently, the Reserve Bank of India (RBI) will introduce a comprehensive framework to
enhance transparency and establish proper rules for resetting Equated Monthly
Installments (EMIs) for floating rate loans.
● This move aims to address borrower concerns and ensure fair practices by financial
institutions.
● Floating rate loans are loans that have an interest rate that changes periodically,
depending on a benchmark rate or the base rate.
● This base rate, such as the repo rate - rate at which RBI lends money to financial
institutions - is influenced by market forces.
● Floating-rate loans are also known as variable or adjustable-rate loans, as they can vary
over the term of the loan.
● Floating rate loans are common for credit cards, mortgages, and other consumer loans.
● Floating rate loans are beneficial to borrowers when interest rates are expected to drop
in the future.

229. Consider the following statements :


Statement-I :
Recently, scientists from the Nigerian Centre for Disease Control conducted a study utilizing
metagenomic sequencing for pathogen surveillance.
Statement-II
Metagenomics is the study of microbes by keeping them in an artificial environment, which
involves the complex chemical processes in which they usually do not exist.
Which one of the following is correct in respect of the above statements?
A. Both Statement-I and Statement-II are correct and Statement-II is the correct explanation
for Statement-I

AMIGOS IAS 134


B. Both Statement-I and Statement-II are correct and Statement-II is not the correct
explanation for Statement-I
C. Statement-I is correct but Statement-II is incorrect
D. Statement-I is incorrect but Statement-II is correct
Explanation : C
● Recently, scientists from the Nigerian Centre for Disease Control conducted a study
utilizing metagenomic sequencing for pathogen surveillance.
● The devastation caused by Covid-19 pandemic led to the rapid development of new
techniques, like metagenomics, and ignited a paradigm shift in how we identify, monitor,
and respond to emerging pathogens.
Metagenomics
● Metagenomics is the study of microbes in their natural living environment, which involves
the complex microbial communities in which they usually exist.
● The study examines the genomic composition of an entire organism, including each of the
microbes that exist within it. It facilitates direct sequencing of patient samples, removing
the need for prior knowledge of the infectious agent.

230. Consider the following statements regarding Building and Other Construction Workers
(Regulation of Employment and Conditions of Service) Act, 1996.
1. It plays a significant role in safeguarding the rights and interests of workers in the
construction sector.
2. It provides for the levy and collection of cess at 1-2% of the cost of construction, as the
Central government may notify.
3. Any worker aged between eighteen and forty five years, engaged in building or
construction activities for a minimum of one year is eligible.
Which of the above given Statements is/are correct ?
A. 1 Only B. 2 Only
C. 3 Only D. 1 and 2 Only
Explanation : D
• Recently, the Minister of Labour and Employment provided valuable insights into the
Building and Other Construction Workers (Regulation of Employment and Conditions of
Service) Act, 1996 during a written reply in the Rajya Sabha.
● The Building and Other Construction Workers (Regulation of Employment and Conditions
of Service) Act, 1996, commonly referred to as the BOCW Act, plays a significant role in
safeguarding the rights and interests of workers in the construction sector.
● It provides for the levy and collection of cess at 1-2% of the cost of construction, as the
Central government may notify.
● As per the provisions, any worker aged between eighteen and sixty years, engaged in
building or construction activities for a minimum of ninety days in the preceding twelve
months, is eligible for registration as a beneficiary with the State Welfare Boards.

AMIGOS IAS 135


Major Provisions:
● The constitution of State Welfare Boards to administer the welfare fund and to provide
various benefits to the registered workers such as accident insurance, medical assistance,
education, housing, pension, etc.
● The fixing of hours of work, wages for overtime work, prohibition of employment of
certain persons in certain building or other construction work, and the provision of
drinking water, latrines, urinals, accommodation, creches, first-aid, canteens, etc.
● The establishment of safety committees and safety officers at every establishment and
the framing of rules for the safety and health of building workers.
● It provides for the levy and collection of cess at 1-2% of the cost of construction, as the
Central government may notify.

231. Consider the following statements regarding the A-HELP programme.


1. It is part of the Azadi Ka Amrit Mahotsav initiative, Rashtriya Gokul Mission etc for
fostering livestock enhancement.
2. The programme enlists trained women agents for disease control, artificial insemination,
animal tagging, and livestock insurance.
3. It focuses on critical areas of livestock health, disease management, and animal infertility.
Which of the above given Statements is/are correct ?
A. 1 and 2 Only B. 2 and 3 Only
C. 1 and 3 Only D. 1,2 and 3
Explanation : D
A-HELP Programme
● Recently, the Union Minister for Animal Husbandry and Dairying inaugurated the 'A-HELP'
(Accredited Agent for Health and Extension of Livestock Production) programme.
● A-HELP programme is part of the Azadi Ka Amrit Mahotsav initiative, Pashudhan Jagrati
Abhiyaan, and Rashtriya Gokul Mission's goals, fostering livestock enhancement.
● The 'A-HELP' programme enlists trained women agents for disease control, artificial
insemination, animal tagging, and livestock insurance.
● Focuses on critical areas of livestock health, disease management, and animal infertility.
● By empowering women and enhancing livestock, the 'A-HELP' program contributes to the
socio-economic progress of rural communities.

232. Consider the following statements regarding the production of Pulses in India.
1. India is the largest producer, consumer and importer of Pulses in the world.
2. Pulses account for around 12% of the area under foodgrains
3. Kharif pulses contribute more than 60% of the total production.
4. Pulses account for around 7-10% of the total foodgrains production in the country.

AMIGOS IAS 136


How many of the above given Statements is/are correct?
A. Only one B. Only two
C. Only three D. All four
Explanation : B
• Recently, the Union Minister of Agriculture and Farmers Welfare provided valuable insights
in a written reply in Rajya Sabha regarding the comprehensive strategies being employed to
enhance pulses production in India.
India’s Pulses Production:
● India is the largest producer (25% of global production), consumer (27% of world
consumption) and importer (14%) of pulses in the world.
● Pulses account for around 20% of the area under foodgrains and contribute around 7-
10% of the total foodgrains production in the country.
● Though pulses are grown in both Kharif and Rabi seasons, Rabi pulses contribute more
than 60% of the total production.
● Madhya Pradesh, Maharashtra, Rajasthan, Uttar Pradesh and Karnataka are the top five
pulses-producing states.

233. Consider the following statements with reference to Lake Titicaca.


1. It is the largest freshwater lake in Europe
2. It is the highest of the world's largest lakes.
3. Increased evaporation due to rising temperatures have reduced the inflow and volume of
the lake.
Which of the above given Statements is/are correct ?
A. 1 and 2 Only B. 2 and 3 Only
C. 1 and 3 Only D. 1,2 and 3
Explanation : B
● Lake Titicaca is facing a serious threat from climate change and drought. It is the largest
freshwater lake in South America and the highest of the world's largest lakes.
● The lake, which lies on the border between Bolivia and Peru, has seen its water levels
drop to near-record lows.
● The lack of rainfall and the increased evaporation due to rising temperatures have
reduced the inflow and volume of the lake.
● This has resulted in stranded boats, exposed shorelines, and diminished fish populations.
● The lake is also home to more than 500 species of plants and animals, some of which are
endemic and endangered.

234. Which Indian State's 'Koosina Mane' initiative marks a progressive stride towards enhancing
women's labor force participation and addressing gender disparities ?

AMIGOS IAS 137


A. Andhra Pradesh B. Haryana
C. Karnataka D. Madhya Pradesh
Explanation : C
● Karnataka's 'Koosina Mane' initiative, introduced in its 2023-24 Budget, marks a
progressive stride towards enhancing women's labor force participation and addressing
gender disparities.
● This initiative aims to establish childcare centers across 4,000 gram panchayats,
supporting working mothers under the MGNREGA and others nearby.
● It addresses the "triple burden" women face by redistributing childcare responsibilities,
potentially enabling sustained employment and upskilling.
● It will address the issue of the 'motherhood penalty,' which is considered to be one of the
reasons why women drop out of the labor force.

235. Consider the following statements regarding the PM Vishwakarma Yojana.


1. It is a Central Sector Scheme with a financial outlay of Rs.13,000 crore
2. Its aim is to strengthen and nurture the family-based practice of traditional skills by
artisans and craftspeople
3. A credit Support upto 10 lakh rupees with a concessional interest rate of 5% is provided
under the scheme.
Which of the above given Statements is/are correct ?
A. 1 and 2 Only B. 3 Only
C. 1 and 3 Only D. 2 Only
Explanation : A
• Recently, the Cabinet Committee on Economic Affairs chaired by the Prime Minister of
India approved the PM Vishwakarma scheme.
PM Vishwakarma Yojana:
● It is a Central Sector Scheme with a financial outlay of Rs.13,000 crore.
● Time period: Five years (FY 2023-24 to FY 2027-28).
● To strengthen and nurture the Guru-Shishya parampara or family-based practice of
traditional skills by artisans and craftspeople working with their hands and tools.
● The scheme also aims at improving the quality, as well as the reach of products and
services of artisans and craftspeople and to ensure that the Vishwakarmas are integrated
with the domestic and global value chains.
● Under this scheme, the artisans and craftspeople will be provided recognition through
PM Vishwakarma certificate and ID card, Credit Support upto Rs.1 lakh (First Tranche) and
Rs.2 lakh (Second Tranche) with a concessional interest rate of 5%.

236. Consider the following statements regarding Pong Dam.


1. It is an earth-fill embankment dam in the state of Haryana.

AMIGOS IAS 138


2. The purpose of the dam is water storage for irrigation and hydroelectric power
generation.
3. The raised water level of this dam created an artificial lake called the Maharana Pratap
Sagar.
Which of the above given Statements is/are correct ?
A. 1 and 2 Only B. 3 Only
C. 2 and 3 Only D. 1 and 3 Only
Explanation : C
• Several villages within the Beas River catchment area in Punjab have been submerged
following the release of a substantial amount of water from the Pong Dam.
Pong Dam:
● The Pong Dam, also known as the Beas Dam, is an earth-fill embankment dam on the Beas
River in the state of Himachal Pradesh.
● The purpose of the dam is water storage for irrigation and hydroelectric power
generation.
● The construction of the dam began in the year 1961 and was completed in 1974 and at
that time was known as the tallest of its type in the country.
● The raised water level thus invariably created an artificial lake called the Maharana Pratap
Sagar, after the great ruler of Mewar.
● The lake became a renowned bird sanctuary for birds of numerous species including Bar-
headed Goose and the Red-necked Grebe.
237. Recently, which of the following Indian States have become the first Indian state to offer free in
vitro fertilization (IVF) treatment ?
A. Tamilnadu B. Andhra Pradesh
C. Karnataka D. Goa
Explanation : D
● In a first, Goa has become the first Indian state to offer free in vitro fertilization (IVF)
treatment.
In Vitro Fertilization (IVF):
● It is a medical procedure used to assist individuals or couples who are facing fertility
challenges in achieving pregnancy.
● IVF is the most common and effective type of assisted reproductive technology (ART).
● IVF is a complex process that involves retrieving eggs from ovaries and manually
combining them with sperm in a lab for fertilization.
● Several days after fertilization, the fertilized egg (now called an embryo) is placed inside
a uterus.
● Pregnancy occurs when this embryo implants itself into the uterine wall.
● The procedure can be done using a couple's own eggs and sperm. Or IVF may involve eggs,
sperm or embryos from a known or anonymous donor.

AMIGOS IAS 139


238. Which of the following organisations/Institutions/ Ministries/ Departments have recently
launched a Centralised Web Portal UDGAM ?
A. Ministry of MSME
B. Reserve Bank of India
C. Department of Commerce
D. Director General of Foreign Trade
Explanation : B
● Unclaimed Deposits – Gateway to Access inforMation (UDGAM) Portal
○ The Reserve Bank of India (RBI) recently launched a Centralised Web Portal, UDGAM
(Unclaimed Deposits – Gateway to Access inforMation), for the public to search for their
unclaimed deposits across multiple banks at one place.

239. “Gandhinagar Declaration” the WHO South-East Asia Region recently committed to further
accelerate efforts to end ?
A. Cancer B. Diabetes
C. TB D. Cardiovascular diseases
Explanation : C
Gandhinagar Declaration:
● It was adopted at the end of the two-day meeting held in Gandhinagar, Gujarat, to follow
up on the progress made to end tuberculosis (TB) by the countries of the WHO South-East
Asia Region.
● The Declaration calls for establishing a high-level multisectoral commission reporting to
the highest political level in each country for synergy of efforts among various
stakeholders and to monitor progress towards ending TB and other priority diseases.
● These high-level multisectoral commissions on TB could also help build responsive health
systems and advance universal health coverage and health security.
● The declaration calls for ensuring appropriate adoption and use of science and technology
for equitable and human rights-based TB services that are accessible to all, irrespective
of any social, cultural, or demographic divide, through an integrated, primary health care
approach.

240. Consider the following statements regarding the Amazon Cooperation Treaty Organisation
(ACTO).
1. Amazon Cooperation Treaty was signed in 1978 to promote the harmonious development
of Amazonian territories
2. The Permanent Secretariat of ACTO was established in Brasilia in 2002 and permanently
installed in 2003.
3. Argentina, Chile and Peru are some of its members.

AMIGOS IAS 140


Which of the above given Statements is/are correct ?
A. 2 Only B. 3 Only
C. 1 and 2 Only D. 1 Only
Explanation : C
• Amazon Summit under the aegis of the Amazon Cooperation Treaty Organisation (ACTO)
came to the Belem City of Brazil. The Summit produced an outcome document known as the
Belem Declaration. This is only the fourth summit-level meeting under ACTO.
Amazon Cooperation Treaty Organisation (ACTO)
● Amazon Cooperation Treaty Organisation (ACTO) is an intergovernmental organisation
formed by 8 Amazonian countries: Bolivia, Brazil, Colombia, Ecuador, Guyana, Peru,
Suriname and Venezuela (8 countries).
● The above Eight nations decided to create ACTO to strengthen and implement under the
Amazon Cooperation Treaty in 1995 to strengthen and implement the Amazon
Cooperation Treaty objectives.
● The Permanent Secretariat of ACTO was established in Brasilia in 2002 and permanently
installed in 2003.
● Permanent Secretariat of ACTO is to facilitate the exchange, knowledge, cooperation and
joint projection among ACTO Member Countries to fulfil the mandate of Amazon
Cooperation Treaty.

241. Consider the following statements regarding the PUShP portal.


1. It was launched by the Ministry of New and Renewable Energy.
2. It was launched to ensure surplus availability of power during the winter season at a price
lower than the ceiling
3. This move will increase the fixed cost burden on the DISCOMs.
How many of the above given Statements is/are correct ?
A. Only one B. Only two
C. All three D. None of them
Explanation : D
● Recently,the National Power Committee (NPC) has asked States to provide suggestions
for offering incentives to buyers and sellers on the surplus power portal PUSHp.
PUShP portal:
● A High Price Day Ahead Market (HP-DAM) and Surplus Power Portal (PUShP) was
launched by the Ministry of Power.
● It was launched to ensure greater availability of power during the peak demand season
at a price higher than the ceiling of Rs 12 per unit by a certain category of sellers.
● The power distribution companies (DISCOMs) will be able to indicate their surplus power
in block times / days / months on the portal.
● Those DISCOMs who need power will be able to requisition the surplus power.

AMIGOS IAS 141


● This will reduce the fixed cost burden on the DISCOMs, and will also enable all the
available generation capacity to be utilized.
Day Ahead Market?
● It is a physical electricity trading market where power is delivered within 24 hours of the
next day starting from midnight.

242. Consider the following statements regarding a protected area that has been in the news recently
1. It is located in Pilibhit, Lakhimpur Kheri and Bahraich District of Uttar Pradesh.
2. The river Gomti originates from this area.
3. The Sharda Sagar Dam is on the boundary of this protected area.
4. The sal woodland is very dense in this area.
Choose the correct answer from the below given codes :
A. Amangarh Tiger Reserve
B. Pilibhit Tiger Reserve
C. Dudhwa Tiger Reserve
D. Ranipur Tiger Reserve
Explanation : B
Pilibhit Tiger Reserve
Pilibhit Tiger Reserve (PTR) has embarked on an ambitious journey by formulating its
inaugural 'five-year-plan' to safeguard turtles, conduct species identification and estimate
their populations both within and outside the reserve.
Pilibhit Tiger Reserve:
● It is located in Pilibhit District, Lakhimpur Kheri District and Bahraich District of Uttar
Pradesh.
● It lies along the India-Nepal border in the foothills of the Himalayas and the plains of the
‘terai’ in Uttar Pradesh.
● The river Gomti originates from the reserve, which is also the catchment of several others
like Sharda, Chuka and Mala Khannot.
● The Sharda Sagar Dam extending up to a length of 22 km (14 mi) is on the boundary of
the reserve.
● It is characterized by sal forests, tall grasslands and swamps, maintained by periodic
flooding from rivers.
● The sal woodland is very dense with good natural regeneration, amounting to almost 76%
of the reserve area.

243. It is a star discovered by the Hubble Space Telescope in 2022 and it is the most distant and
earliest known star. It is a massive B-type star that is more than twice as hot as our Sun and
about a million times more luminous.It is located in the Sunrise Arc Galaxy and is around 12.9
billion years away. The celestial body that has been in the news recently is ?

AMIGOS IAS 142


A. Farfarout B. Arrokoth
C. Kurious D. Earendel
Explanation : D
● Recently, the James Webb Space Telescope has revealed important details about
Earendel, the far away celestial body.
Earendel:
● It is a star discovered by the Hubble Space Telescope in 2022 and it is the most distant
and earliest known star.
● It is a massive B-type star that is more than twice as hot as our Sun and about a million
times more luminous.
● It is located in the Sunrise Arc Galaxy and is around 12.9 billion years away.
● Both Webb and Hubble were only able to detect it due to a natural phenomenon called
gravitational lensing.
● The star happened to be aligned behind a wrinkle in space-time created by the galaxy
cluster WHL0137-08, which is located between us and Earendel.

244. Consider the following statements regarding Takakia that has been in the news recently.
1. It is a rare genus of moss that adapted over millions of years to life at high altitudes.
2. It has developed the ability to survive frost, life-threatening UV radiation and other
hazardous conditions.
3. It combines features found in mosses, liverworts and green algae
Which of the above given Statements is/are correct ?
A. 3 Only B. 1 and 2 Only
C. 2 Only D. 1,2 and 3

Explanation : D
● Takakia that has survived for millions of years is now threatened by climate change.

Takakia:
● It is a rare genus of moss that adapted over millions of years to life at high altitudes.
● When seen from a distance, it looks like a later of moss or green algae on the rocks where
it grows. But closer inspection would reveal slender turf around one centimeter in length
with an arrangement of short, finger-like leaves.
● It is a genus that only has two species ( lepidozioides and T. ceratophylla ) and both of
them are found together only in the Tibetan Plateau.
● It has developed the ability to survive frost, life-threatening UV radiation and other
hazardous conditions.
● It combines features found in mosses, liverworts and green algae.

AMIGOS IAS 143


● It is a moss that separated from other mosses 390 million years ago, not long after the
first land plants evolved.

245. Consider the following statements regarding India Web Browser Development Challenge
(IWBDC).
1. The challenge is spearheaded by Niti Aayog in collaboration with Ministry of Science and
Technology
2. Only Indian Tech Start-ups are eligible to apply for this challenge.
3. The Challenge seeks to inspire and empower technology enthusiasts to create an
indigenous web browser.
How many of the above given Statements is/are correct ?
A. Only one B. Only two
C. All three D. None of them
Explanation : A
The Ministry of Electronics and Information Technology (MeitY) recently launched the
Indian Web Browser Development Challenge (IWBDC).

India Web Browser Development Challenge (IWBDC):


● The challenge is spearheaded by MeitY in collaboration with the Controller of Certifying
Authorities (CCA) and the Centre for Development of Advanced Computing (C-DAC)
Bangalore.
● The Challenge seeks to inspire and empower technology enthusiasts, innovators, and
developers from all corners of the country to create an indigenous web browser.
● The desired browser will have its own trust store, use a root certificate from India's CCA,
and offer cutting-edge functionalities and enhanced security & data privacy protection
features.
● As part of the challenge the government has announced a cash prize of Rs 3.4 crores for
the developers.
Who can apply ?
● The Indian Tech Start-ups, MSMEs, Companies, and LLPs registered in India under the
Companies Act 2013 can apply.
● The entity must have at least 51% shareholding with Indian citizens or persons of Indian
origin.
● The applicant's entity should not be a subsidiary company of any foreign corporation.
● Participants can apply either as “individuals” or “organization”. Members applying as
“individuals” should have a minimum of 3 and a maximum of 7 members in their Group.
● All other applicants (Startups, MSME, etc.) should apply under the “organization”
category only.

AMIGOS IAS 144


246. Consider the following statements regarding Lymphatic Filariasis.
1. It is a neglected tropical disease.
2. Wuchereria bancrofti is responsible for 90% of the cases of Lymphatic Filariasis.
3. India is committed to eradicating Lymphatic Filariasis by 2030
Which of the above given Statements is/are correct ?
A. 1 and 2 Only B. 2 and 3 Only
C. 1 and 3 Only D. 1,2 and 3
Explanation : A
• Union Health & Family Welfare Minister recently said India is committed to eradicating
Lymphatic Filariasis by 2027, surpassing the global target by three years.
Lymphatic Filariasis:
● Lymphatic filariasis, commonly known as elephantiasis, is a neglected tropical disease.
● Infection occurs when filarial parasites are transmitted to humans through mosquitoes.
● This impairs the lymphatic system and can lead to the abnormal enlargement of body
parts, causing pain, severe disability and social stigma.
● It is caused by infection with parasites classified as nematodes (roundworms) of the family
Filariodidea. There are 3 types of these thread-like filarial worms:
○ Wuchereria bancrofti, which is responsible for 90% of the cases.
○ Brugia malayi, which causes most of the remainder of the cases.
○ Brugia timori, which also causes the disease.

247. Consider the following statements regarding a solar flare .


1. It is an intense burst of radiation coming from the release of magnetic energy associated
with sunspots.
2. Flares are our solar system’s smallest explosive events.
3. They are seen as dark areas on the sun, and they can last from days to years.
How many of the above given Statements is/are correct ?
A. Only one B. Only two
C. All three D. None of them
Explanation : A
A powerful solar flare disrupted radio and navigation signals across North America recently.
Solar Flare:
● A solar flare is an intense burst of radiation coming from the release of magnetic energy
associated with sunspots.
● Flares are our solar system’s largest explosive events.
● They are seen as bright areas on the sun, and they can last from minutes to hours.

AMIGOS IAS 145


● In a matter of just a few minutes, they heat the material to many millions of degrees and
produce a burst of radiation across the electromagnetic spectrum, including from radio
waves to x-rays and gamma rays.
● Although solar flares can be visible in white light, they are often more readily noticed via
their bright X-ray and ultraviolet emissions.

248. Consider the following statements regarding the Cott-Ally App.


1. It is developed by the the Cotton Corporation of India Limited.
2. The app provides direct interaction with CCI through Live chat to resolve doubts &
grievances.
3. It aims to maximise the use of the digital media among the farmers.
Which of the above given Statements is/are correct ?
A. 1 and 2 Only B. 2 and 3 Only
C. 1 and 3 Only D. 1,2 and 3

Explanation : D
Recently, the Union Minister of State for Ministry of Textiles informed Lok Sabha about
Cott-Ally App.
Cott-Ally App:
● The Cotton Corporation of India Limited I has developed a mobile app - “Cott-Ally”
exclusively for the cotton farmers.
● To maximise the use of the digital media among the farmers to increase the reach of all
government schemes especially in the cotton farming sector.
● To have direct interaction and outreach with the cotton farmers in their regional
languages.
● The various features of “Cott-Ally” are as follows:
○ Easy to operate in regional languages.
○ MSP cotton rates state-wise, variety-wise and quality-wise can be seen.
○ Farmers can track real time payment status of their kapas sold to CCI under MSP.

249. The “Exercise Zayed Talwar” is a Bilateral Exercise between ?


A. India and Kazakhstan B. India and Saudi Arabia
C. India and UAE D. India and Kyrgyzstan
Explanation : C

Exercise Zayed Talwar:


● Two ships of the Indian Navy, INS Visakhapatnam, and INS Trikand are participating in the
exercise.

AMIGOS IAS 146


● During the visit, the ships will undertake professional interactions with UAE Naval Force
on multiple elements of maritime operations.
● They will further share best practices to enhance cooperation and strengthen ties
between the two navies.
● This exercise will boost the maritime partnership between the two navies and foster a
common understanding of the security challenges in the region.

250. Consider the following statements regarding the Cash Reserve Ratio (CRR).
1. It is the share of a bank’s total deposit that is mandated by the Reserve Bank of India (RBI)
to be maintained with the latter as reserves in the form of liquid cash.
2. CRR applies to scheduled commercial banks, Regional Rural Banks and NBFCs.
3. The bank cannot use this amount for lending and investment purposes
How many of the above given Statements is/are correct ?
A. Only one B. Only two
C. All three D. None of them
Explanation : B
The Reserve Bank of India (RBI) recently asked Indian banks to set aside a larger share of
incremental deposits under the Cash Reserve Ratio (CRR).
Cash Reserve Ratio (CRR):
● Cash Reserve Ratio (CRR) is the share of a bank’s total deposit that is mandated by the
Reserve Bank of India (RBI) to be maintained with the latter as reserves in the form of
liquid cash.
● The bank cannot use this amount for lending and investment purposes and does not get
any interest from the RBI.
● CRR applies to scheduled commercial banks, while the regional rural banks and NBFCs are
excluded.

251. Consider the following statements regarding Raisina Dialogue.


1. It is modeled on the lines of the Munich Security Conference and Shangri-La Dialogue.
2. It is held annually since 2016 in Singapore.
3. It is organized by the Observer Research Foundation (ORF).
Which of the above given Statements is/are correct ?
A. 1 and 2 Only B. 2 and 3 Only
C. 1 and 3 Only D. 1,2 and 3
Explanation : C

AMIGOS IAS 147


Foreign Secretary recently told a Parliament committee that the Raisina Dialogue is now
among the most significant global level think tank events in the world.
Raisina Dialogue:
● It is India’s premier conference on geopolitics and geoeconomics, committed to
addressing the most challenging issues facing the global community.
● It is modeled on the lines of the Munich Security Conference and Singapore’s Shangri-La
Dialogue.
● It is held annually since 2016 in New Delhi.
● The Dialogue is structured as a multi-stakeholder, cross-sectoral discussion involving
heads of state, cabinet ministers, and local government officials, who are joined by
thought leaders from the private sector, media, and academia.
● It is organized by the Observer Research Foundation (ORF) in partnership with the
Ministry of External Affairs, Government of India.

252. Consider the following statements regarding the National Pharmaceutical Pricing Authority
(NPPA).
1. It was constituted vide Government of India Resolution dated 29th August 1997
2. It is an attached office of the Department of Pharmaceuticals (DoP), Ministry of Chemicals
& Fertilizers.
Which of the above given Statements is/are correct ?
A. 1 only
B. 2 Only
C. Both 1 and 2
D. Neither 1 nor 2
Explanation : C
• Drug price regulator National Pharmaceutical Pricing Authority (NPPA) recently fixed the
retail prices of 44 new drug formulations.

National Pharmaceutical Pricing Authority (NPPA):
● It is a government regulatory agency established with the aim of controlling and
regulating the prices of pharmaceutical drugs and medical devices in the country to
ensure their availability at affordable rates.
● It was constituted vide Government of India Resolution dated 29th August 1997 as an
attached office of the Department of Pharmaceuticals (DoP), Ministry of Chemicals &
Fertilizers.

253. Consider the following statements regarding Small Modular Reactors (SMRs).
1. They are advanced nuclear reactors that have a power capacity of up to 5000 MW per
unit.

AMIGOS IAS 148


2. SMRs are designed with a higher core damage frequency
3. SMRs can generate less electricity when demand is down and are particularly useful for
power generation in remote locations.
How many of the above given Statements is/are correct ?
A. Only one B. Only two
C. All three D. None of them
Explanation : A
Small Modular Reactors (SMRs) are advanced nuclear reactors that have a power capacity
of up to 300 MW per unit, which is about one-third of the generating capacity of
traditional nuclear power reactors.
● SMRs are designed to operate for 40-60 years with capacity factors exceeding 90%.
● They can produce a large amount of low-carbon electricity. They could be paired with and
increase the efficiency of renewable sources in a hybrid energy system.
● SMRs can generate less electricity when demand is down and are particularly useful for
power generation in remote locations.
● SMRs can be installed in decommissioned thermal power plant sites by repurposing
existing infrastructure.
Safety features of SMRs:
● SMRs are designed with a smaller core damage frequency (the likelihood that an accident
will damage the nuclear fuel) and source term (a measure of radioactive contamination)
compared to conventional NPPs.
● They also include enhanced seismic isolation for more safety.
● SMR designs are also simpler and include several passive safety features, resulting in a
lower potential for the uncontrolled release of radioactive materials into the
environment.

254. Consider the following statements regarding the National Social Assistance Programme (NSAP).
1. It was launched in 1995 as a Centrally Sponsored Scheme of the Ministry of Rural
Development, Government of India.
2. The National Maternity Benefit Scheme (NMBS) is also part of the scheme.
3. It is a significant step towards the fulfillment of the Directive Principle in Article 41 of the
Indian Constitution.
How many of the above given Statements is/are correct ?
A. Only one B. Only two
C. All three D. None of them
Explanation : B
NSAP
● It was launched in 1995 as a Centrally Sponsored Scheme of the MoRD, Government of
India.

AMIGOS IAS 149


● It provides financial assistance [to Below Poverty Line (BPL) individuals] to the elderly,
widows and persons with disabilities and bereaved families on death of primary
breadwinner, in the form of social pensions.
● It represents a significant step towards the fulfillment of the Directive Principles in Article
41 of the Indian Constitution.
● Article 41 directs the State to provide public assistance to its citizens in case of
unemployment, old age, sickness, disablement, etc., within the limit of its economic
capacity and development.
● At its inception, NSAP had three components namely National Old Age Pension Scheme
(NOAPS), National Family Benefit Scheme (NFBS) and National Maternity Benefit Scheme
(NMBS).
● The National Maternity Benefit Scheme (NMBS) was subsequently transferred to the
Ministry of Health and Family Welfare in 2001.

255. Consider the following statements regarding the UDYAM ASSIST PLATFORM (UAP).
1. The Government launched UAP under the MSME formalization project to bring Informal
Micro Enterprises (IMEs) under the formal ambit.
2. The platform has been developed by the State Bank of India.
3. As per the Government of India IMEs are those enterprises which are unable to get
registered on the Udyam Registration Portal (URP)
How many of the above given Statements is/are correct ?
A. Only one B. Only two
C. All three D. None of them
Explanation : B
● The Government launched UAP under the MSME formalization project on 11.01.2023 to
bring Informal Micro Enterprises (IMEs) under the formal ambit for availing benefits under
Priority Sector Lending.
● The platform has been developed by the Small Industries Development Bank of India
(SIDBI).
● The Government of India has clarified that IMEs are those enterprises which are unable
to get registered on the Udyam Registration Portal (URP) due to lack of mandatory
required documents such as Permanent Account Number (PAN) or Goods and Services
Tax Identification Number (GSTIN).

256. Consider the following statements regarding the Committee of Privileges in Parliament.
1. The Constitution, has defined the powers, privileges and immunities of Parliament and
MP's.
2. The Privileges committee consists of 15 members in Lok Sabha elected by the memebers
among themselves.
3. In the Rajya Sabha, the Chairperson heads the committee of privileges.

AMIGOS IAS 150


How many of the above given Statements is/are correct ?
A. Only one B. Only two
C. All three D. None of them
Explanation : D
● Rajya Sabha Chairman recently directed the Rajya Sabha Privileges Committee to
investigate the complaints filed by four MPs who accused another member of proposing
their name for a House panel without their consent in violation of rules.
● According to the Constitution, the powers, privileges and immunities of Parliament and
MP's are to be defined by Parliament.
● No law has so far been enacted in this respect. In the absence of any such law, it continues
to be governed by British Parliamentary conventions.
● This committee consists of 15 members in Lok Sabha (10 in case of Rajya Sabha)
nominated by the Speaker(Chairman in case of Rajya Sabha).
● In the Rajya Sabha, the deputy chairperson heads the committee of privileges.

257. Athidhi Portal launched by the Government of Kerala has been in the news recently . It was
launched for the purpose of ?
A. Solving Drug menace B. Registration of Migrant workers
C. Reducing crimes against foreign tourists D. Enhancing ease of doing business
Explanation : B
• Athidhi Portal was launched by State government of Kerala to expedite the registration
process for migrant workers.

258. Consider the following statements regarding the Solar Power Capacity of India.
1. India ranks second in Asia and third globally for new solar PV capacity.
2. Gujarat is the largest producer of Solar energy in India
3. India aims for 175 GW renewable capacity by 2022 and 500 GW by 2030.
Which of the above given Statements is/are correct ?
A. 1 and 2 Only B. 2 and 3 Only
C. 1 and 3 Only D. 1,2 and 3
Explanation : C
● According to a reply in the Parliament, the total installed capacity of India is 70.1 GW.
● Solar Potential Assessment: National Institute of Solar Energy projects India's solar
potential at 748 GW, considering 3% wasteland coverage with Solar PV modules.
● Ambitious Renewable Goals: India aims for 175 GW renewable capacity by 2022 and 500
GW by 2030, marking the world's largest expansion plan in renewable energy.

AMIGOS IAS 151


● Solar PV Leadership: India ranks second in Asia and third globally for new solar PV capacity
(13 GW added in 2021); fourth in total installations (60.4 GW), surpassing Germany (59.2
GW) for the first time.
● Leading solar energy producers in India: Rajasthan (17839.98 MW), Gujarat (10133.66
MW), Andhra Pradesh (4552.12 MW), Tamil Nadu (6892.81 MW), Karnataka (9050.59
MW).

259. The declaration was released during the Amazon Summit and recognizes Indigenous knowledge
as a condition for biodiversity conservation. It calls for ensuring full and effective participation of
Indigenous Peoples in decision-making and public policy formulation processes. It also promotes
sustainable use of biodiversity resources in the Amazon. The declaration being discussed is ?
A. Stockholm Declaration B. Rio Declaration
C. Belem Declaration D. Copenhagen declaration
Explanation : C
Belem Declaration
● The summit was hosted by Brazil which aimed at saving the rainforest of the region.
● The leaders from the eight amazon countries failed to agree on the goal to protect the
rainforest at the ongoing Amazon Summit organised by the Amazon Cooperation Treaty
Organization (ACTO).
● Amazon countries – Bolivia, Brazil, Colombia, Ecuador, Guyana, Peru, Suriname and
Venezuela.
● ACTO – ACTO is an international organisation aimed at the promotion of sustainable
development of the Amazon Basin.

260. Consider the following statements :


Statement-I :
Recently, scientists at the Indian Institute of Astrophysics have discovered a unique star
named HE 1005-1439 classified as a carbon-enhanced metal-poor (CEMP) star.

Statement-II :
These stars are characterized by diverse heavy elements abundance patterns and are
primarily classified into four groups, based on which groups of heavy elements are more
abundant.
Which one of the following is correct in respect of the above statements?
A. Both Statement-I and Statement-II are correct and Statement-II is the correct explanation
for Statement-I
B. Both Statement-I and Statement-II are correct and Statement-II is not the correct
explanation for Statement-I
C. Statement-I is correct but Statement-II is incorrect
D. Statement-I is incorrect but Statement-II is correct

AMIGOS IAS 152


Explanation : A
● Recently, scientists at the Indian Institute of Astrophysics (IIA, Bangalore) under the
Department of Science & Technology (DST), Govt. of India, have discovered a unique star
named HE 1005-1439 classified as a carbon-enhanced metal-poor (CEMP) star.
Carbon-enhanced metal-poor (CEMP) star:
● These stars are characterized by diverse heavy elements abundance patterns and are
primarily classified into four groups, based on which groups of heavy elements are more
abundant.
● These are mostly dwarf stars, sub-giant stars, or giant stars, and stars that belong to these
evolutionary stages cannot produce elements heavier than iron.
261. Consider the following statements regarding “Swathi Mountains”.
1. It is an advanced electronically scanned phased array radar.
2. It was developed by Defence Research and Development Organisation
3. It is specifically designed for operations in mountainous and high-altitude areas.
Which of the above given Statements is/are correct?
A. 1 and 2 Only B. 3 Only
C. 1 and 3 Only D. 2 only
Explanation: C
Indian Army inducted the lighter and more compact version of the indigenously developed Weapon
Locating Radar (WLR-M) called "Swathi Mountains."

Swathi Mountains:
● The Swathi Mountains WLR is an advanced electronically scanned phased array radar.
● It is specifically designed for operations in mountainous and high-altitude areas.
● It was developed by Bharat Electronics Limited (BEL) in Bengaluru.
● Weapon Locating Radar (WLR) stands as a critical asset for modern militaries, employing
advanced signal processing techniques to autonomously detect and track hostile artillery,
mortars, and rocket launchers.

262. Consider the following statements with reference to the “Durand Cup”.
1. It is Asia’s oldest football tournament, in which top Indian football clubs from across the
country participate.
2. The inaugural edition took place in Shimla in 1998
3. The tournament is named after its founder Sir Henry Mortimer Radcliffe.
4. The Durand Cup tournament is unique where the winning team walks away with three
trophies.
How many of the above given Statements is/are correct?

AMIGOS IAS 153


A. Only one B. Only two
C. Only three D. All four
Explanation: B
Recently, the union Ministry of Defence inaugurated the 132nd edition of Durand Cup in Kokrajhar.

Durand Cup:
● It is Asia’s oldest and the world’s third oldest football tournament, in which top Indian football
clubs from across the country participate.
● The inaugural edition took place in Shimla in 1888, when it started off as an Army Cup, open
only to the British Indian Army troops in India.
● The tournament is named after its founder Sir Henry Mortimer Durand.
● The Durand Cup tournament is unique where in the winning team walks away with three
trophies,
○ The Durand Cup (a rolling trophy and the original prize).
○ The Shimla Trophy (also a rolling trophy and first given by the residents of Shimla in
1904).
○ The President's Cup (for permanent keep and first presented by Dr. Rajendra Prasad,
India’s first President, in 1956).

263. Consider the following statements regarding the NIDHI Program.


1. It is an umbrella programme conceived and developed by the Innovation & Entrepreneurship
division, Department of Science & Technology.
2. It is funded by the Science and Engineering Research Board (SERB)
Which of the above given Statements is/are incorrect?
A. 1 Only B. 2 Only
C. Both 1 and 2 D. Neither 1 nor 2
Explanation: B
Recently, the union Minister of State (Independent Charge) Science & Technology informed the
Rajya Sabha about the NIDHI Program.
NIDHI Program:
● The National Initiative for Developing and Harnessing innovations (NIDHI) program was
launched in 2016.
● It is an umbrella programme conceived and developed by the Innovation & Entrepreneurship
division, Department of Science & Technology.
● Aim: To nurture start-ups through scouting, supporting and scaling innovations.
● Key stakeholders: Various departments and ministries of the central government, state
governments, academic and R & D institutions, mentors, financial institutions, angel
investors, venture capitalists and private sectors.

AMIGOS IAS 154


● Funding: By the National Science & Technology Entrepreneurship Development Board
(NSTEDB).

264. Consider the following statements regarding the Committee of the Parliament on Official
Language.
1. It was set up in 1976 under Section 4 of The Official Languages Act, 1963.
2. Under the provisions of the 1963 Act, the panel submits its report to the Parliament.
3. The Committee is chaired by the Union Minister for Language and culture.
4. The committee consists of 30 members all hailing from the Lok Sabha
How many of the above given Statements is/are incorrect?
A. Only one B. Only two
C. Only three D. All four
Explanation: C
Recently, the Union Home Minister chaired the 38th meeting of the Committee of Parliament on
Official Language.

Committee of Parliament on Official Language:


● The Committee of Parliament on Official Language was set up in 1976 under Section 4 of The
Official Languages Act, 1963.
● Section 4 of the Act says “there shall be constituted a Committee on Official language, on a
resolution to that effect being moved in either House of Parliament with the previous
sanction of the President and passed by both Houses”.
● Under the provisions of the 1963 Act, the panel submits its report to the President, who “shall
cause the report to be laid before each House of Parliament, and sent to all the State
Governments”.
● The Committee is chaired by the Union Home Minister, and has, in accordance with the
provisions of the 1963 Act.
● The committee consists of 30 members — 20 MPs from Lok Sabha and 10 MPs from Rajya
Sabha.
● The mandate of the Committee is to review the progress made in the use of Hindi for official
purposes, and to make recommendations to increase the use of Hindi in official
communications.
● The first Report of the Committee was submitted in 1987.

265. Scientists and engineers need devices that can process information efficiently with ultra-low
power consumption. Recently, there have been new developments in logic and memory devices
that use the spin of electrons, in addition to their electronic charge, to store and process
information. To accomplish this, the device architecture needs a strong interaction between the
spin of electrons and their orbital moments. This coupling is called as?
A. Resonance Effect B. Hyperconjugation

AMIGOS IAS 155


C. Electromeric effect D. Rashba effect

Explanation: D
● Using some 2-D carbides or nitrides of transition metals, a team of scientists have
computationally designed a new composite quantum material that exhibits an exotic
quantum property called Rashba splitting, in colossal scale, in a metallic environment.
● Rashba effect allows for easy manipulation of spin currents and can lead to lower energy
consumption. In particular, it can facilitate voltage-driven magnetization switching for logic
and memory computing with reduced energy consumption, as proposed by Intel. However,
to be useful in this regard, this effect must be significantly large even at zero applied voltages,
and this has been difficult to realize in traditional semiconducting and metallic systems.

266. The Jeddah peace summit follows talks in Copenhagen in June that were designed to be informal
and did not yield an official statement. The Jeddah talks are for peace between?
A. Israel and Palestine
B. Russia and Ukraine
C. Afghanistan and Taliban
D. China and Taiwan
Explanation: B
● Saudi Arabia hosted Ukraine, the U.S., some European countries and major developing
countries including India and Brazil for peace talks on the Russia-Ukraine war.
● The aim of this summit was to reach an agreement on key principles for a peaceful end to
Russia's war in Ukraine.
● The two-day summit in Jeddah was the second of its kind, after a similar forum in Copenhagen
earlier this summer, and aims to draft key principles on how to end Russia’s war in Ukraine.

267. Consider the following statements regarding the Government of National Capital Territory of
Delhi (Amendment) Bill,2023.
1. The Bill establishes the National Capital Civil Services Authority to make recommendations to
the LG on certain matters related to services.
2. In the case of a difference of opinion between the LG and the Authority, the latter's decision
will be final.
3. Under the Act LG should not act on the matters outside the legislative competence of the
Delhi Legislative Assembly.
Which of the above given Statements is/are correct?
A. 1 Only B. 3 Only
C. 2 and 3 Only D. 1, 2 and 3
Explanation: A

AMIGOS IAS 156


● The Government of National Capital Territory of Delhi (Amendment) Bill, 2023, which
replaces the ordinance on control of services in Delhi, has been passed by both the Houses
of the Parliament recently.
Key Features of the Bill –
1. National Capital Civil Services Authority (NCCSA) –
The Bill establishes the National Capital Civil Services Authority to make recommendations to
the LG on certain matters related to services.
2. Under the Act, matters where the LG may act on his discretion are: (i) matters outside the
legislative competence of the Delhi Legislative Assembly but which have been delegated to
the LG, or (ii) matters where he is required by a law to act in his discretion or exercise any
judicial or quasi-judicial functions.
3. It expands the discretionary role of the LG by giving him powers to approve the
recommendations of the Authority, or return them for reconsideration.
4. In the case of a difference of opinion between the LG and the Authority, the former’s decision
will be final.

268. Consider the following statements regarding the World Breastfeeding Week 2023.
1. World Breastfeeding Week, observed annually during the first week of August in
commemoration of the 1990 Innocenti Declaration.
2. In 1991, the World Alliance for Breastfeeding Action (WABA) was formed as a global network.
3. Since 2016, WBW has been aligned with the Sustainable Development Goals (SDGs).
How many of the above given Statements is/are correct?
A. Only one B. Only two
C. All three D. None of them
Explanation: C
On the occasion of World Breastfeeding Week 2023, the United Nations Children's Fund (UNICEF)
and the World Health Organization (WHO) lauded the progress made by several countries in
increasing exclusive breastfeeding rates, while also highlighting the potential for further
advancements if breastfeeding is protected and supported, particularly in the workplace.

World Breastfeeding Week


● World Breastfeeding Week, observed annually during the first week of August in
commemoration of the 1990 Innocenti Declaration.
● In 1990, the Innocenti Declaration was signed by government policymakers, UN health
agencies, and other organisations to protect, promote, and support breastfeeding.
● In 1991, the World Alliance for Breastfeeding Action (WABA) was formed as a global network,
and since 1992, the world has marked Breastfeeding Week, annually.
● Since 2016, WBW has been aligned with the Sustainable Development Goals (SDGs).
● Breastfeeding can help achieve many of the 17 SDGs, including goals on poverty, hunger,
health, education, gender equality, and sustainable consumption.

AMIGOS IAS 157


● Theme 2023: "Let's make breastfeeding at work, work”.

269. Consider the following statements regarding India's performance in the recently held “World
Archery Championship”.

1. Indian women's compound team secured India's inaugural Gold medal at the event.
2. Parneet Kaur became the youngest-ever senior world champion and the first Indian to win
an individual compound gold medal
3. Brussels hosted the championship this year.

How many of the above given Statements is/are correct?


A. Only one B. Only two
C. All three D. None of them

Explanation: A
● The 2023 World Archery Championships was held from 31 July to 6 August 2023 in Berlin,
Germany. The event served as a qualification event for the 2024 Summer Olympics in Paris,
France.
● The Prime Minister extended congratulations to the Indian women's compound team
comprising Jyothi Surekha Vennam, Parneet Kaur, and Aditi Gopichand Swami, for securing
India's inaugural Gold medal at the World Archery Championship in Berlin.
● Aditi Gopichand Swami became the youngest-ever senior world champion and the first Indian
to win an individual compound gold medal at the World Archery Championships at the age
of 17.
● World Archery is based in the Olympic capital of Lausanne, Switzerland.

270. Consider the following statements regarding the Endemic Birds of India.
1. 15% of Indian bird species are endemic and are not reported in other parts of the world.
2. Jerdon’s Courser is one of the endemic birds to India.
3. The highest number of endemic species have been recorded in the Andaman and Nicobar
Islands.
How many of the above given Statements is/are correct?
A. Only one B. Only two
C. All three D. None of them
Explanation: A
● The publication, titled 75 Endemic Birds of India, was recently released on the 108th
foundation day of the ZSI.

AMIGOS IAS 158


● India is home to 1,353 bird species, which represents approximately 12.40% of global bird
diversity
● Of these 1,353 bird species, 78 (5%) are endemic to the country and are not reported in other
parts of the world.
● Of the 78 species, three species have not been recorded in the last few decades.
● They are the Manipur Bush Quail, Himalayan Quail, and the Jerdon’s Courser.
● The highest number of endemic species have been recorded in the Western Ghats, with 28
bird species.
● Andaman and Nicobar Islands: 25 bird species are endemic to the Andaman and Nicobar
Islands.
271. Consider the following statements regarding the JALDOST airboat.
1. It is designed to remove excess aquatic weed and floating waste from water bodies.
2. It has a closed airtight pontoon type hull to make it inherently unsinkable.
3. It is developed by the Hindustan Aeronautics Limited (HAL).
Which of the above given Statements is/are correct ?
A. 1 and 2 Only B. 2 and 3 Only
C. 1 and 3 Only D. 1,2 and 3
Explanation : A
Recently, the National Aerospace Laboratories (NAL) unveiled the JALDOST airboat..
JALDOST:
● It is an airboat that operates on water.
● It is designed to remove excess aquatic weed and floating waste from water bodies.
● It has a closed airtight pontoon type hull to make it inherently unsinkable.
● According to NAL, it has a hybrid propulsion system, comprising air propulsion and paddle
wheel propulsion.
● The ability to travel through weed makes JALDOST an ideal platform to collect them and
bring them to the shore.
● A steel mesh belt conveyor system fixed in the front collects the waste. The collected
waste falls on the horizontal deck conveyor.
● After reaching the shore, the collected waste is unloaded by a rear conveyor system to
trucks or tractors.

272. Consider the following statements regarding the MASI Portal.


1. It was developed for synchronous monitoring of the Child Care Institutions (CCIs) and their
inspection mechanisms across the country.
2. This application has been developed by the Aryabhatta Research Institute of
Observational Sciences (ARIES), Nainital.
Which of the above given Statements is/are correct ?

AMIGOS IAS 159


A. 1 only B. 2 Only
C. Both 1 and 2 D. Neither 1 nor 2
Explanation : A
Recently, the Minister of Women and Child Development informed the Rajya Sabha about
MASI Portal.

MASI Portal:
● Monitoring App for Seamless Inspection (MASI) was developed for synchronous
monitoring of the Child Care Institutions (CCIs) and their inspection mechanisms across
the country.
● The National Commission for Protection of Child Rights (NCPCR) has developed this
application.
● The effective and efficient functioning of the mechanism for inspection of CCIs provided
under the Juvenile Justice Act, 2015 (as amended in 2021).
● The app is linked to the monitoring Portal where the automatic reports are generated.

273. Consider the following statements with reference to Ayush Visa.


1. It is specifically designed for foreign nationals looking to receive medical treatment in
India through traditional Indian systems of medicine.
2. The introduction of the Ayush Visa is part of India's roadmap for the Heal in India initiative.
Which of the above given Statements is/are correct ?
A. 1 Only B. 2 Only
C. Both 1 and 2 D. Neither 1 nor 2
Explanation : C
The Ministry of Home Affairs recently notified the creation of a new category of Ayush
visa for foreign nationals for treatment under Ayush systems/Indian systems of medicine.
Ayush Visa:
● It is specifically designed for foreign nationals looking to receive medical treatment in
India through traditional Indian systems of medicine.
● This visa aims to cater to those interested in Ayurveda, Yoga, and other traditional forms
of therapeutic care and wellness.
● A new chapter, e., Chapter 11A, Ayush Visa, has been incorporated after Chapter 11 -
Medical Visa of the Visa Manual, which deals with treatment under the Indian systems of
medicine, and accordingly, necessary amendments have been made in various chapters
of the Visa Manual, 2019.
● The introduction of the Ayush Visa category is part of the country's roadmap for the Heal
in India initiative.

AMIGOS IAS 160


● Heal in India initiative: It seeks to provide “integrated and holistic treatment” to the world
in India and enhance patient mobility for access to world-class, affordable, and quality
healthcare services”.
274. Consider the following statements regarding the Kuril Islands.
1. These are a set of four islands situated between the Sea of Okhotsk and the Pacific Ocean.
2. They are part of the Pacific Ring of Fire
3. The Soviet Union had seized the islands at the end of World War II
Which of the above given Statements is/are correct ?
A. 1 and 2 Only B. 3 Only
C. 2 Only D. 1,2 and 3
Explanation : D
In Japan, conservative voices are hinting that the Russian and Ukraine war could give
Japan a chance to take control of the disputed Kuril Islands.
Kuril Islands
● These are a set of four islands situated between the Sea of Okhotsk and the Pacific Ocean
near the north of Japan's northernmost prefecture, Hokkaido.
● Japan refers to them as Northern territories, Russia calls them the Kuril Islands and South
Korea named them as Dokdo islands.
● These are part of the Pacific Ring of Fire belt and have over 100 volcanoes, of which 35
are said to be active volcanoes along with hot springs.
● Both Russia and Japan claim sovereignty over them though the islands have been under
Russian control since the end of World War II.
● The Soviet Union had seized the islands at the end of World War II and by 1949 had
expelled its Japanese residents.

275. According to a new study some people of African descent possess a CHD1L gene variant that
likely controls a major disease. Which of the following is the correct answer?
A. Cancer B. HIV
C. Diabetes D. Alzheimer's
Explanation : B
CHD1L gene:
● The Chromodomain Helicase DNA Binding Protein 1 Like (CHD1L), which contains
information to make proteins that allow the body to repair DNA damage.
● A variant of the CHD1L gene, specifically present in the African population, has been
linked to the reduced viral load (amount of HIV in the blood) of the most common and
virulent type of HIV, called HIV-1 (more common & severe retrovirus compared to HIV-2).
● They analysed the DNA of almost 4,000 people of African ancestry living with HIV-1 and
found a gene variant CHD1L on chromosome 1.

AMIGOS IAS 161


● People carrying this variant had a low viral load. This lowers their risk of spreading the
virus and slowing the progress of their own illness.
● The researchers said between 4 per cent and 13 percent of people of African origin could
be carrying this particular variant.
276. Rajmarg Yatra Application has been in the news recently. Consider the following statements
regarding the application.
1. It empowers travellers with comprehensive information on Indian National Highways
2. This is a user-friendly app which is now available for download on both Google Play Store
and iOS App Store.
3. The app is available in all Indian languages.
Which of the above given Statements is/are correct ?
A. 1 and 3 Only B. 2 Only
C. 3 Only D. 1 and 2 Only
Explanation : D
Recently, the National Highways Authority of India (NHAI) takes a significant stride
towards improving the highway user experience with the launch of 'Rajmargyatra
application.
Rajmargyatra Application:
● It empowers travellers with comprehensive information on Indian National Highways
while also offering an efficient complaint redressal system.
● This is a user-friendly app which is now available for download on both Google Play Store
and iOS App Store.
● The app is currently available in Hindi and English.

277. The e-CARe portal was launched recently. Consider the following statements regarding the
portal.
1. The portal will facilitate easy and swift transportation of the deceased Indians across
international borders.
2. The purpose of a dedicated portal is to remove the barriers
3. A nodal officer will review them in a time-bound manner of 48 hours
Which of the above given Statements is/are correct ?
A. 1 and 3 Only B. 2 Only
C. 2 and 3 Only D. 1,2 and 3
Explanation : D
● The e-CARe (e-Clearance for Afterlife Remains) portal was launched recently.
e-CARe Portal

AMIGOS IAS 162


● The portal will facilitate easy and swift transportation of the deceased Indians across
international borders.
● The existing system has many barriers which cause delays. The purpose of a dedicated
portal is to remove the barriers
● A nodal officer will review them in a time-bound manner and approve the application
within 48 hours.

278. Consider the following statements regarding the Small Modular Reactors (SMRs).
1. These are advanced nuclear reactors that have a power capacity of up to 1000 MW(e) per
unit.
2. Power plants based on SMRs may require more frequent refueling, every 1 to 2 years
3. Some SMRs are designed to operate for up to 30 years without refuelling.
Which of the above given Statements is/are correct ?
A. 1 and 2 Only B. 2 Only
C. 2 and 3 Only D. 3 Only
Explanation : D
● India is exploring the options of collaborating with other countries and taking up
indigenous development of Small Modular Reactors (SMRs).
● For this, the provisions of Atomic Energy Act, 1962 are being examined to allow
participation of private sector and start-ups to promote SMRs technology in the country.
Small Modular Reactors (SMRs):
● These are advanced nuclear reactors that have a power capacity of up to 300 MW(e) per
unit, which is about one-third of the generating capacity of traditional nuclear power
reactors.
● Power plants based on SMRs may require less frequent refuelling, every 3 to 7 years, in
comparison to between 1 and 2 years for conventional plants. Some SMRs are designed
to operate for up to 30 years without refuelling.

279. According to a paper released recently, the South Korean team has created a groundbreaking
new material, LK-99. What is LK-99 ?
A. A cutting edge Missile technology B. A new clean fuel technology
C. An advanced Superconductor D. A new treatment method for Cancer

Explanation : C
● According to a paper released recently, the South Korean team has created a
groundbreaking new material, LK-99, a room-temperature superconductor working at
ambient pressure.

280. Consider the following statements regarding the Desiccation-tolerant vascular plant.

AMIGOS IAS 163


1. They are able to withstand extreme dehydration, losing up to 95% of their water content.
2. The global population of these species ranges between 300 and 1,500,
3. They are spread mainly in rock outcrops and partially shaded tree trunks inside the
forests.
4. The genes of these plants could be used to create a high-temperature tolerant variety of
crops
How many of the above given Statements is/are correct ?
A. Only one B. Only two
C. Only three D. All four

Explanation : D
New study discovers 62 desiccation-tolerant vascular plant species in India's Western
Ghats, with potential applications in agriculture & conservation.
Desiccation-tolerant vascular plant
● They are able to withstand extreme dehydration, losing up to 95% of their water content,
and they revive themselves once water is available again.
● This unique ability allows them to survive in harsh, arid environments that would be
uninhabitable for most other plants.
● DT plant varieties are found in both flowering and non-flowering species and in both
temperate and tropical climates.
● The global population of these species ranges between 300 and 1,500,
● They are spread mainly in rock outcrops and partially shaded tree trunks inside the
forests.
● They have been studied for their possible applications in agriculture, particularly in areas
with limited water resources.
● The genes of these plants could be used to create a high-temperature tolerant variety of
crops to improve climate resilience and ensure food security for the masses.

281. Consider the following statements regarding the Performance of India towards Tiger
Conservation:
1. As per 5th tiger census India’s tiger population increased to 3,682 in 2022 from 1,411 in
2006.
2. In 2022, the maximum number of tigers (785) were reported to be in Karnataka.
3. Nearly a quarter of the tigers were reportedly inside protected areas in India.
How many of the above given Statements is/are correct ?
A. Only one B. Only two
C. All three D. None
Explanation : A

AMIGOS IAS 164


● July 29 is celebrated world over as the International Tiger Day in a bid to raise awareness
on various issues surrounding tiger conservation.
Performance of India towards Tiger Conservation:
● As per the World Wildlife Fund (WWF), while countries in Southeast Asia struggled to
control population decline, India fared much better.
● According to the Wildlife Institute of India's (WII) 5th tiger census (quadrennial), India’s
tiger population increased to 3,682 in 2022 (revised from 3,167 recently), up from 1,411
in 2006.
● In 2022, the maximum number of tigers (785) were reported to be in MP, followed by
Karnataka (563), Uttarakhand (560), and Maharashtra (444).
● Nearly a quarter of the tigers were reportedly outside protected areas.

282. Consider the following statements regarding Government e-Marketplace (GeM).


1. GeM is an online platform for public procurement, launched in 2016 by the Ministry of
Commerce and Industry (MoC&I).
2. The platform is owned by Central and state governments with equal share
3. It was created with the objective to create an open and transparent procurement
platform for government buyers.

How many of the above given Statements is/are correct ?


A. Only one B. Only two
C. All three D. None

Explanation : B
While the Ministry of S&T has announced its intent to galvanise research in India through
the National Research Foundation Bill 2023, scientists say that the mandatory
procurement via GeM is a major stumbling block.

Government e-Marketplace (GeM)


● GeM is an online platform for public procurement, launched in 2016 by the Ministry of
Commerce and Industry (MoC&I), Government of India.
● It was created with the objective to create an open and transparent procurement
platform for government buyers to facilitate the online procurement of goods and
services.
● The purchases through GeM by Government users have been authorised and made
mandatory by the Ministry of Finance under the General Financial Rules, 2017.
● The platform is owned by GeM SPV (Special Purpose Vehicle) which is a 100%
Government-owned, non-profit company under the MoC&I.

AMIGOS IAS 165


● The portal features over 11,000 product categories with more than 29 lakh listed
products, as well as over 270 service categories with more than 2.5 lakh service offerings.

283. Consider the following statements regarding Graded Response Action Plan (GRAP).
1. The GRAP was first notified in January 2017 by the Ministry of Environment, Forest and
Climate Change.
2. The Commission for Air Quality Management (CAQM) has constituted a sub-committee
for the operationalization of the GRAP.
3. It is a set of emergency measures that kick in to prevent further deterioration of air quality
all over India.
Which of the above given Statements is/are correct ?
A. 1 and 2 Only B. 3 Only
C. 2 and 3 Only D. 2 Only
Explanation : A
Recently, the Commission for Air Quality Management in NCR & Adjoining Areas (CAQM)
announced revision of the Graded Response Action Plan (GRAP) which will come into
force w.e.f. 1st October, 2023 in the entire National Capital Region.
Graded Response Action Plan:
● It is a set of emergency measures that kick in to prevent further deterioration of air quality
once it reaches a certain threshold in the Delhi-NCR region.
● The GRAP was first notified in January 2017 by the Ministry of Environment, Forest and
Climate Change.
● The Commission for Air Quality Management (CAQM) has constituted a sub-committee
for the operationalization of the GRAP.
● This body includes officials from the CAQM, member secretaries of pollution control
boards of Uttar Pradesh, Delhi, Haryana, Rajasthan, the Central Pollution Control Board,
a scientist from the IMD and one from the IITM and Health Advisor.
● The sub-committee is required to meet frequently to issue orders to invoke the GRAP.
● The orders and directions of the CAQM will prevail in case of any conflict between
directions issued by the State governments and the CAQM.
284. Consider the following statements regarding Parkachik Glacier which has been in the news
recently.
1. The Parkachik glacier is one of the largest glaciers in the Pir Panjal valley.
2. It is situated at a higher altitude than other glaciers in the Zanskar region
3. It found that three glacial lakes are likely to form around Parkachik Glacier in Ladakh due
to rapid ice melt.
How many of the above given Statements is/are correct ?
A. 1 Only B. 3 Only
C. 2 and 3 Only D. 1 and 2 Only

AMIGOS IAS 166


Explanation : D
• A new study by scientists of Wadia Institute of Himalayan Geology has found that three
glacial lakes are likely to form around Parkachik Glacier in Ladakh due to rapid ice melt.
Parkachik Glacier:
● The Parkachik glacier is one of the largest glaciers in the Suru River valley, covering an
area of 53 square km and is 14 km long.
● The Suru River valley is a part of the southern Zanskar Ranges in the western Himalayas.
● There are two main reasons for the rapid melting of the glacier.
● The first is global warming and increasing temperatures in the region.
● The second is that it is at a lower altitude than other glaciers in the Zanskar region.
285. Consider the following statements regarding Blue whales.
1. Blue whales are the largest animals ever to live on our planet.
2. They are found in all oceans.
3. They sometimes swim in small groups but are more often found alone or in pairs.
4. India does not have consists of Blue Whales
How many of the above given Statements is/are correct ?
A. Only one B. Only two
C. Only three D. All four
Explanation : B
• Recently, the carcass of a blue whale washed ashore at Meghavaram beach of
Santabommali mandal in Srikakulam district of Andhra Pradesh.
Blue whale:
● Blue whales are the largest animals ever to live on our planet.
● Whales are at the top of the food chain and have an important role in the overall health
of the marine environment.
● They are found in all oceans except the Arctic Ocean.
● They generally migrate seasonally between summer feeding grounds and winter breeding
grounds
● The average lifespan is estimated at around 80 to 90 years. Scientists can estimate the
age of whales by counting the layers of wax-like earplugs collected from deceased
animals.

286. Consider the following statements regarding International Institute for Population Sciences.
1. It was established in July 1956 under the joint sponsorship of Sir Dorabji Tata Trust, the
Government of India and the United Nations
2. The Institute is under the administrative control of the Ministry of Statistics and
Programme implementation, Government of India.
Which of the above given Statements is/are incorrect ?

AMIGOS IAS 167


A. 1 only B. 2 Only
C. Both 1 and 2 D. Neither 1 nor 2
Explanation : B
Recently, the Central government has suspended the Director of the International
Institute for Population Sciences (IIPS) citing irregularity in recruitment.
International Institute for Population Sciences:
● It was formerly known as the Demographic Training and Research Centre (DTRC) till 1970.
● It was established in July 1956 under the joint sponsorship of Sir Dorabji Tata Trust, the
Government of India and the United Nations.
● It serves as a regional centre for Training and Research in Population Studies for the ESCAP
region.
● The Institute was re-designated to its present title in 1985 to facilitate the expansion of
its academic activities.
● The Institute is under the administrative control of the Ministry of Health and Family
Welfare, Government of India.
● Institute had conducted many prime surveys like National Family Health Survey (NFHS),
District Level Household Survey (DLHS), Assessment of National Rural Health Mission
(NRHM), YOUTH in India Project etc.

287. Consider the following statements regarding National Cooperative Consumers Federation of India
(NCCF).
1. It was established on 16th October 1965 to function as the apex body of consumer
cooperatives in the country.
2. It is registered under the Societies Act, 1860.
3. NCCF functions under the Ministry of Cooperation, Government of India.
4. The management of NCCF vests in the Board of Directors
How many of the above given Statements is/are correct ?
A. Only one B. Only two
C. Only three D. All four

Explanation : B
• Hyperlocal e-commerce startup magicpin in a pact with NCCF recently started selling
tomatoes for Rs.70 per kilogram through select online platforms registered on the
government-backed ONDC.
National Cooperative Consumers Federation of India (NCCF):
● It was established on 16th October 1965 to function as the apex body of consumer
cooperatives in the country.

AMIGOS IAS 168


● It is an organization to promote consumer cooperative movement in the country, aspires
to facilitate the voluntary formation and democratic functioning of cooperatives, based
on self-reliance and mutual aid for overall economic betterment and financial autonomy.
● It is registered under the Multi-State Co-operative Societies Act, 2002.
● NCCF functions under the Ministry of Consumer Affairs, Food and Public Distribution,
Government of India.
● Headquarters: New Delhi
● The management of NCCF vests in the Board of Directors.

288. Consider the following statements regarding the Gobardhan portal.


1. It serves as a centralized repository for assessing investment and participation in the
Biogas/CBG (Compressed Biogas) sector at a pan-India level.
2. It allows any government, cooperative or private entity operating or intending to set up
a Biogas/CBG/Bio-CNG plant in India to obtain a registration number by enrolling in the
portal.
Which of the above given Statements is/are correct ?
A. 1 only B. 2 Only
C. Both 1 and 2 D. Neither 1 nor 2
Explanation : C
Over 1,200 biogas plants have registered on the Gobardhan portal since it was launched,
according to an official release.
Gobardhan portal:
● It serves as a centralized repository for assessing investment and participation in the
Biogas/CBG (Compressed Biogas) sector at a pan-India level.
● To streamline the process of setting up CBG/Biogas plants in the country.
● It allows any government, cooperative or private entity operating or intending to set up
a Biogas/CBG/Bio-CNG plant in India to obtain a registration number by enrolling in the
portal.
● This registration number will enable them to avail a multitude of benefits and support
from the Ministries and Departments of the Government of India.

289. Consider the following statements regarding the LiFE Mission of India.
1. It was launched by the Indian Prime Minister at the 26th UN Climate Change Conference
of the Parties (COP26) in Glasgow.
2. It emboldens the spirit of the P3 model.
3. It makes the fight against climate change democratic
Which of the above given Statements is/are correct ?
A. 1 and 2 Only B. 2 and 3 Only

AMIGOS IAS 169


C. 1 and 3 Only D. 1,2 and 3
Explanation : D
The Indian Embassy in Kathmandu and the Indian Consulate General in Birgunj recently
held a series of events on the LiFE mission.
LiFE Mission:
● Mission LiFE, or Lifestyle for Environment, is an India-led global mass movement to nudge
individual and community action to protect and preserve the environment.
● It was launched by the Indian Prime Minister at the 26th UN Climate Change Conference
of the Parties (COP26) in Glasgow in November 2021.
● The program hopes to “mobilize one billion Indians as well as people in other countries
to become individuals who practice sustainable lifestyles.
● The global movement will showcase sustainable goals and climate actions taken by
countries and individuals around the world.
● It makes the fight against climate change democratic, in which everyone can contribute
with their respective capacities.
● It emboldens the spirit of the P3 model, i.e., Pro Planet People.
● It functions on the basic principles of ‘Lifestyle of the planet, for the planet and by the
planet’.

290. Consider the following statements regarding Stapled Visa that has been in the news recently.
1. A stapled visa is a stamped piece of paper that is attached permanently to a page of the
passport and can not be torn off or detached at will.
2. China has made it a practice to issue stapled visas to Indian nationals from Arunachal
Pradesh and Jammu and Kashmir.
Which of the above given Statements is/are incorrect ?
A. 1 only B. 2 Only
C. Both 1 and 2 D. Neither 1 nor 2
Explanation : A
What is a stapled visa?
● Passports, visas, and other kinds of immigration controls reiterate the idea of a nation-
state and its sovereignty which is inalienable and inviolable. A passport is the certificate
of its holder’s identity and citizenship.
● A stapled visa is an unstamped piece of paper that is attached by a pin or staples to a page
of the passport and can be torn off or detached at will.
● This is different from a regular visa that is affixed to the passport by the issuing authority
and stamped.
● China has made it a practice to issue stapled visas to Indian nationals from Arunachal
Pradesh and Jammu and Kashmir. It says the visas are valid documents, but the
Government of India has consistently refused to accept this position.

AMIGOS IAS 170


291. Consider the following statements regarding the Performance of India towards Tiger
Conservation:
1. As per 5th tiger census India’s tiger population increased to 3,682 in 2022 from 1,411 in
2006.
2. In 2022, the maximum number of tigers (785) were reported to be in Karnataka.
3. Nearly a quarter of the tigers were reportedly inside protected areas in India.
How many of the above given Statements is/are correct ?
A. Only one B. Only two
C. All three D. None
Explanation : A
● July 29 is celebrated world over as the International Tiger Day in a bid to raise awareness
on various issues surrounding tiger conservation.
Performance of India towards Tiger Conservation:
● As per the World Wildlife Fund (WWF), while countries in Southeast Asia struggled to
control population decline, India fared much better.
● According to the Wildlife Institute of India's (WII) 5th tiger census (quadrennial), India’s
tiger population increased to 3,682 in 2022 (revised from 3,167 recently), up from 1,411
in 2006.
● In 2022, the maximum number of tigers (785) were reported to be in MP, followed by
Karnataka (563), Uttarakhand (560), and Maharashtra (444).
● Nearly a quarter of the tigers were reportedly outside protected areas.
292. Consider the following statements regarding Government e-Marketplace (GeM).
1. GeM is an online platform for public procurement, launched in 2016 by the Ministry of
Commerce and Industry (MoC&I).
2. The platform is owned by Central and state governments with equal share
3. It was created with the objective to create an open and transparent procurement
platform for government buyers.

How many of the above given Statements is/are correct ?


A. Only one B. Only two
C. All three D. None
Explanation : B
While the Ministry of S&T has announced its intent to galvanise research in India through
the National Research Foundation Bill 2023, scientists say that the mandatory
procurement via GeM is a major stumbling block.
Government e-Marketplace (GeM)

AMIGOS IAS 171


● GeM is an online platform for public procurement, launched in 2016 by the Ministry of
Commerce and Industry (MoC&I), Government of India.
● It was created with the objective to create an open and transparent procurement
platform for government buyers to facilitate the online procurement of goods and
services.
● The purchases through GeM by Government users have been authorised and made
mandatory by the Ministry of Finance under the General Financial Rules, 2017.
● The platform is owned by GeM SPV (Special Purpose Vehicle) which is a 100%
Government-owned, non-profit company under the MoC&I.
● The portal features over 11,000 product categories with more than 29 lakh listed
products, as well as over 270 service categories with more than 2.5 lakh service offerings.

293. Consider the following statements regarding Graded Response Action Plan (GRAP).
1. The GRAP was first notified in January 2017 by the Ministry of Environment, Forest and
Climate Change.
2. The Commission for Air Quality Management (CAQM) has constituted a sub-committee
for the operationalization of the GRAP.
3. It is a set of emergency measures that kick in to prevent further deterioration of air quality
all over India.
Which of the above given Statements is/are correct ?
A. 1 and 2 Only B. 3 Only
C. 2 and 3 Only D. 2 Only
Explanation : A
Recently, the Commission for Air Quality Management in NCR & Adjoining Areas (CAQM)
announced revision of the Graded Response Action Plan (GRAP) which will come into
force w.e.f. 1st October, 2023 in the entire National Capital Region.
Graded Response Action Plan:
● It is a set of emergency measures that kick in to prevent further deterioration of air quality
once it reaches a certain threshold in the Delhi-NCR region.
● The GRAP was first notified in January 2017 by the Ministry of Environment, Forest and
Climate Change.
● The Commission for Air Quality Management (CAQM) has constituted a sub-committee
for the operationalization of the GRAP.
● This body includes officials from the CAQM, member secretaries of pollution control
boards of Uttar Pradesh, Delhi, Haryana, Rajasthan, the Central Pollution Control Board,
a scientist from the IMD and one from the IITM and Health Advisor.
● The sub-committee is required to meet frequently to issue orders to invoke the GRAP.
● The orders and directions of the CAQM will prevail in case of any conflict between
directions issued by the State governments and the CAQM.

AMIGOS IAS 172


294. Consider the following statements regarding Parkachik Glacier which has been in the news
recently.
1. The Parkachik glacier is one of the largest glaciers in the Pir Panjal valley.
2. It is situated at a higher altitude than other glaciers in the Zanskar region
3. It found that three glacial lakes are likely to form around Parkachik Glacier in Ladakh due
to rapid ice melt.
How many of the above given Statements is/are correct ?
A. 1 Only B. 3 Only
C. 2 and 3 Only D. 1 and 2 Only
Explanation : D
● A new study by scientists of Wadia Institute of Himalayan Geology has found that three
glacial lakes are likely to form around Parkachik Glacier in Ladakh due to rapid ice melt.

Parkachik Glacier:
● The Parkachik glacier is one of the largest glaciers in the Suru River valley, covering an
area of 53 square km and is 14 km long.
● The Suru River valley is a part of the southern Zanskar Ranges in the western Himalayas.
● There are two main reasons for the rapid melting of the glacier.
● The first is global warming and increasing temperatures in the region.
● The second is that it is at a lower altitude than other glaciers in the Zanskar region.

295. Consider the following statements regarding Blue whales.


1. Blue whales are the largest animals ever to live on our planet.
2. They are found in all oceans.
3. They sometimes swim in small groups but are more often found alone or in pairs.
4. India does not have consists of Blue Whales
How many of the above given Statements is/are correct ?
A. Only one B. Only two
C. Only three D. All four

Explanation : B
Recently, the carcass of a blue whale washed ashore at Meghavaram beach of
Santabommali mandal in Srikakulam district of Andhra Pradesh.

Blue whale:
● Blue whales are the largest animals ever to live on our planet.

AMIGOS IAS 173


● Whales are at the top of the food chain and have an important role in the overall health
of the marine environment.
● They are found in all oceans except the Arctic Ocean.
● They generally migrate seasonally between summer feeding grounds and winter breeding
grounds
● The average lifespan is estimated at around 80 to 90 years. Scientists can estimate the
age of whales by counting the layers of wax-like earplugs collected from deceased
animals.

296. Consider the following statements regarding International Institute for Population Sciences.
1. It was established in July 1956 under the joint sponsorship of Sir Dorabji Tata Trust, the
Government of India and the United Nations
2. The Institute is under the administrative control of the Ministry of Statistics and
Programme implementation, Government of India.
Which of the above given Statements is/are incorrect ?
A. 1 only B. 2 Only
C. Both 1 and 2 D. Neither 1 nor 2
Explanation : B
Recently, the Central government has suspended the Director of the International
Institute for Population Sciences (IIPS) citing irregularity in recruitment.
International Institute for Population Sciences:
● It was formerly known as the Demographic Training and Research Centre (DTRC) till 1970.
● It was established in July 1956 under the joint sponsorship of Sir Dorabji Tata Trust, the
Government of India and the United Nations.
● It serves as a regional centre for Training and Research in Population Studies for the ESCAP
region.
● The Institute was re-designated to its present title in 1985 to facilitate the expansion of
its academic activities.
● The Institute is under the administrative control of the Ministry of Health and Family
Welfare, Government of India.
● Institute had conducted many prime surveys like National Family Health Survey (NFHS),
District Level Household Survey (DLHS), Assessment of National Rural Health Mission
(NRHM), YOUTH in India Project etc.

297. Consider the following statements regarding National Cooperative Consumers Federation of India
(NCCF).
1. It was established on 16th October 1965 to function as the apex body of consumer
cooperatives in the country.

AMIGOS IAS 174


2. It is registered under the Societies Act, 1860.
3. NCCF functions under the Ministry of Cooperation, Government of India.
4. The management of NCCF vests in the Board of Directors
How many of the above given Statements is/are correct ?
A. Only one B. Only two
C. Only three D. All four
Explanation : B
Hyperlocal e-commerce startup magicpin in a pact with NCCF recently started selling
tomatoes for Rs.70 per kilogram through select online platforms registered on the
government-backed ONDC.
National Cooperative Consumers Federation of India (NCCF):
● It was established on 16th October 1965 to function as the apex body of consumer
cooperatives in the country.
● It is an organization to promote consumer cooperative movement in the country, aspires
to facilitate the voluntary formation and democratic functioning of cooperatives, based
on self-reliance and mutual aid for overall economic betterment and financial autonomy.
● It is registered under the Multi-State Co-operative Societies Act, 2002.
● NCCF functions under the Ministry of Consumer Affairs, Food and Public Distribution,
Government of India.
● Headquarters: New Delhi
● The management of NCCF vests in the Board of Directors.

298. Consider the following statements regarding the Gobardhan portal .


1. It serves as a centralized repository for assessing investment and participation in the
Biogas/CBG (Compressed Biogas) sector at a pan-India level.
2. It allows any government, cooperative or private entity operating or intending to set up
a Biogas/CBG/Bio-CNG plant in India to obtain a registration number by enrolling in the
portal.
Which of the above given Statements is/are correct ?
A. 1 only B. 2 Only
C. Both 1 and 2 D. Neither 1 nor 2
Explanation : C
Over 1,200 biogas plants have registered on the Gobardhan portal since it was launched,
according to an official release.

Gobardhan portal:

AMIGOS IAS 175


● It serves as a centralized repository for assessing investment and participation in the
Biogas/CBG (Compressed Biogas) sector at a pan-India level.
● To streamline the process of setting up CBG/Biogas plants in the country.
● It allows any government, cooperative or private entity operating or intending to set up
a Biogas/CBG/Bio-CNG plant in India to obtain a registration number by enrolling in the
portal.
● This registration number will enable them to avail a multitude of benefits and support
from the Ministries and Departments of the Government of India.

299. Consider the following statements regarding the LiFE Mission of India.
1. It was launched by the Indian Prime Minister at the 26th UN Climate Change Conference
of the Parties (COP26) in Glasgow.
2. It emboldens the spirit of the P3 model.
3. It makes the fight against climate change democratic
Which of the above given Statements is/are correct ?
A. 1 and 2 Only B. 2 and 3 Only
C. 1 and 3 Only D. 1,2 and 3
Explanation : D
The Indian Embassy in Kathmandu and the Indian Consulate General in Birgunj recently
held a series of events on the LiFE mission.
LiFE Mission:
● Mission LiFE, or Lifestyle for Environment, is an India-led global mass movement to nudge
individual and community action to protect and preserve the environment.
● It was launched by the Indian Prime Minister at the 26th UN Climate Change Conference
of the Parties (COP26) in Glasgow in November 2021.
● The program hopes to “mobilize one billion Indians as well as people in other countries
to become individuals who practice sustainable lifestyles.
● The global movement will showcase sustainable goals and climate actions taken by
countries and individuals around the world.
● It makes the fight against climate change democratic, in which everyone can contribute
with their respective capacities.
● It emboldens the spirit of the P3 model, i.e., Pro Planet People.
● It functions on the basic principles of ‘Lifestyle of the planet, for the planet and by the
planet’.

300. Consider the following statements regarding Stapled Visa that has been in the news recently.
1. A stapled visa is a stamped piece of paper that is attached permanently to a page of the
passport and can not be torn off or detached at will.

AMIGOS IAS 176


2. China has made it a practice to issue stapled visas to Indian nationals from Arunachal
Pradesh and Jammu and Kashmir.
Which of the above given Statements is/are incorrect ?
A. 1 only B. 2 Only
C. Both 1 and 2 D. Neither 1 nor 2
Explanation : A
What is a stapled visa?
● Passports, visas, and other kinds of immigration controls reiterate the idea of a nation-
state and its sovereignty which is inalienable and inviolable. A passport is the certificate
of its holder’s identity and citizenship.
● A stapled visa is an unstamped piece of paper that is attached by a pin or staples to a page
of the passport and can be torn off or detached at will.
● This is different from a regular visa that is affixed to the passport by the issuing authority
and stamped.
● China has made it a practice to issue stapled visas to Indian nationals from Arunachal
Pradesh and Jammu and Kashmir. It says the visas are valid documents, but the
Government of India has consistently refused to accept this position.

301. In which of the following ways does the US Federal Reserve Interest Hike can Impact Indian
Economy ?
1. The difference between interest rates in the US and India shrinks, which affects the
currency trade.
2. Foreign investors will be tempted to withdraw from the Indian market and invest in US
assets
3. This makes the rupee stronger, and it prompts RBI for a rate reduction in India.
4. Depletes the domestic Forex reserve.
How many of the above given Statements is/are correct ?
A. Only one B. Only two
C. Only three D. All four
Explanation : C

Possible impacts of US Federal Reserve Interest Hike on the Indian Economy:


● After a rate hike by the US Fed, the difference between interest rates in US and India
shrinks, which affects the currency trade negatively.
● Foreign investors will be tempted to withdraw from the Indian market and invest in US
assets, as the Dollar and the US Treasury yield become more attractive in the US and the
Indian market begins to see capital outflow.
● Thus, an interest rate hike in the US increases the relative returns on dollar investments,
leading the US currency to strengthen.

AMIGOS IAS 177


● This makes the rupee weaker, and it prompts RBI for a rate hike in India.
● So, when the US Fed increases rates, RBI also has to increase interest rates here in India
so that the outflows of funds from the FIIs (Foreign Institutional Investors) can be
curtailed to safeguard the rupee.
● If the rupee falls significantly, the RBI may be forced to sell some dollars to help shore up
the domestic currency. This depletes the domestic Forex reserve.

302. Consider the following statements regarding the Gulf Stream.


1. It is a swift and cold ocean current that flows along the western coast of south America
2. It is primarily formed by the convergence of warm waters from the Caribbean Sea and the
Gulf of Mexico
3. It follows a north-westward path across the western South Atlantic Ocean.
Which of the above given Statements is/are correct ?
A. 1 and 2 Only B. 2 and 3 Only
C. 1 and 3 Only D. 1,2 and 3

Explanation : C
A recent study suggests the Gulf Stream system could collapse as soon as 2025.
Gulf Stream
● The Gulf Stream is a swift and warm ocean current that flows along the eastern coast of
North America and crosses the Atlantic Ocean towards Europe
● It originates in the Gulf of Mexico and is primarily formed by the convergence of warm
waters from the Caribbean Sea and the Gulf of Mexico. It then travels northward along
the eastern coast of the United States.
● It follows a north-eastward path across the western North Atlantic Ocean.
● The current carries warm water from the tropics (around 25 to 28°C or 77 to 82°F) to
higher latitudes.
303. Consider the following statements regarding the Maitree Super Thermal Power Project (STPP).
1. It is a 1,320MW coal-fired power station under construction in Bihar
2. It is being financed through a £1.3bn ($1.6bn) loan from the Export-Import (EXIM) Bank
of India.
3. The construction of the project commenced in April 2017.
Which of the above given Statements is/are correct ?
A. 1 and 2 Only B. 3 Only
C. 2 and 3 Only D. 2 Only
Explanation : C

AMIGOS IAS 178


State-owned engineering firm Bharat Heavy Electricals recently announced the
synchronisation of 660-MW unit-2 of the 1,320-MW Maitree Super Thermal Power
Project (STPP) with the electricity grid in Bangladesh.
Maitree Super Thermal Power Project (STPP):
● It is a 1,320MW coal-fired power station under construction in Rampal, Bangladesh.
● It is being developed on an 1,834-acre site on the bank of Passur River, approximately
14km away from the Sundarbans.
● It is being developed by Bangladesh India Friendship Power Company (BIFPCL), a 50:50
joint venture between India’s state-run National Thermal Power Corporation (NTPC) and
Bangladesh Power Development Board (BPDB).
● The construction of the project commenced in April 2017.
● It will be one of the biggest coal-fired power plants in Bangladesh, along with the Payra
Power Plant in Pataukhali, which commenced test production in January 2020.
● It is being financed through a £1.3bn ($1.6bn) loan from the Export-Import (EXIM) Bank
of India.
● BIFPCL entered into a loan agreement with the EXIM Bank of India in March 2017.

304. Consider the following statements regarding Conjunctivitis.


1. It can be caused by viruses, bacteria or by allergies
2. All types of conjunctivitis are highly contagious
3. Indirectly, it can spread via shared personal items like towels, makeup, pillows or contact
lenses.
Which of the above given Statements is/are correct ?
A. 1 and 2 Only B. 2 and 3 Only
C. 1 and 3 Only D. 1,2 and 3
Explanation : C
Amid heavy rainfall in Delhi and nearby areas over the past few weeks, multiple cases of
conjunctivitis are being reported in the National Capital Region.
Conjunctivitis:
● Conjunctivitis, commonly known as Pink Eye, is an infection or inflammation of the
transparent membrane that covers the eyelid and eyeball. This membrane is called the
conjunctiva.
Causative Agent:
● It can be caused by viruses, bacteria or by allergies.
● Both bacterial and viral conjunctivitis are highly contagious, while allergic conjunctivitis is
not.
Transmission:
● It usually occurs through direct or indirect contact.

AMIGOS IAS 179


● Direct transmission happens through droplets from the cough or sneeze of an infected
person or through hand-to-eye contact.
● Indirectly, it can spread via shared personal items like towels, makeup, pillows or contact
lenses.

305. Consider the following statements regarding Hematene.


1. It is extracted from naturally occurring hematite, the mineral form of iron(III) oxide.
2. It is not ferromagnetic unlike a common magnet.
3. It is not capable of withstanding and acting as shield from high laser intensities
4. It measures 3 atoms thick and has a more efficient photocatalysis
How many of the above given Statements is/are correct ?
A. Only one B. Only two
C. Only three D. All four
Explanation : B
● Researchers have come up with a new and highly efficient optical limiter using a novel 2D
material, ‘hematene’.

Hematene:
● It is extracted from naturally occurring hematite, the mineral form of iron(III) oxide, using
a combination of sonication, centrifugation and vacuum-assisted filtration.
● It measures 3 atoms thick and has a more efficient photocatalysis.
● It is ferromagnetic (the mechanism by which certain materials like iron form permanent
magnets, or are attracted to magnets) like a common magnet.
● It is capable of withstanding and acting as shield from high laser intensities

Uses of Hemantane
● Nanoflakes of a material called hematene extracted from iron ore have been found
capable of withstanding and acting as shield from high laser intensities.
● Hence it could be used to make devices called optical limiters that can protect sensitive
optical equipment from light-induced damage.
● Radiation from laser sources is highly concentrated and powerful and can be detrimental
to sensitive equipment such as sensors, detectors, and other optical devices.
● When the input intensity increases optical limiters control the amount of light that passes
through, thereby preventing damage to the optical component.
● These devices are often useful in laser technologies, military, telecommunications,
aircrafts, and scientific research in several ways.

306. Consider the following statements regarding Seagrass.

AMIGOS IAS 180


1. It is a non- flowering plant that grows submerged in deep marine waters.
2. Seagrasses do not have roots, stems, and leaves and produce flowers and seeds.
3. Seagrass also photosynthesizes and manufactures their own food and releases oxygen.
4. They are found on all continents except the Arctic region.
How many of the above given Statements is/are incorrect ?
A. Only one B. Only two
C. Only three D. All four
Explanation : C
● Citizen divers restore seagrass in just off the coast of Kiel in northern Germany, Baltic sea
to fight climate change.
Seagrass:
● It is a flowering plant that grows submerged in shallow marine waters like bays and
lagoons.
● They are so-named because most species have long green, grass-like leaves.
● Seagrasses have roots, stems, and leaves and produce flowers and seeds.
● Like terrestrial plants, seagrass also photosynthesizes and manufactures their own food
and releases oxygen.
● They evolved around 100 million years ago, and there are approximately 72 different
seagrass species that belong to four major groups.
● They are found on all continents except Antarctica.

307. Consider the following statements regarding Scrub typhus.


1. It is a life-threatening infection caused by a Virus
2. It spreads to people through bites of infected chiggers
3. Scrub typhus should be treated with the antibiotic doxycycline
4. It will spread from person to person
How many of the above given Statements is/are correct ?
A. Only one B. Only two
C. Only three D. All four
Explanation : B
● Recently, the Kerala Health department has issued an alert against scrub typhus as fresh
cases of the disease have been reported from Kalath and District Court wards in
Alappuzha municipality.
Scrub typhus:
● It is a life-threatening infection caused by Orientia tsutsugamushi bacteria which is a
major public health threat in South and Southeast Asia.
● It spreads to people through bites of infected chiggers (larval mites).

AMIGOS IAS 181


● Symptoms: The most common symptoms of scrub typhus include fever, headache, body
aches, and sometimes rash.
● Treatment: Scrub typhus should be treated with the antibiotic doxycycline. Doxycycline
can be used in persons of any age.
● There is no vaccine available for this disease.
● It will not spread from person to person.
● India is one of the hotspots with at least 25% of the disease burden.
308. Consider the following statements regarding the Meri Maati Mera Desh campaign.
1. Under this campaign soil collected from different parts of the country and will be used to
develop a garden in Delhi.
2. For this purpose, veers include freedom fighters, defence personnel, personnel of the
CAPF and State Police, who laid down their lives in the line of duty.
Which of the above given Statements is /are correct ?
A. 1 only B. 2 Only
C. Both 1 and 2 D. Neither 1 nor 2
Explanation : C
● The Union government has launched the ‘Meri Maati Mera Desh’ campaign, envisaged
as a culminating event of the ‘Azadi Ka Amrit Mahotsav’ celebration of 75 years of Indian
Independence, in which soil collected from different parts of the country in August will be
used to develop a garden along the Kartavya Path in Delhi.
● Events have been planned at the panchayat, village, block, urban local body, and State
and national levels, respectively. The panchayat-level programmes will be organised
between August 9 and 15
● For this purpose, veers (bravehearts) include freedom fighters, defence personnel,
personnel of the Central Armed Police Forces (CAPF), and State Police, who laid down
their lives in the line of duty.
● The five-point agenda includes the installation of a shilaphalakam (memorial plaque), as
per specifications, bearing the “names of those who have made the supreme sacrifice”.

309. Consider the following statements regarding Silvopasture systems.


1. It is an ancient and proven practice that harmoniously integrates trees, forage and
livestock on the same land.
2. The trees on silvopasture lands act as natural carbon sinks, sequestering significantly five-
10 times more carbon
3. Silvopasture plots contribute to nutrient cycling, improved soil stability and quality.
Which of the above given Statements is/are correct ?
A. 1 and 2 Only B. 2 Only
C. 1 Only D. 1,2 and 3
Explanation : D

AMIGOS IAS 182


• In light of the global deterioration of natural resources and forests, silvopasture systems
offer a relevant solution to deforestation trends.
Silvopasture system:
● It is an ancient and proven practice that harmoniously integrates trees, forage and
livestock on the same land.
Advantages of Silvopasture system
● This system buffer against temperature and wind extremes, providing a favourable living
environment for livestock.
● The trees on silvopasture lands act as natural carbon sinks, sequestering significantly five-
10 times more carbon then pastures without trees, all while maintaining or enhancing
productivity.
● The extensive root systems of trees within silvopasture plots contribute to nutrient
cycling, improved soil stability and quality, while effectively combating erosion.
● Also, this system combine trees and livestock on the same land, silvopasture can play a
vital role in reversing the negative trend of deforestation for pasture land.

310. Consider the following statements regarding UK-India Young Professionals Scheme.
1. It was conceived as part of an India-U.K. Migration and Mobility MoU
2. This scheme makes India the first visa-national country to benefit from the scheme.
3. A person must not be selected in the India Young Professionals Scheme, before they can
apply for a Young Professionals Scheme visa.
Which of the above given Statements is/are correct ?
A. 1 and 2 Only B. 2 and 3 Only
C. 1 and 3 Only D. 1,2 and 3
Explanation : A
● The British Government recently announced the opening of the second ballot of the
Young Professional Scheme for Indian citizens.
Young Professionals Scheme:
● It was conceived as part of an India-U.K. Migration and Mobility MoU signed in May 2021
and was announced in November at the G20 summit in Bali.
● It was formally launched in February 2023.
● It will permit up to 3,000 of their degree-holding citizens aged between 18 and 30 to live
and work in each other’s countries for two years.
● This scheme makes India the first visa-national country to benefit from the scheme.
● A person must be selected in the India Young Professionals Scheme ballot before they can
apply for a Young Professionals Scheme visa.

311. Consider the following statements regarding the International Day for the Conservation of the
Mangrove Ecosystem 2023.

AMIGOS IAS 183


1. This day was adopted by the General Conference of the UNEP in 2010
2. The purpose of this day is to raise awareness of the importance of mangrove ecosystems.
3. It is celebrated every year on July 26
How many of the above given Statements is/are correct ?
A. 1 and 2 Only B. 2 and 3 Only
C. 1 and 3 Only D. 1,2 and 3
Explanation : B
International Day for the Conservation of the Mangrove Ecosystem:
● It is celebrated every year on July 26.
● This International Day was adopted by the General Conference of the UN Educational,
Scientific and Cultural Organization (UNESCO) in 2015.
● Its purpose is to raise awareness of the importance of mangrove ecosystems as “a unique,
special and vulnerable ecosystem” and to promote solutions for their sustainable
management, conservation and uses.
312. Consider the following statements regarding the Krishnaraja Sagar (KRS) Dam.
1. It is located below the confluence of river Kaveri with in the state of Karnataka
2. It is the main source of drinking water for Mysore, Mandya and Bengaluru city.
3. The water released from this dam flows into the state of Tamil Nadu and is stored in the
Mettur dam.
Which of the above given Statements is/are correct ?
A. 1 and 2 Only B. 2 and 3 Only
C. 3 Only D. 1,2 and 3

Explanation : D
● The water level in the Krishnaraja Sagar (KRS) across the Cauvery recently breached the
100-ft mark as against its capacity of 124.80 feet owing to heavy rain in the catchment
area of the river.

Krishnaraja Sagar (KRS) Dam:


● It is a type of gravity dam.
● It is located below the confluence of river Kaveri with its tributaries, Hemavati and
Lakshmana Tirtha, in the district of Mandya in Karnataka.
● The water from the Dam is used for irrigation in Mysore and Mandya and is the main
source of drinking water for Mysore, Mandya and Bengaluru city.
● It also ensures power supply to the Shivanasamudra hydroelectric power station.
● The water released from this dam flows into the state of Tamil Nadu and is stored in the
Mettur dam in the Salem district.

AMIGOS IAS 184


313. Consider the following statements regarding Pradhan Mantri Jan Arogya Yojana.
1. The households included in this Scheme are based on the deprivation and occupational
criteria of Socio-Economic Caste Census 2011
2. It provides a cover of Rs. 5 lakhs per family per year for secondary and tertiary care
hospitalization
3. There are restrictions on the family size, age or gender in this scheme
Which of the above given Statements is/are correct ?
A. 1 and 2 Only B. 2 and 3 Only
C. 1 and 3 Only D. 1,2 and 3
Explanation : A
● Over a third of hospitals empanelled under Ayushman Bharat are inactive, official data
showed, in a worrying trend for the world’s largest health insurance scheme aiming for
universal health coverage.
Pradhan Mantri Jan Arogya Yojana:
● It is the world’s largest health insurance/ assurance scheme fully financed by the
government.
● The households included are based on the deprivation and occupational criteria of Socio-
Economic Caste Census 2011 (SECC 2011) for rural and urban areas respectively.
● It provides a cover of Rs. 5 lakhs per family per year for secondary and tertiary care
hospitalization across public and private empanelled hospitals in India.
● It provides cashless access to health care services for the beneficiary at the point of
service, that is, the hospital.
● There is no restriction on the family size, age or gender.
314. Which of the following statements is/are correct regarding the events that happened in the
Miocene epoch.
1. India collided with Asia causing a massive upthrust of the Asian continent
2. The earth began to cool once more
3. Grasslands underwent a major expansion
4. Accelerated evolution among marine plankton and molluscs
Choose the correct answer from the below given codes:
A. 1,2 and 3 Only B. 2,3 and 4 Only
C. 1,3 and 4 Only D. 1,2,3 and 4
Explanation : D
● Recently, a research group in Japan has uncovered an exceptionally well-preserved fossil
forest which belongs to the late Miocene epoch.
Miocene Epoch

AMIGOS IAS 185


● The Miocene was a long-lasting epoch in which the earth's climate rebounded from the
cooling of the Oligocene and there was a marked increase in both global temperatures
and the total number of mammal species.
What changes occurred during this period?
1. India collided with Asia, causing a massive upthrust of the Asian continent.
2. As the Miocene progressed, the earth began to cool once more; the climate became more
arid and mammalian diversity began to decrease again.
3. Grasslands underwent a major expansion in the early Miocene and mammalian
herbivores.
4. In the oceans, the Miocene was a time of changing circulation patterns, probably due to
global cooling
5. The Miocene was a time of accelerated evolution among marine plankton and molluscs,
many groups showing increases in biological diversity.
6. The drop in sea levels during the Miocene associated with the growth of polar ice caps
led to the first exposure of what is now Florida in the Miocene.
315. Recently, the UNESCO’s Asia Pacific Cultural Heritage award has been received by which of the
following entities of India?
A. Chhatrapati Shivaji Terminus
B. Byculla Railway station
C. Howrah Junction
D. Sealdah railway station
Explanation : B
● Recently, the heritage Byculla Railway station which has been restored to its original glory,
received the UNESCO award.

UNESCO’s Asia Pacific Cultural Heritage award


● The UNESCO Asia-Pacific Awards for Cultural Heritage Conservation is supported by a
partnership between UNESCO and Ng Teng Fong Charitable Foundation since 2021.
● UNESCO introduced the new category, ‘Special Recognition for Sustainable
Development’, in 2020, together with an updated set of Awards Criteria.
● It is to acknowledge the role and contribution of cultural heritage to sustainable
development within the broader framework of the UN 2030 Agenda.
● Since 2000, UNESCO Asia-Pacific Awards for Cultural Heritage Conservation have been
recognizing the achievement of the private sector and public-private initiatives in
successfully conserving or restoring structures, places and properties of heritage value in
the region.

316. Consider the following statements regarding the No confidence motion :


1. A motion of confidence or no confidence is mentioned in the Indian Constitution under
Article 75.

AMIGOS IAS 186


2. If individuals or parties abstain from voting during the No confidence motion they are not
taken into account for counting
3. Any Lok Sabha MP, who can garner the support of 50 colleagues, can, at any point of time,
introduce a motion of no-confidence against the Council of Ministers.
Which of the above given Statements is/are incorrect ?
A. 2 and 3 Only B. 1,2 and 3
C. 3 Only D. 1 only
Explanation : C
● A motion of confidence or no confidence is not mentioned in the Indian Constitution. The
Lok Sabha is the Council of Ministers' sole source of accountability, according to Article
75.
● For testing this collective responsibility, the rules of Lok Sabha provide a particular
mechanism – a motion of no-confidence. Any Lok Sabha MP, who can garner the support
of 50 colleagues, can, at any point of time, introduce a motion of no-confidence against
the Council of Ministers.
● A no-confidence motion needs a majority vote to pass the House. If individuals or parties
abstain from voting, those numbers will be removed from the overall strength of the
House and then the majority will be taken into account

317. Consider the following statements with reference to Full-reserve banking.


1. It refers to a system in which banks are obliged to hold a small percentage of the client's
deposit in its reserve.
2. Under this system, banks are not allowed to use customers’ deposits to make loans or
investments.
3. With 100% reserve backing, there is no risk of a bank run

Which of the above given Statements is/are correct ?


A. 1 and 2 Only B. 2 and 3 Only
C. 1 and 3 Only D. 1,2 and 3
Explanation : B
● Full-reserve banking, also known as 100% reserve banking, is a banking system in which
banks are required to hold 100% of their customers’ demand deposits in reserve.
● Under this system, banks are not allowed to use customers’ deposits to make loans or
investments.
● Instead, they act solely as custodians, holding the deposits securely in their vaults and
charging a fee for this service.
● With 100% reserve backing, there is no risk of a bank run since the bank will always have
enough physical cash to meet all withdrawal demands.

AMIGOS IAS 187


318. Consider the following statements regarding Biological Diversity (Amendment) Bill, 2021.
1. The amendment aims to simplify patent application processes for traditional Indian
medicine practitioners.
2. It also aims at easing compliance burden, boosting investment and simplifying patent
application processes
3. The Bill sought to exempt registered AYUSH medical practitioners from giving prior
intimation to State biodiversity boards to access biological resources for certain purposes

Which of the above given Statements is/are correct ?


A. 1 and 2 Only B. 2 and 3 Only
C. 1 and 3 Only D. 1, 2 and 3
Explanation : D
● The Bill sought to exempt registered AYUSH medical practitioners and people accessing
codified traditional knowledge, among others, from giving prior intimation to State
biodiversity boards to access biological resources for certain purposes.
● The Biological Diversity (Amendment) Bill, 2021 seeks to exempt registered AYUSH
medical practitioners from intimating biodiversity boards before accessing biological
resources
● It also aims at easing compliance burden, boosting investment and simplifying patent
application processes
● Environmental organisations have raised concerns about potential for “bio piracy” they
say amendments favour industry and there is lack of clarity on benefit sharing with local
communities

319. Consider the following statements regarding Fluorochemicals.


1. Currently all fluorochemicals are generated from toxic and corrosive HF gas in a highly
energy-intensive process.
2. HF is produced by reacting Fluorspar with sulfuric acid.
3. Fluorochemicals are also used in the production of lithium-ion batteries.
Which of the above given Statements is/are correct ?
A. 1 and 2 Only B. 2 and 3 Only
C. 1 and 3 Only D. 1,2 and 3

Explanation : D
● Scientists have devised a new safer production method for fluorochemicals without use
of hazardous Hydrogen Fluoride (HF) gas.
● New method takes inspiration from natural Biomineralization process which forms teeth
and bones.

AMIGOS IAS 188


● In this method, fluorochemicals are made directly from Fluorspar, bypassing production
of HF.
● Fluorochemicals have a wide range of industrial applications including polymers,
agrochemicals, pharmaceuticals, and lithium-ion batteries.
● Currently all fluorochemicals are generated from toxic and corrosive HF gas in a highly
energy-intensive process.
● HF is produced by reacting Fluorspar (Calcium Fluoride) with sulfuric acid.

320. Consider the following statements regarding an Industrial License.


1. Any industrial license, where commercial production has not started within an extended
period shall be treated as automatically lapsed.
2. In India, industrial licenses are issued under and are regulated by Industries (Development
and Regulation) Act (IDRA), 1951.
Which of the above given Statements is/are incorrect ?
A. 1 Only B. 2 Only
C. Both 1 and 2 D. Neither 1 nor 2
Explanation : D
● Department for Promotion of Industry and Internal Trade (DPIIT) has extended the period
of validity of industrial license from existing three years to fifteen years to promote ease
of doing business.
● Concerned administrative ministry can further grant a three years extension in case
license holder has not commenced commercial production within 15 years of issue of
license.
● Any industrial license, where commercial production has not started within extended
period (15+3 years), shall be treated as automatically lapsed.
● In India, industrial licenses are issued under and are regulated by Industries (Development
and Regulation) Act (IDRA), 1951.
321. Consider the following statements with reference to the Zero FIR.
1. A Zero FIR is a written complaint registered by any police station for an alleged offense
under another police station’s jurisdiction, as per Section 154 of the Code of Criminal
Procedure (CrPC)
2. The Justice Verma Committee recommended the provision of Zero FIR to expedite trials
and increase penalties for crimes against women, particularly after the 2012 Delhi gang
rape case.

Which of the above given Statements is/are correct ?


A. 1 Only B. 2 Only
C. Both 1 and 2 D. Neither 1 nor 2
Explanation : C

AMIGOS IAS 189


● Recently, two Zero FIRs were filed in Manipur, one for the incident of two women being
stripped and paraded, and another for the alleged abduction, rape and murder of two
Kuki-Zomi women.
What is a Zero FIR?
● A Zero FIR is a written complaint registered by any police station for an alleged offense
under another police station’s jurisdiction, as per Section 154 of the Code of Criminal
Procedure (CrPC).
● The concept of Zero FIR was established to ensure that victims do not face delays in
getting their complaints registered and that timely action can be taken after the filing of
the FIR.
● The Justice Verma Committee recommended the provision of Zero FIR to expedite trials
and increase penalties for crimes against women, particularly after the 2012 Delhi gang
rape case.
● When a police station receives a complaint about an offense that is not within its
jurisdiction, it registers a Zero FIR and transfers it to the relevant police station for further
investigation.

322. Consider the following statements regarding National Tele Mental Health Programme:
1. It was launched during the lockdown period in 2020.
2. The service is accessible through the toll-free numbers with options to choose preferred
languages
3. It aims to provide free tele-mental health services all over the country round the clock,
Which of the above given Statements is/are correct ?
A. 1 and 2 Only B. 2 and 3 Only
C. 1 and 3 Only D. 1,2 and 3
Explanation : B
● Recently, the union health ministry said the Tele-Manas helpline under the National Tele
Mental Health Programme has received over 200,000 calls since its launch in October
2022.
National Tele Mental Health Programme:
● Tele Mental Health Assistance and Networking Across States (Tele MANAS) was launched
during October 2022.
● It aims to provide free tele-mental health services all over the country round the clock,
particularly catering to people in remote or under-served areas.
● There are 42 active Tele Manas cells across 31 states and Union Territories.
● The service is accessible through the toll-free numbers with options to choose preferred
languages (20 languages included till now).
● Tele-MANAS will be organized in two tier system:
○ Tier 1: It comprises state Tele-MANAS cells which include trained counselors and mental
health specialists.

AMIGOS IAS 190


■ Tier 2: It will comprise specialists at District Mental Health Programme (DMHP)/Medical
College resources for physical consultation and/or e-Sanjeevani for audio visual
consultation.

323. Consider the following statements regarding INS Kirpan which has been in the news recently.
1. INS Kirpan is a Khukri class missile corvette that was commissioned into the Navy in 1991.
2. It performs a wide variety of roles, including anti-piracy and Humanitarian Assistance and
Disaster Relief (HADR) operations.
3. Recently, India gifted the Corvette INS Kirpan to Bangladesh to enhance that country’s
naval capabilities.
Which of the above given Statements is/are correct ?
A. 1 and 2 Only B. 2 and 3 Only
C. 2 Only D. 1, 2 and 3

Explanation : A
● Recently, India gifted the Corvette INS Kirpan to Vietnam to enhance that country’s naval
capabilities.
INS Kirpan:
● It is an indigenously-built in-service missile.
● It is a Khukri class missile corvette commissioned into the Navy on January 12, 1991.
● It has a displacement displacing capacity of close to 1,400 tonnes.
● It is capable of a speed of more than 25 knots.
● The Khukri class is equipped with Diesel Engines assembled in India.
● The ship is fitted with a medium-range gun, 30 mm close-range guns, chaff launchers, and
surface-to-surface missiles.
● It performs a wide variety of roles, including coastal and offshore patrol, coastal security,
surface warfare, anti-piracy, and Humanitarian Assistance and Disaster Relief (HADR)
operations.

324. Consider the following statements regarding Ludwigia peruviana which have been in the news
recently.
1. It is an invasive weed threatening the elephant habitats and foraging areas in Valparai,
Tamil Nadu.
2. It is native to Central and South America.
3. It grows in swamps and there is little scope to use machinery which may further destroy
the ecosystem.
Which of the above given Statements is/are correct ?
A. 1 and 2 Only B. 2 and 3 Only

AMIGOS IAS 191


C. 1 and 3 Only D. 1,2 and 3
Explanation : D
● Recently, wildlife scientists said that Ludwigia Peruviana , an invasive weed threatening
the elephant habitats and foraging areas in Valparai, Tamil Nadu.
Ludwigia Peruviana:
● It is popularly called primrose willow, Ludwigia Peruviana, is a native of Central and South
America.
● Its flower is pale yellowish in colour and the plant grows to a height of about 12 feet.
● It is an aquatic plant, which is now challenging the existence of local vegetation in various
swampy areas around the world.
● The rapid large-scale spread of the weed — which was probably introduced as an
ornamental plant for its tiny yellow flowers
● It grows faster in wetlands than other harmful weeds.
● The pre-monsoon temperature and monsoon rains help this weed grow faster.
● It has shaken the balance of these perennial foraging grounds, limiting the growth of grass
and native plants that are palatable to elephants and other animals including gaur.
● It is among the 22 priority invasive plants in Tamil Nadu.
● Unlike other invasive plants, Ludwigia poses a unique challenge as it grows in swamps and
there is little scope to use machinery which may further destroy the ecosystem.

325. Consider the following statements :


Statement-I :
Private sector lender ICICI Bank recently said the bank is ready to move to an expected
credit loss (ECL) framework for provisioning.
Statement-II :
Under this practice, a bank is required to estimate expected credit losses based on
forward-looking estimations rather than wait for credit losses to be actually incurred
before making corresponding loss provisions.
Which one of the following is correct in respect of the above statements?
A. Both Statement-I and Statement-II are correct and Statement-II is the correct explanation
for Statement-I
B. Both Statement-I and Statement-II are correct and Statement-II is not the correct
explanation for Statement-I
C. Statement-I is correct but Statement-II is incorrect
D. Statement-I is incorrect but Statement-II is correct

AMIGOS IAS 192


Explanation: A
What is a loan-loss provision?
● The RBI defines a loan loss provision as an expense that banks set aside for defaulted
loans.
● Banks set aside a portion of the expected loan repayments from all loans in their portfolio
to cover the losses either completely or partially.
● In the event of a loss, instead of taking a loss in its cash flows, the bank can use its loan
loss reserves to cover the loss.
● The level of loan loss provision is determined based on the level expected to protect the
safety and soundness of the bank.
Expected Credit Loss (ECL) regime?
● Under this practice, a bank is required to estimate expected credit losses based on
forward-looking estimations rather than wait for credit losses to be actually incurred
before making corresponding loss provisions.
● As per the proposed framework, banks will need to classify financial assets (primarily
loans) as Stage 1, 2, or 3, depending on their credit risk profile, with Stage 2 and 3 loans
having higher provisions based on the historical credit loss patterns observed by banks.
● Thus, through ECL, banks can estimate the forward-looking probability of default for each
loan, and then by multiplying that probability by the likely loss given default, the bank
gets the percentage loss that is expected to occur if the borrower defaults.

326. Consider the following statements regarding the Global report on Food crises (GRFC) 2023.
1. It is an annual report to achieve a consensus-based assessment of acute food insecurity
in the world
2. It is released recently by the Food and Agriculture Organisation (FAO)
3. The report reveals that the number of people suffering from acute food insecurity has
decreased
How many of the above given Statements is/are correct ?
A. 1 and 2 Only B. 2 and 3 Only
C. 1 and 3 Only D. 1,2 and 3
Explanation : C
● The Global Report on Food Crises (GRFC) for 2023 reveals that the number of people
suffering from acute food insecurity has increased, with over 258 million people across
58 countries and territories facing acute hunger.
● The Global Report on Food Crisis is an annual report to achieve a consensus-based
assessment of acute food insecurity in the world
● It is released by the Food Security Information Network (FSIN) and Global Network
against Food Crises (GNAFC)

What is FSIN?

AMIGOS IAS 193


● A global initiative co-sponsored by Food and Agriculture Organization (FAO), World Food
Programme (WFP), and International Food Policy Research Institute (IFPRI) to strengthen
food and nutrition security information systems for producing reliable and accurate data
to guide analysis and decision-making.

What is GNAFC?
● An alliance of humanitarian and development actors founded by the European Union,
FAO, and WFP in 2016 to prevent, prepare for and respond to food crises and support the
Sustainable Development Goal to End Hunger (SDG 2)

327. According to the recent announcement by the Union Minister of State (Independent Charge)
Science & Technology, which of the following States is going to pioneer India's first Cannabis
Medicine Project ?
A. Karnataka B. Jammu and Kashmir
C. West Bengal D. Uttarakhand

Explanation : B
● Recently, Union Minister of State (Independent Charge) Science & Technology said that
Jammu is going to pioneer India's first Cannabis Medicine Project.

Cannabis Medicine Project:


● ‘Cannabis Research Project’ of CSIR-IIIM Jammu is a first of its kind in India initiated under
the leadership of Science & Technology Ministry in Private Public Partnership with a
Canadian firm, which has a great potential to put substance of abuse for the good of
mankind especially for patients suffering from neuropathies, cancer and epilepsy,
malignancies.
● This project of CSIR-IIIM is also important from the perspective of Atma- Nirbhar Bharat
as it will be able to produce export quality drugs meant for different kinds of
neuropathies, diabetic pains etc.
● This project will explore the therapeutic properties of Cannabis, a plant which is otherwise
prohibited and known for abuse.

328. Consider the following statements regarding the Software-Defined Radio Tactical (SDR-Tac).
1. It is built indigenously by the Navy’s Weapons and Electronics Systems Engineering
Establishment
2. It is intended to serve ship-to-ship, ship-to-shore, and ship-to-air voice-data
communication for network-centric operations.
3. The indigenous SDR-Tac provides real-time voice, data and video information.
Which of the above given Statements is/are correct ?
A. 1 and 2 Only B. 2 and 3 Only

AMIGOS IAS 194


C. 1 and 3 Only D. 1,2 and 3

Explanation : D
● Recently, the Indian Navy has begun equipping its warships with 'Made in India' Software-
Defined Radio.
Software-Defined Radio Tactical (SDR-Tac):
● Equipped with several multimedia capabilities, the indigenous SDR-Tac provides real-time
voice, data and video information.
● It is built indigenously by the Navy’s Weapons and Electronics Systems Engineering
Establishment (WESEE).
What does SDR-Tac do?
● SDR-Tac is a four-channel multi-mode, multi-band, 19-inch rack-mountable, ship-borne
software-defined radio system.
● It is intended to serve ship-to-ship, ship-to-shore, and ship-to-air voice-data
communication for network-centric operations.
● It supports the simultaneous operation of all four channels covering V/UHF- and L-Band.
● This SDR system houses multiple types of waveforms for narrow-band and wide-band
applications.

329. Consider the following statements regarding India Climate Energy Dashboard (ICED) 3.0.
1. It is India's one-stop platform for real-time data on the energy sector, and related
economic datasets based on government published sources.
2. It was developed by NITI Aayog in collaboration with the energy and climate think-tank
Vasudha Foundation.
Which of the above given Statements is/are correct ?
A. 1 Only
B. 2 Only
C. Both 1 and 2
D. Neither 1 nor 2
Explanation : C
India Climate Energy Dashboard (ICED) 3.0:
● It is the country’s one-stop platform for near real-time data on the energy sector, climate,
and related economic datasets based on government published sources.
● It was developed by NITI Aayog in collaboration with the energy and climate think-tank
Vasudha Foundation.
● Developed as a user-friendly platform, ICED 3.0 enables users to freely access and
analyses datasets using an analytical engine.

AMIGOS IAS 195


● It will facilitate insights and enhance understanding about the energy and climate sectors
while identifying the key challenges.

330. Consider the following statements :


Statement-I : Researchers have successfully developed a technology called hygroelectricity.
Statement-II : Hygroelectricity is the generation of electricity from the Temperature difference
of two water bodies .
Which one of the following is correct in respect of the above statements?
A. Both Statement-I and Statement-II are correct and Statement-II is the correct explanation
for Statement-I
B. Both Statement-I and Statement-II are correct and Statement-II is not the correct
explanation for Statement-I
C. Statement-I is correct but Statement-II is incorrect
D. Statement-I is incorrect but Statement-II is correct
Explanation : C
Hygroelectricity:
● Hygroelectricity is the generation of electricity from the humidity of the air.
● It is a type of renewable energy that has the potential to be a major source of power in
the future.

How it works?
● The key to harvesting electricity from humid air lies in a tiny device comprising two
electrodes and a thin layer of material filled with nanopores.
● These nanopores, each less than 100 nanometres in diameter, allow water molecules
from the air to pass through the device.
● As these molecules move from an upper chamber to a lower chamber, they interact with
the edges of the nanopores, leading to a buildup of electric charge imbalances between
the chambers.
● This process effectively transforms the device into a miniature battery, generating
continuous electricity.

331. Consider the following statements with reference to the Zero FIR.
1. A Zero FIR is a written complaint registered by any police station for an alleged offense
under another police station’s jurisdiction, as per Section 154 of the Code of Criminal
Procedure (CrPC)
2. The Justice Verma Committee recommended the provision of Zero FIR to expedite trials
and increase penalties for crimes against women, particularly after the 2012 Delhi gang
rape case.
Which of the above given Statements is/are correct ?

AMIGOS IAS 196


A. 1 Only B. 2 Only
C. Both 1 and 2 D. Neither 1 nor 2
Explanation : C
● Recently, two Zero FIRs were filed in Manipur, one for the incident of two women being
stripped and paraded, and another for the alleged abduction, rape and murder of two
Kuki-Zomi women.
What is a Zero FIR?
● A Zero FIR is a written complaint registered by any police station for an alleged offense
under another police station’s jurisdiction, as per Section 154 of the Code of Criminal
Procedure (CrPC).
● The concept of Zero FIR was established to ensure that victims do not face delays in
getting their complaints registered and that timely action can be taken after the filing of
the FIR.
● The Justice Verma Committee recommended the provision of Zero FIR to expedite trials
and increase penalties for crimes against women, particularly after the 2012 Delhi gang
rape case.
● When a police station receives a complaint about an offense that is not within its
jurisdiction, it registers a Zero FIR and transfers it to the relevant police station for further
investigation.
332. Consider the following statements regarding National Tele Mental Health Programme:
1. It was launched during the lockdown period in 2020.
2. The service is accessible through the toll-free numbers with options to choose preferred
languages
3. It aims to provide free tele-mental health services all over the country round the clock,
Which of the above given Statements is/are correct ?
A. 1 and 2 Only B. 2 and 3 Only
C. 1 and 3 Only D. 1,2 and 3
Explanation : B
● Recently, the union health ministry said the Tele-Manas helpline under the National Tele
Mental Health Programme has received over 200,000 calls since its launch in October
2022.
National Tele Mental Health Programme:
● Tele Mental Health Assistance and Networking Across States (Tele MANAS) was launched
during October 2022.
● It aims to provide free tele-mental health services all over the country round the clock,
particularly catering to people in remote or under-served areas.
● There are 42 active Tele Manas cells across 31 states and Union Territories.
● The service is accessible through the toll-free numbers with options to choose preferred
languages (20 languages included till now).
● Tele-MANAS will be organized in two tier system:

AMIGOS IAS 197


○ Tier 1: It comprises state Tele-MANAS cells which include trained counselors and mental
health specialists.
■ Tier 2: It will comprise specialists at District Mental Health Programme (DMHP)/Medical
College resources for physical consultation and/or e-Sanjeevani for audio visual
consultation.

333. Consider the following statements regarding INS Kirpan which has been in the news recently.
1. INS Kirpan is a Khukri class missile corvette that was commissioned into the Navy in 1991.
2. It performs a wide variety of roles, including anti-piracy and Humanitarian Assistance and
Disaster Relief (HADR) operations.
3. Recently, India gifted the Corvette INS Kirpan to Bangladesh to enhance that country’s
naval capabilities.
Which of the above given Statements is/are correct ?
A. 1 and 2 Only B. 2 and 3 Only
C. 2 Only D. 1, 2 and 3
Explanation : A
● Recently, India gifted the Corvette INS Kirpan to Vietnam to enhance that country’s naval
capabilities.
INS Kirpan:
● It is an indigenously-built in-service missile.
● It is a Khukri class missile corvette commissioned into the Navy on January 12, 1991.
● It has a displacement displacing capacity of close to 1,400 tonnes.
● It is capable of a speed of more than 25 knots.
● The Khukri class is equipped with Diesel Engines assembled in India.
● The ship is fitted with a medium-range gun, 30 mm close-range guns, chaff launchers, and
surface-to-surface missiles.
● It performs a wide variety of roles, including coastal and offshore patrol, coastal security,
surface warfare, anti-piracy, and Humanitarian Assistance and Disaster Relief (HADR)
operations.

334. Consider the following statements regarding Ludwigia peruviana which have been in the news
recently.
1. It is an invasive weed threatening the elephant habitats and foraging areas in Valparai,
Tamil Nadu.
2. It is native to Central and South America.
3. It grows in swamps and there is little scope to use machinery which may further destroy
the ecosystem.
Which of the above given Statements is/are correct ?

AMIGOS IAS 198


A. 1 and 2 Only B. 2 and 3 Only
C. 1 and 3 Only D. 1,2 and 3

Explanation : D
● Recently, wildlife scientists said that Ludwigia Peruviana , an invasive weed threatening
the elephant habitats and foraging areas in Valparai, Tamil Nadu.
Ludwigia Peruviana:
● It is popularly called primrose willow, Ludwigia Peruviana, is a native of Central and South
America.
● Its flower is pale yellowish in colour and the plant grows to a height of about 12 feet.
● It is an aquatic plant, which is now challenging the existence of local vegetation in various
swampy areas around the world.
● The rapid large-scale spread of the weed — which was probably introduced as an
ornamental plant for its tiny yellow flowers
● It grows faster in wetlands than other harmful weeds.
● The pre-monsoon temperature and monsoon rains help this weed grow faster.
● It has shaken the balance of these perennial foraging grounds, limiting the growth of grass
and native plants that are palatable to elephants and other animals including gaur.
● It is among the 22 priority invasive plants in Tamil Nadu.
● Unlike other invasive plants, Ludwigia poses a unique challenge as it grows in swamps and
there is little scope to use machinery which may further destroy the ecosystem.

335. Consider the following statements :


Statement-I :
Private sector lender ICICI Bank recently said the bank is ready to move to an expected credit loss
(ECL) framework for provisioning.
Statement-II :
Under this practice, a bank is required to estimate expected credit losses based on forward-
looking estimations rather than wait for credit losses to be actually incurred before making
corresponding loss provisions.

Which one of the following is correct in respect of the above statements?


A. Both Statement-I and Statement-II are correct and Statement-II is the correct explanation
for Statement-I
B. Both Statement-I and Statement-II are correct and Statement-II is not the correct
explanation for Statement-I
C. Statement-I is correct but Statement-II is incorrect
D. Statement-I is incorrect but Statement-II is correct

AMIGOS IAS 199


Explanation: A
What is a loan-loss provision?
● The RBI defines a loan loss provision as an expense that banks set aside for defaulted
loans.
● Banks set aside a portion of the expected loan repayments from all loans in their portfolio
to cover the losses either completely or partially.
● In the event of a loss, instead of taking a loss in its cash flows, the bank can use its loan
loss reserves to cover the loss.
● The level of loan loss provision is determined based on the level expected to protect the
safety and soundness of the bank.
Expected Credit Loss (ECL) regime?
● Under this practice, a bank is required to estimate expected credit losses based on
forward-looking estimations rather than wait for credit losses to be actually incurred
before making corresponding loss provisions.
● As per the proposed framework, banks will need to classify financial assets (primarily
loans) as Stage 1, 2, or 3, depending on their credit risk profile, with Stage 2 and 3 loans
having higher provisions based on the historical credit loss patterns observed by banks.
● Thus, through ECL, banks can estimate the forward-looking probability of default for each
loan, and then by multiplying that probability by the likely loss given default, the bank
gets the percentage loss that is expected to occur if the borrower defaults.

336. Consider the following statements regarding the Global report on Food crises (GRFC) 2023.
1. It is an annual report to achieve a consensus-based assessment of acute food insecurity
in the world
2. It is released recently by the Food and Agriculture Organisation (FAO)
3. The report reveals that the number of people suffering from acute food insecurity has
decreased

How many of the above given Statements is/are correct ?


A. 1 and 2 Only B. 2 and 3 Only
C. 1 and 3 Only D. 1,2 and 3
Explanation : C
● The Global Report on Food Crises (GRFC) for 2023 reveals that the number of people
suffering from acute food insecurity has increased, with over 258 million people across
58 countries and territories facing acute hunger.
● The Global Report on Food Crisis is an annual report to achieve a consensus-based
assessment of acute food insecurity in the world
● It is released by the Food Security Information Network (FSIN) and Global Network
against Food Crises (GNAFC)
What is FSIN?

AMIGOS IAS 200


● A global initiative co-sponsored by Food and Agriculture Organization (FAO), World Food
Programme (WFP), and International Food Policy Research Institute (IFPRI) to strengthen
food and nutrition security information systems for producing reliable and accurate data
to guide analysis and decision-making.

What is GNAFC?
● An alliance of humanitarian and development actors founded by the European Union,
FAO, and WFP in 2016 to prevent, prepare for and respond to food crises and support the
Sustainable Development Goal to End Hunger (SDG 2)

337. According to the recent announcement by the Union Minister of State (Independent Charge)
Science & Technology, which of the following States is going to pioneer India's first Cannabis
Medicine Project ?
A. Karnataka B. Jammu and Kashmir
C. West Bengal D. Uttarakhand
Explanation : B
● Recently, Union Minister of State (Independent Charge) Science & Technology said that
Jammu is going to pioneer India's first Cannabis Medicine Project.

Cannabis Medicine Project:


● ‘Cannabis Research Project’ of CSIR-IIIM Jammu is a first of its kind in India initiated under
the leadership of Science & Technology Ministry in Private Public Partnership with a
Canadian firm, which has a great potential to put substance of abuse for the good of
mankind especially for patients suffering from neuropathies, cancer and epilepsy,
malignancies.
● This project of CSIR-IIIM is also important from the perspective of Atma- Nirbhar Bharat
as it will be able to produce export quality drugs meant for different kinds of
neuropathies, diabetic pains etc.
● This project will explore the therapeutic properties of Cannabis, a plant which is otherwise
prohibited and known for abuse.

338. Consider the following statements regarding the Software-Defined Radio Tactical (SDR-Tac).
1. It is built indigenously by the Navy’s Weapons and Electronics Systems Engineering
Establishment
2. It is intended to serve ship-to-ship, ship-to-shore, and ship-to-air voice-data
communication for network-centric operations.
3. The indigenous SDR-Tac provides real-time voice, data and video information.

Which of the above given Statements is/are correct ?

AMIGOS IAS 201


A. 1 and 2 Only B. 2 and 3 Only
C. 1 and 3 Only D. 1,2 and 3
Explanation : D
● Recently, the Indian Navy has begun equipping its warships with 'Made in India' Software-
Defined Radio.

Software-Defined Radio Tactical (SDR-Tac):


● Equipped with several multimedia capabilities, the indigenous SDR-Tac provides real-time
voice, data and video information.
● It is built indigenously by the Navy’s Weapons and Electronics Systems Engineering
Establishment (WESEE).

What does SDR-Tac do?


● SDR-Tac is a four-channel multi-mode, multi-band, 19-inch rack-mountable, ship-borne
software-defined radio system.
● It is intended to serve ship-to-ship, ship-to-shore, and ship-to-air voice-data
communication for network-centric operations.
● It supports the simultaneous operation of all four channels covering V/UHF- and L-Band.
● This SDR system houses multiple types of waveforms for narrow-band and wide-band
applications.

339. Consider the following statements regarding India Climate Energy Dashboard (ICED) 3.0.
1. It is India's one-stop platform for real-time data on the energy sector, and related
economic datasets based on government published sources.
2. It was developed by NITI Aayog in collaboration with the energy and climate think-tank
Vasudha Foundation.
Which of the above given Statements is/are correct ?
A. 1 Only B. 2 Only
C. Both 1 and 2 D. Neither 1 nor 2

Explanation : C

India Climate Energy Dashboard (ICED) 3.0:


● It is the country’s one-stop platform for near real-time data on the energy sector, climate,
and related economic datasets based on government published sources.
● It was developed by NITI Aayog in collaboration with the energy and climate think-tank
Vasudha Foundation.

AMIGOS IAS 202


● Developed as a user-friendly platform, ICED 3.0 enables users to freely access and
analyses datasets using an analytical engine.
● It will facilitate insights and enhance understanding about the energy and climate sectors
while identifying the key challenges.

340. Consider the following statements :


Statement-I :
Researchers have successfully developed a technology called hygroelectricity.

Statement-II :
Hygroelectricity is the generation of electricity from the Temperature difference of two water
bodies .
Which one of the following is correct in respect of the above statements?
A. Both Statement-I and Statement-II are correct and Statement-II is the correct explanation
for Statement-I
B. Both Statement-I and Statement-II are correct and Statement-II is not the correct
explanation for Statement-I
C. Statement-I is correct but Statement-II is incorrect
D. Statement-I is incorrect but Statement-II is correct
Explanation : C
Hygroelectricity:
● Hygroelectricity is the generation of electricity from the humidity of the air.
● It is a type of renewable energy that has the potential to be a major source of power in
the future.

How it works?
● The key to harvesting electricity from humid air lies in a tiny device comprising two
electrodes and a thin layer of material filled with nanopores.
● These nanopores, each less than 100 nanometres in diameter, allow water molecules
from the air to pass through the device.
● As these molecules move from an upper chamber to a lower chamber, they interact with
the edges of the nanopores, leading to a buildup of electric charge imbalances between
the chambers.
● This process effectively transforms the device into a miniature battery, generating
continuous electricity.

AMIGOS IAS 203


341. Consider the following statements regarding Bura Chapori Wildlife Sanctuary.
1. It is located on the southern bank of river Ganga
2. It comprises of alluvial grassland, riparian and semi-evergreen forests
3. It includes the highly endangered Bengal Florican
Which of the above given Statements is/are correct ?
A. 1 and 2 Only B. 3 Only
C. 2 Only D. 2 and 3 Only

Explanation : D
Bura Chapori Wildlife Sanctuary:
● It is located on the southern bank of river Brahmaputra in the Sonitpur district, Assam.
● It is a part of the Laokhowa-Bura Chapori ecosystem.
● It comprises a mosaic of wet alluvial grassland, riparian and semi-evergreen forests dotted
by wetland and river systems.
● The grassland is also rich in various kinds of medicinal herbs and plants.
● It is home to the Great Indian one-horned rhinoceros, tiger, leopard, wild buffalo, hog
deer, wild pig, and elephants.
● It includes the highly endangered Bengal Florican.
● It also makes it an ideal breeding place for several species of migratory birds during the
winter season.

342. Picolinic acid has been in the news recently. Which of the following statements is/are correct
regarding Picolinic acid.
1. It is an intermediate in the metabolism of tryptophan
2. It has a role as a MALDI matrix material and a human metabolite.
3. It is a conjugate acid of a picolinate.
4. It is not effective against enveloped viruses
How many of the above given Statements is/are correct ?
A. Only one B. Only two
C. Only three D. All four
Explanation : C
Recently, a new study by researchers at the Indian Institute of Science (IISc.) and
collaborators has revealed that picolinic acid, a natural compound produced by
mammalian cells, can block several viruses, including those responsible for SARS-CoV-2
and influenza A.
Picolinic acid:

AMIGOS IAS 204


● It is a pyridinemonocarboxylic acid in which the carboxy group is located at position
● It is an intermediate in the metabolism of tryptophan.
● It has a role as a MALDI matrix material and a human metabolite.
● It is a conjugate acid of a picolinate.
● It is effective against a variety of enveloped viruses, including flaviviruses like the Zika
virus and the Japanese encephalitis virus.
● When the compound was tested in SARS-CoV-2 and influenza animal models, it was found
to protect the animals from infection.
● It was also found to reduce viral load in the lungs when given to infected animals.
● In addition, the researchers found that picolinic acid led to an increase in the number of
immune cells in the animals.

343. Consider the following statements regarding Sovereign Green Bonds(SGBs).


1. They are bonds issued by any sovereign entity, inter-governmental groups or alliances
and corporations.
2. These are issued with a longer maturity period.
3. In India, there is no framework for the sovereign green bonds.
Which of the above given Statements is/are correct ?
A. 1 and 2 Only B. 2 Only
C. 1 and 3 Only D. 1,2 and 3
Explanation : A
Recently, the Chairman of Pension Fund Regulatory and Development Authority (PFRDA)
said the regulator is keen on pension funds investing in sovereign green bonds (SGBs).
Sovereign Green Bonds:
● These are bonds issued by any sovereign entity, inter-governmental groups or alliances
and corporates
● The proceeds of the bonds are utilized for projects classified as environmentally
sustainable.
● These are issued with a longer maturity period.
● In India, the framework for the sovereign green bond was issued by the government on
November 9, 2022.

344. Consider the following statements regarding Commission on Genetic Resources for Food and
Agriculture (CGRFA).
1. It is the only permanent intergovernmental body that deals with all components of
biodiversity for food and agriculture
2. India is not a member of this commission.
3. It was established in the year 1983

AMIGOS IAS 205


Which of the above given Statements is/are correct ?
A. 1 and 2 Only B. 1 and 3 Only
C. 1,2 and 3 D. 3 Only
Explanation : B

CGRFA:
● It is the only permanent intergovernmental body that deals with all components of
biodiversity for food and agriculture.
● It was established in 1983.
● To reach international consensus on policies for the sustainable use and conservation of
genetic resources for food and agriculture and the fair and equitable sharing of benefits
derived from their use
● It has 179 countries as its members.
● India is also a member of this commission
● The intergovernmental body guides the preparation of periodic global assessments of the
status and trends of genetic resources and biodiversity for food and agriculture.
● It also develops global plans of action, codes of conduct or other policy instruments and
monitors their implementation.
● Some of the Commission’s landmark achievements include: the International Treaty on
Plant Genetic Resources for Food and Agriculture (ITPGRFA).

345. Consider the following statements regarding C295 Transport Aircraft.


1. It is a new-generation tactical airlifter that replaces the Avro aircraft of IAF.
2. Short take-off/land from semi-prepared surfaces is one of its features.
3. The aircraft, with a flight endurance of up to 5 days can carry out multi-role operations

Which of the above given Statements is/are correct ?


A. 1 and 2 Only B. 2 and 3 Only
C. 1 and 3 Only D. 1,2 and 3
Explanation : A
• Senior Airbus officials recently said that a fully operational factory will be in place in Vadodara
in Gujarat by November 2024, from where the first Indian-made C295 military transport
aircraft will start rolling out 2026 onward.
C295 Transport Aircraft:
● It is a new-generation tactical airlifter in the light and medium segment that will replace
the ageing Avro aircraft of the Indian Air Force.
● It was designed and built by Airbus, a European multinational aerospace corporation.

AMIGOS IAS 206


● It is robust and reliable but also highly versatile in terms of the number of different
missions it can perform.
● The aircraft, with a flight endurance of up to 11 hours, can carry out multi-role operations
under all weather conditions.
● It has a rear ramp door for quick reaction and para-dropping of troops and cargo.
● Short take-off/land from semi-prepared surfaces is another of its features.

346. According to a New Delhi-based think tank Global Trade Research Initiative (GTRI)'s recent report
The Foreign Subsidies Regulation (FSR) is likely to hit India's exports. FSR is a regulation of ?
A. World Trade Organisation B. European Union
C. United States of America D. International Monetary Fund
Explanation : B
• The Foreign Subsidies Regulation (FSR) of the European Union (EU) is likely to hit India's
exports, the Global Trade Research Initiative (GTRI), a New Delhi-based think tank, recently
said in a report.
Foreign Subsidies Regulation (FSR):
● The European Union’s FSR entered into force on 12 January 2023.
● Its aim is to combat distortions of competition on the EU internal market caused by
foreign subsidies.
● It will allow the EU to investigate financial contributions granted by non-EU governments
to companies active in the EU, and prohibit the award of public contracts to a company
that has unduly profited from foreign subsidies.
347. Consider the following statements regarding India-US Strategic Clean Energy Partnership(SCEP).
1. The SCEP was launched in accordance with the US - India Climate and Clean Energy
Agenda 2030 Partnership
2. The SCEP advances energy security and innovation with greater emphasis on
electrification and decarbonization of processes and end uses
Which of the above given Statements is/are incorrect?
A. 1 Only B. 2 Only
C. Both 1 and 2 D. Neither 1 nor 2
Explanation : D
• The Ministerial meeting of the US-India Strategic Clean Energy Partnership (SCEP) was
held between the Indian Minister of Petroleum and Natural Gas and the US Energy
Secretary in New Delhi.
Strategic Clean Energy Partnership
● The SCEP was launched in accordance with the US - India Climate and Clean Energy
Agenda 2030 Partnership announced by both countries at the Leaders’ Summit on Climate
held in 2021.

AMIGOS IAS 207


● The SCEP advances energy security and innovation with greater emphasis on
electrification and decarbonization of processes and end uses; scaling up emerging clean
energy technologies; finding solutions for hard-to-decarbonize sectors; and deploying
technical solutions.
● The SCEP was earlier established as the Strategic Energy Partnership in 2018 and had
replaced the U.S.-India Energy Dialogue, the previous intergovernmental engagement for
energy cooperation.

348. Consider the following statements regarding the “Festival of Libraries 2023”.
1. The Festival is being organised by the Union Ministry of Education.
2. The festival will initiate a conversation on the modernization and digitization of libraries
in India.
3. This festival is a part of the 'Azadi ka Amrit Mahotsav'.
Which of the above given Statements is/are correct?
A. 1 and 2 Only B. 3 Only
C. 2 and 3 Only D. 2 Only
Explanation : C
● President of India Smt. Droupadi Murmu will inaugurate the unique ‘Festival of Libraries
2023’, a two-day event to be held on 5th - 6th August 2023.
● The Festival is being organised by the Union Ministry of Culture.
● This festival is a part of the second phase of the 'Azadi ka Amrit Mahotsav' and is in line
with the Prime Minister's vision to foster the development and digitization of libraries and
cultivate a culture of reading in India.

349. Consider the following statements regarding Thirujnana Sambandar.


1. He was a Vaishnavite poet-saint from Tamil Nadu who lived in the 7th century CE.
2. Sambandar's works are part of the first three volumes of the Tirumurai.
3. Information about Thirugnana sambandar mainly comes from Periya Puranam.
Which of the above given Statements is/are correct ?
A. 1 and 2 Only B. 2 and 3 Only
C. 1 and 3 Only D. 1,2 and 3
Explanation : B
• Prime Minister Narendra Modi has thanked USA for returning 105 trafficked antiquities which
include a 14-15 Century idol of Shaiva Poet-Saint Thirujnana Sambandar.
● Sambandar, also known as Tirugnana Sambandar, was a Shaiva poet-saint from Tamil
Nadu who lived in the 7th century CE.
● He was a child prodigy and lived for only 16 years but composed an impressive oeuvre of
16,000 hymns in complex meters

AMIGOS IAS 208


● Among his compositions, 383 hymns with 4,181 stanzas have survived, expressing intense
devotion to the Hindu god Shiva.
● Sambandar's works are part of the first three volumes of the Tirumurai, contributing to
the philosophical foundation of Shaiva Siddhanta.
● He is considered one of the prominent Nayanars, Tamil Shaiva bhakti saints, and was a
contemporary of another Shaiva poet-saint, Appar.
● Information about Thirugnanasambandar mainly comes from Periya Puranam.

350. “Digital Time Vouchers” have been in the news recently. They are for ?
A. Farmers B. Tourists
C. Small scale industries D. Political Parties
Explanation : D
● Recently, the Election Commission of India introduced digital time vouchers for political
parties' campaigning on Doordarshan & All India Radio during elections.
● Time Voucher Denominations: Political parties receive 5-minute and 10-minute digital
time vouchers for telecasts on Doordarshan and broadcasts on AIR.
● Parties can choose representatives, with a maximum of 20 minutes per representative of
either Doordarshan or AIR.

351. Consider the following statements regarding External Commercial Borrowings (ECBs).
1. ECBs refer to the borrowing by Indian companies from foreign sources.
2. ECB is subject to regulatory oversight by the RBI
3. In order to access ECB,Companies need not meet any eligibility criteria
Which of the above given Statements is/are correct ?
A. 1 and 2 and B. 2 and 3 Only
C. 1 and 3 Only D. 1,2 and 3
Explanation : A
The Reserve Bank of India (RBI) recently said that agreements for external commercial
borrowings (ECBs) signed by Corporate India jumped to a massive $12 billion in the April-
June quarter of 2023, three times the level in the year-ago period.
External Commercial Borrowings (ECBs):
● ECBs refer to the borrowing of funds by Indian companies from foreign sources in the
form of loans, bonds, or other financial instruments.
● It can be used to finance a variety of purposes, including the expansion of business, the
acquisition of assets, and the repayment of existing debt.
● ECB can be obtained from a variety of sources, including foreign banks, international
financial institutions, and foreign subsidiaries of Indian companies.

AMIGOS IAS 209


● ECB can be in the form of rupee-denominated loans, which are repaid in Indian rupees,
or foreign currency-denominated loans, which are repaid in a foreign currency.
● ECB is subject to regulatory oversight by the RBI, which sets limits on the amount of ECB
that Indian companies can obtain and the purposes for which it can be used.
● Companies must also meet certain eligibility criteria in order to access ECB, such as
minimum credit ratings and debt-equity ratios.

352. Consider the following statements regarding Bay of Bengal Initiative for Multi-Sectoral Technical
and Economic Cooperation (BIMSTEC).
1. It came into being in 1997 through the Bangkok Declaration.
2. It constitutes five member states from South Asia and two member states from Southeast
Asia
3. Permanent Secretariat of BIMSTEC is operational in New Delhi
Which of the above given Statements is/are incorrect ?
A. 2 Only B. 1 Only
C. 3 Only D. 1 and 2 Only
Explanation: C
The first-ever Foreign Ministers’ meeting of the Bay of Bengal Initiative for Multi-Sectoral
Technical and Economic Cooperation (BIMSTEC) recently began in Bangkok.
BIMSTEC:
● It is a regional organization comprising seven Member States lying in the littoral and
adjacent areas of the Bay of Bengal, constituting a contiguous regional unity.
● It came into being on 6 June 1997 through the Bangkok Declaration.
● It constitutes seven Member States: five deriving from South Asia, including Bangladesh,
Bhutan, India, Nepal, Sri Lanka, and two from Southeast Asia, including Myanmar and
Thailand.
● Permanent Secretariat of BIMSTEC has been operational since September 2014 in Dhaka.

353. Consider the following statements with reference to the National Multidimensional Poverty
Index.
1. It is prepared based on the latest National Family Health Survey of 2019-21
2. Totally 12 parameters of health, education and standard of living are examined in the
report.
3. Government of India recently released the first edition of the National Multidimensional
Poverty Index (MPI)
Which of the above given Statements is/are correct ?
A. 1 Only B. 3 Only
C. 2 and 3 Only D. 1 and 2 Only

AMIGOS IAS 210


Explanation : D
According to the ‘National Multidimensional Poverty Index: A Progress Review 2023, India
has registered a significant decline of 9.89 percentage points in the number of
multidimensionally poor, from 24.85% in 2015-16 to 14.96% in 2019-2021.

National Multidimensional Poverty Index:


● It is prepared based on the latest National Family Health Survey of 2019-21 and is the
second edition of the National Multidimensional Poverty Index (MPI).
● Totally 12 parameters of health, education and standard of living are examined in the
report.
● These include nutrition, child and adolescent mortality, maternal health, years of
schooling, school attendance, cooking fuel, sanitation, drinking water, electricity,
housing, assets, and bank accounts.
● The report follows the Alkire-Foster methodology developed by its technical partners --
the Oxford Poverty and Human Development Initiative (OPHI) and United Nations
Development Programme (UNDP).
354. Consider the following statements regarding Bhoomi Samman Award 2023.
1. The Award is an initiative of the Union government, which emphasizes the significance of
digital technologies in agricultural development
2. 68 District Collectors and nine State Secretaries received this award recently under the
Digital India initiative.
Which of the above given Statements is/are correct ?
A. 1 Only B. 2 Only
C. Both 1 and 2 D. Neither 1 nor 2
Explanation : B
What is the Bhoomi Samman Award?
● The Bhoomi Samman Award is a prestigious recognition presented by the Indian
government to honor officials and individuals for their outstanding contributions towards
achieving 100% digitization of land records.
● The award recognizes efforts that directly benefit citizens by providing easy access to
land-related information, reducing disputes and facilitating timely and accurate property
documentation.

355. Recently, Bura Chapori Wildlife Sanctuary has been in the news. It is located in the states of ?
A. Meghalaya B. Assam
C. Manipur D. Arunachal Pradesh
Explanation : B
Bura Chapori Wildlife Sanctuary

AMIGOS IAS 211


● Recently, a woman was tragically shot dead, and six others, including three forest guards,
were injured in Assam's Bura Chapori Wildlife Sanctuary.
● Bura Chapori Wildlife Sanctuary is situated in the northeastern state of Assam, India.
● The sanctuary is located on the northern bank of the Brahmaputra River and covers an
area of approximately 44 square kilometers.
● The sanctuary is particularly recognized for its significant bird population, with over 220
species of birds recorded, including several migratory species.
● Bura Chapori is home to diverse mammalian species, including the Gangetic dolphin,
water buffalo, capped langur, Bengal fox and various species of deer.

356. Consider the following statements:

Statement-I:
Synthetic biology researchers and companies around the world are harnessing the power of
nature to solve problems in medicine, manufacturing and agriculture.
Statement-II:
Synthetic Biology is a field of science that involves redesigning organisms for useful purposes by
engineering them to have new abilities.

Which one of the following is correct in respect of the above statements?


A. Both Statement-I and Statement-II are correct and Statement-II is the correct explanation
for Statement-I
B. Both Statement-I and Statement-II are correct and Statement-II is not the correct
explanation for Statement-I
C. Statement-I is correct but Statement-II is incorrect
D. Statement-I is incorrect but Statement-II is correct

Explanation : A
• A new field of science has been emerging called synthetic biology.

What is Synthetic Biology?


● Synthetic biology is a field of science that involves redesigning organisms for useful
purposes by engineering them to have new abilities.
● Synthetic biology researchers and companies around the world are harnessing the power
of nature to solve problems in medicine, manufacturing and agriculture.
● Researchers attempted to understand how a synthetic life-form would adapt or evolve
over time, especially in situations where the raw materials required to do so could be
limited, forcing the genome to die or adapt through evolution.

AMIGOS IAS 212


○ The researchers found that the synthetic bacteria that had evolved through 300 days
could significantly out-compete the non-evolved minimal version of the organism.

357. Consider the following statements regarding Export Preparedness Index (EPI) 2022.
1. It is a comprehensive tool that assesses the export preparedness of the States and UTs in
India
2. The report is annually released by the Director General of Foreign Trade (DGFT)
3. The report also examines the performance of different sectors in terms of exports.
Which of the above given Statements is/are correct ?
A. 1 and 2 Only B. 2 and 3 Only
C. 1 and 3 Only D. 1,2 and 3
Explanation : C
The 3rd edition of the Export Preparedness Index (EPI) for States/UTs of India for the year
2022 was released by NITI Aayog recently.
Export Preparedness Index (EPI)
● The Export Preparedness Index (EPI) is a comprehensive tool that assesses the export
preparedness of the States and UTs in India. Exports are crucial for stimulating economic
growth and development in a country, which requires understanding the factors that
influence export performance.
● The index conducts a comprehensive analysis of States and UTs across export-related
parameters to identify their strengths and weaknesses.
● The methodology for the index is an evolving process that constantly incorporates
stakeholder feedback.

358. Consider the following statements regarding the “India – Indonesia Economic and Financial
Dialogue” unveiled recently.
1. The Dialogue will enhance collaboration on financial matters by bringing together
economic policymakers and financial regulators from both countries.
2. Macroeconomic challenges and field of fintech for financial inclusion are also part of this
Dialogue
Which of the above given Statements is/are correct ?
A. 1 only B. 2 only
C. Both 1 and 2 D. Neither 1 nor 2

Explanation : C
• India – Indonesia Economic and Financial Dialogue(EFD):

AMIGOS IAS 213


1. Joint collaboration: The EFD Dialogue will enhance collaboration on bilateral and
international economic and financial matters by bringing together economic
policymakers and financial regulators from both countries.
2. Areas of cooperation: Macroeconomic challenges and global economic prospects,
bilateral investment relations, field of fintech for financial inclusion and cooperation in
G20 and ASEAN matters.

359. Recently, RBI and the Central Bank of which of the following countries agreed to establish a
framework for enabling the use of the two countries’ local currencies for cross-border
transactions ?
A. Malaysia B. Singapore
C. UAE D. Saudi Arabia

Explanation : C
● The Reserve Bank of India (RBI) and the Central Bank of the UAE agreed on July 15 to
establish a framework for enabling the use of the two countries’ local currencies for cross-
border transactions.
● The move towards permitting payments in the Indian rupee and the UAE dirham is aimed
at promoting their use bilaterally, thus reducing the dependence on a third country’s
currency such as the U.S. dollar as an intermediary for settling transactions.

360. Consider the following statements regarding Solomon Islands.


1. It is situated in the Northeast Pacific Ocean
2. More than 90% of the islanders are ethnic Melanesians.
3. Pijin is the official language of Solomon Islands
Which of the above given Statements is/are correct?
A. 1 only B. 2 and 3 Only
C. 2 Only D. 1 and 2 Only
Explanation : C
• Recently, the leader of Solomon Islands on Monday hit back at criticism of his nation's
deepening security ties with China, saying the United States and Australia had nothing to
fear.
Solomon Islands:
● It is an island country consisting of six major islands and over 900 smaller islands in
Oceania.
● It is situated in the southwest Pacific Ocean, approximately 2,000 km to the northeast of
Australia.
● Its capital, Honiara, is located on the largest island, Guadalcanal.
● The terrain is mountainous and heavily forested.

AMIGOS IAS 214


● More than 90% of the islanders are ethnic Melanesians.
● Once a British protectorate, the Solomon Islands achieved independence as a republic in
1978.
● English is the official language, but Pijin is the common language for the majority of
people.

361. Consider the following statements regarding Production Linked Incentive Scheme 2.0.
1. The scheme proposes a financial incentive to boost domestic manufacturing
2. For the calculation of net incremental sales of manufactured goods, the base year will be
the financial year 2019-20.
3. The target segments under PLI 2.0 Scheme shall include Laptops, Tablets, All-in-One PCs
etc.
Which of the above given Statements is/are correct ?
A. 1 and 2 Only B. 2 and 3 Only
C. 1 and 3 Only D. 1,2 and 3

Explanation : C
• Recently, the Operational Guidelines of Production Linked Incentive Scheme 2.0 for IT
Hardware has been finalized.

Production Linked Incentive Scheme 2.0:


● The scheme proposes a financial incentive to boost domestic manufacturing and attract
large investments in the value chain.
● The target segments under PLI 2.0 Scheme shall include Laptops, Tablets, All-in-One PCs
and Servers and Ultra Small Form Factor.
● The incentives provided under the PLI 2.0 Scheme will be applicable for a period of 6 years
● For the calculation of net incremental sales of manufactured goods, the base year will be
the financial year 2022-23.

362. Consider the following statements regarding Namda Art.


1. It is said to have begun in the 16th century under Mughal Emperor Humayun
2. Namda is a type of traditional Kashmiri felted carpet that is created using sheep wool
3. The distinct feature of this craft is that wool is felted and not woven.
Which of the above given Statements is/are incorrect?
A. 1 Only B. 1 and 3 Only

AMIGOS IAS 215


C. 2 and 3 Only D. 2 Only
Explanation : C
• Recently, the Union Minister of State for Skill Development and Entrepreneurship flagged
off the first batch of Namda Art products for export to the United Kingdom (UK).
What is Namda Art ?
● It is said to have begun in the 16th century when Mughal Emperor Akbar wanted to get a
covering for his horses to protect them from the cold.
● It was introduced by a Sufi saint named Shah-e-Hamdan to Kashmiris.
● Namda is a type of traditional Kashmiri felted carpet that is created using sheep wool and
has colourful hand embroidery.
● The distinct feature of this Kashmiri craft is that wool is felted and not woven.

363. Consider the following statements regarding Coal gasification.


1. It is the process of producing syngas
2. It a mixture of carbon monoxide (CO), hydrogen (H2), carbon dioxide (CO2), methane
(CH4), and water vapour (H2O)
3. Coal gasification aggravates the local pollution problems
Which of the above given Statements is/are correct ?
A. 1 and 2 Only B. 2 and 3 Only
C. 1 and 3 Only D. 1,2 and 3
Explanation : A
• The Ministry of Coal has set a target to gasify 100 million tonnes of coal by FY 2030 in line
with its energy transition plans.
Coal gasification:
● It is the process of producing syngas, a mixture consisting primarily of carbon monoxide
(CO), hydrogen (H2), carbon dioxide (CO2), methane (CH4), and water vapour (H2O) -
from coal and water, air and/or oxygen.
● During gasification, coal is blown with oxygen and steam while also being heated under
high pressure.
● In this reaction, oxygen and water molecules oxidize the coal and produce syngas.
● Coal gasification can help address local pollution problems.
● It is considered a cleaner option compared to the burning of coal.

364. Consider the following statements regarding the Valmiki Tiger Reserve (VTR).
1. It is located at the India-Nepal border on the bank of river Gandak.
2. It forms the easternmost limit of the Himalayan Terai forests
3. It is the only tiger reserve in Bihar.

AMIGOS IAS 216


4. The vegetation type in this National Park include mainly Moist mixed deciduous, Open-
land vegetation
How many of the above given Statements is/are incorrect?
A. Only one B. Only two
C. Only three D. All four
Explanation : D
Following instructions from the National Tiger Conservation Authority (NTCA), VTR
recently intensified patrolling, especially in areas located along the Nepal border.
Valmiki Tiger Reserve (VTR):
● It is located at the India-Nepal border in the West Champaran district of Bihar on the bank
of river Gandak.
● It is situated in the Gangetic Plains bio-geographic zone of the country.
● It forms the easternmost limit of the Himalayan Terai forests in India and is the only tiger
reserve in Bihar.
● The vegetation types include mainly Moist mixed deciduous, Open-land vegetation, Sub-
mountainous semi-evergreen formation, Freshwater swamps, Riparian fringes, Alluvial
grasslands, high hill savannah and Wetlands.

365. Consider the following statements regarding the Comprehensive and Progressive Agreement for
Trans-Pacific Partnership (CPTPP).
1. It is a free trade agreement (FTA) amongst 11 nations including India and Japan
2. All countries of CPTPP are members of the Asia-Pacific Economic Cooperation (APEC).
3. It was signed in the year 2018.
Which of the above given statements is/are correct ?
A. 1 and 2 Only B. 2 and 3 Only
C. 1 and 3 Only D. 1,2 and 3
Explanation : B
The UK has formally agreed to become the first European country to join the
Comprehensive and Progressive Agreement for Trans-Pacific Partnership (CPTPP).
Comprehensive and Progressive Agreement for Trans-Pacific Partnership (CPTPP):
● It is a free trade agreement (FTA) between 11 nations: Australia, Brunei, Canada, Chile,
Japan, Malaysia, Mexico, New Zealand, Peru, Singapore and Vietnam.
● It was signed in March 2018.
● It succeeded the Trans-Pacific Partnership (TPP)after the United States withdrew from the
TPP in 2017.
● All 11 countries of CPTPP are members of the Asia-Pacific Economic Cooperation (APEC).
○ Asia-Pacific Economic Cooperation (APEC) is a forum of 21 Asia-Pacific economies
established in 1989.

AMIGOS IAS 217


366. The Authorities recently informed that the water from Umiam Dam may be released any time in
case there is further rise in water level in the reservoir above the safe level. Umiam Dam is
located in ?
A. West Bengal B. Jammu and Kashmir
C. Rajasthan D. Meghalaya
Explanation : D
● The Umiam Dam, also known as the Barapani Dam, is a large reservoir located in the state
of Meghalaya in northeastern India.
● It is situated on the Umiam River, about 15 kilometres north of Shillong, the capital city
of Meghalaya.
● The primary purpose of the Umiam Dam is to provide hydroelectric power generation and
supply drinking water to the region. It serves as a major water source for Shillong and its
surrounding areas.

367. Consider the following statements regarding the EXERCISE NOMADIC ELEPHANT-23.
1. It is an annual training event with Malaysia which is conducted alternatively in Malaysia
and India
2. The primary theme of the exercise will focus on counter-terrorism operations in
mountainous terrain
Which of the above given Statements is/are correct ?
A. 1 Only B. 2 Only
C. Both 1 and 2 D. Neither 1 nor 2

Explanation : B
Exercise Nomadic Elephant:

● It is the 15th edition of bilateral joint military exercise.


● It is an annual training event with Mongolia which is conducted alternatively in Mongolia
and India
● The primary theme of the exercise will focus on counter-terrorism operations in
mountainous terrain under United Nations mandate.
● The scope of this exercise involves Platoon level Field Training Exercise (FTX).

368. Consider the following statements regarding Hoolock Gibbon.


1. It is the only ape found in India.
2. It is endemic to India in South Asia.
3. Only Eastern Hoolock Gibbon is found in India

AMIGOS IAS 218


Which of the above given Statements is/are correct ?
A. 1 Only B. 2 and 3 Only
C. 3 Only D. 1,2 and 3
Explanation : A
The conservation status of India’s only ape was a cause for concern at a global event on
Hoolock gibbons held recently in China.

Hoolock Gibbon:
● It is the only ape found in India.
● It is native to eastern Bangladesh, Northeast India, Myanmar, and Southwest China.
● Gibbons, the smallest and fastest of all apes, live in tropical and subtropical forests in the
southeastern part of Asia.
● The Hoolock gibbon, unique to India’s northeast, is one of 20 species of gibbons on Earth.
● It is categorised into Western Hoolock Gibbon and Eastern Hoolock Gibbon.
● Western Hoolock Gibbon has a much wider range, as it is found in all the states of the
north-east, restricted between the south of the Brahmaputra river and east of the Dibang
river.
● Eastern Hoolock gibbon inhabits specific pockets of Arunachal Pradesh and Assam in
India, and southern China and north-east Myanmar.

369. The newly notified Ramgarh Vishdhari Tiger Reserve (RVTR) witnessed the birth of cubs for the
first time, a year after a tigress was relocated to the park. The National park is found in ?
A. Chattisgarh B. Jharkhand
C. Rajasthan D. Haryana
Explanation : C
Ramgarh Vishdhari Tiger Reserve:
● It lies in the south-eastern part of Rajasthan in Bundi district with representation of both
Vindhyan and Aravalli elements.
● It is in continuation with the buffer area of Ranthambore tiger reserve in the North-
eastern side and the Mukundara Hills tiger reserve in the southern side.
● The Mez River (a tributary of Chambal River) passes through this tiger reserve.
● The terrain is mostly rugged and hilly.
370. Consider the following statements regarding the presence of Cicadas in India.

1. The genetic diversity of cicadas in India and Bangladesh ranks the highest in the world.
2. Most cicadas are canopy dwellers and are found in natural forests with large trees.
3. Annual cicadas emerge from the ground at different times each winter

AMIGOS IAS 219


Which of the above given Statements is/are correct?
A. 1 and 2 Only B. 2 and 3 Only
C. 2 Only D. 1 Only
Explanation : A
Recently, a ‘foreign’ cicada that is commonly found in several parts of South India has
assumed an Indian identity.
Cicada:
● Cicadas are hemipteran insects known for their loud, complex and species-specific
acoustic signals or songs.
● The generic diversity of cicadas in India and Bangladesh ranks the highest in the world,
followed by China.
● Most cicadas are canopy dwellers and are found in natural forests with large trees.
● The insect species that has now been named as Purana cheeveeda (after its Malayalam
name Cheeveedu) used to be mistaken for Purana tigrina, a species that was first
described in Malaysia in 1850.
● cheeveeda could extend across the tropical evergreen forests ranging from Goa to
Kanyakumari.
● Annual cicadas emerge from the ground at different times each summer.

371. Consider the following statements regarding Electric vehicles (EVs) ecosystem in India:
1. Hybrid Electric Vehicles (HEVs) combine a conventional ICE system with an electric
propulsion system
2. Battery Electric Vehicles (BEVs) have no ICE or fuel tank, and run on a fully electric
drivetrain powered by rechargeable batteries
3. PHEVs (Plug-in hybrid vehicles) have both internal combustion engines as well as
an electric motor.
How many of the above given Statements is/are incorrect?
A. Only one B. Only two
C. All three D. None
Explanation : D
4 Types of EVs:
● HEVs (Hybrid EVs): They combine a conventional ICE system with an electric
propulsion system, resulting in a hybrid drivetrain that substantially lowers fuel
usage. Example, Toyota Hyryder in India.
● PHEVs (Plug-in hybrid vehicles): They have a hybrid drivetrain that uses both an
ICE and electric power for motive power, backed by rechargeable batteries that
can be plugged into a power source. Example, Chevrolet Volt.

AMIGOS IAS 220


● BEVs: They have no ICE or fuel tank, and run on a fully electric drivetrain powered
by rechargeable batteries. Example, Tata Nexon in India, or the Nissan Leaf and
Tesla Model S.
● FCVs (Fuel cell vehicles): FCVs combine hydrogen and oxygen to produce
electricity, which runs the motor, and the only residue of the chemical process is
water. Example, Toyota’s Mirai and Honda’s Clarity.

372. Consider the following statements regarding “Geocoding ”.


1. It converts an address or description of a location into geographic coordinates.
2. It has been introduced to ensure the accuracy of address details in GSTN records
3. The functionality is available only for SEZ units and SEZ developers
Which of the above given Statements is/are correct ?
A. 1 and 2 Only B. 2 and 3 Only
C. 1 and 3 Only D. 1,2 and 3
Explanation : A
● There are concerns over fake registrations and fraudulent availment of input tax
credit under the Goods and Services Tax (GST) regime. Therefore recently, the
Goods and Services Tax Network (GSTN) made the geocoding functionality live for
all states and union territories.
Geocoding
● It converts an address or description of a location into geographic coordinates.
● It has been introduced to ensure the accuracy of address details in GSTN records
and streamline the address location and verification process.
● GST Network has already geocoded 1.8 crore principal places of businesses.
● The functionality is available for normal, composition, SEZ units, SEZ developers,
input service distributor and casual taxpayers who are active, cancelled, and
suspended.
● This is a one-time activity and once submitted, revision in the address is not
allowed.
● The functionality will not be visible to the taxpayers who have already geocoded
their address through new registration or core amendment.

373. Which of the following statements is/are correct regarding the “Convention on Cluster
Munitions”.
1. The Convention is a binding international law
2. India is not a signatory to this convention.

AMIGOS IAS 221


3. The convention prohibits all use, stockpiling, production and transfer of cluster
munitions.
Choose the incorrect answer from the below given codes:
A. 1 and 2 Only B. 2 and 3 Only
C. 3 Only D. None of them
Explanation : D
• Recently, the decision by the United States to supply Ukraine with cluster bombs has
caused concern among key US allies.
Convention on Cluster Munitions
● It was adopted in Dublin on 30 May 2008 and opened for signature in Oslo on 3
December the same year.
● It prohibits all use, stockpiling, production and transfer of cluster munitions.
● Separate articles in the Convention concern destruction of stockpiles, clearance of
contaminated areas, assistance to victims, submission of transparency reports,
and adoption of domestic legislation.
● The Convention became binding international law when it entered into force on 1
August 2010.
● Till date a total of 123 States have joined the Convention – 111 States Parties and
12 Signatories.
● India is not a signatory to this convention.

374. Consider the following statements regarding Performance Grading Index for Districts (PGI-D).
1. It assesses the performance of the school education system at the district level by
creating an index for comprehensive analysis.
2. The Highest achievable Grade in PGI-D is Akanshi-3, given for Districts scoring
more than 90% of the total points in that category or overall.
Which of the above given Statements is/are incorrect?
A. 1 Only B. 2 Only
C. Both 1 and 2 D. Neither 1 nor 2
Explanation : B
● Recently, the Department of School Education and Literacy (DoSE&L), Ministry of
Education released the Performance Grading Index for Districts (PGI-D) combined
report for 2020-21 & 2021-22.
Performance Grading Index for Districts (PGI-D):
● It assesses the performance of school education system at the district level by
creating an index for comprehensive analysis.

AMIGOS IAS 222


● Based on the success of State PGI, 83-indicator based PGI for District (PGI-D) has
been designed to grade the performance of all districts in school education.
● The data is filled by districts through online portal.
● PGI-D grades the districts into ten grades viz., Highest achievable Grade is Daksh,
which is for Districts scoring more than 90% of the total points in that category or
overall.
● The lowest grade in PGI-D is called Akanshi-3 which is for scores upto10% of the
total points. Ultimate objective of PGI-D is to help the districts to priorities areas
for intervention in school education and thus improve to reach the highest grade.

375. Recently, the draft Detailed Project Report of the Raxaul-Kathmandu cross-border railway project
has been handed over to Nepal. The project is being implemented by ?
A. Mumbai Railway Vikas Corporation
B. Konkan Railway Corporation Limited
C. IRCON International Ltd
D. Indian Railway Finance Corporation
Explanation : B
● The draft Detailed Project Report of the Raxaul-Kathmandu cross-border railway
project has been handed over to Nepal.
● After incorporating suggestions from Nepal, India will submit the final DPR to the
Nepal government.
● India’s Konkan Railway Corporation Limited has been doing the DPR of the railway
project which will connect Kathmandu with the Indian border town of Raxaul as
per the memorandum of understanding signed between Nepal and India on
October 8, 2021.

376. The Global Forest Watch report has been published by ?


A. Forests for Future B. World Resources Institute
C. United Nations Forum on Forests D. Conservation International
Explanation : B
● Recently, a new research was established by the World Resources Institute’s (WRI)
Global Forest Watch.
● Tropical areas lost 4.1 million hectares of forest cover. This is equivalent to losing
an area of 11 football fields per minute.
○ This forest loss produced 2.7 billion tonnes of carbon dioxide emissions, which is
around the same as India’s annual emissions due to the combustion of fossil fuels.
● The total global tree cover loss in 2022 declined by 10%.

AMIGOS IAS 223


○ This includes primary, secondary, and planted forests. This decrease, according to
Global Forest Watch, is a direct result of a decrease in fire-related forest losses
which decreased 28% from 2021.
377. Recently, the quality of water in the Ganga has improved due to the feeding habits(feed on meat
and waste products thrown in the river) of which of the following species ?
A. Finfish B. Octopus
C. Turtles D. Clams
Explanation : C
● The quality of water in the Ganga has improved, and definitely turtles have a role
as they feed on meat and waste products thrown in the river.
● Under a joint aegis of the Namami Gange Programme, the Forest and Wildlife
Department, and the Wildlife Institute of India (WII), turtles hatched at one of
India’s first breeding and rehabilitation centres for turtles situated in Varanasi, will
improve the cleanliness of the Ganga.
● The river is polluted by the disposal of half-burnt dead bodies, rotten meat and
garlands of flowers that are thrown into it.

378. Consider the following statements regarding the Indian Space Research Organisation’s (ISRO's)
LVM3.
1. The second stage of this vehicle uses a Cryogenic engine
2. LVM3 uses two S200 solid rocket boosters to provide the huge amount of thrust
required for lift off.
3. LVM3 will be capable of placing the 4 tonne class satellites into Geosynchronous
Transfer Orbits.
How many of the above given Statements is/are correct?
A. 1 and 2 Only B. 2 Only
C. 3 Only D. 2 and 3 Only
Explanation : D

LVM3
● LVM3 is the new heavy lift launch vehicle of ISRO for achieving a 4000 kg
spacecraft launching capability to GTO (Geosynchronous Transfer Orbit) in a cost
effective manner.
● LVM3 is a three stage launch vehicle consisting of two solid propellant S200 strap-
ons and core stages consisting of L110 liquid stage, C25 cryogenic stage, the
equipment bay (EB) and the Encapsulated assembly (EA).
● The C25 is powered by CE-20, India's largest cryogenic engine, designed and
developed by the Liquid Propulsion Systems Centre.

AMIGOS IAS 224


● The powerful cryogenic stage of LVM3 enables it to place heavy payloads into Low
Earth Orbits of 600 km altitude.
● LVM3 will be capable of placing the 4 tonne class satellites of the GSAT series into
Geosynchronous Transfer Orbits.

379. Takeshima islands is a conflicting territory between which of the following Nations ?
A. Japan and North Korea B. Japan and South Korea
C. Japan and Philippines D. Japan and China
Explanation : B
● Recently, Japan lodged a protest with South Korea over military drills it conducted
on disputed Takeshima islands
● The island is situated in the middle of the Sea of Japan.
● It is lying almost equidistant between the Korean Peninsula and Japan.
● It is called Dokdo in South Korea and Takeshima in Japan.

380. Recently, SCORES Platform has been in the news. It is a platform developed by the ?
A. Securities and Exchange Board of India B. Reserve Bank of India
C. NITI Aayog D. Ministry of MSME
Explanation : A

SCORES Platform (SEBI Complaints Redress System (SCORES) Platform)


● SCORES is a web based centralized grievance redress system of SEBI that was
launched in June 2011.
● SCORES enables investors to lodge and follow up their complaints and track the
status of redressal of such complaints online from the above website from
anywhere.
● This enables the market intermediaries and listed companies to receive the
complaints online from investors, redress such complaints and report redressal
online.
● All the activities starting from lodging of a complaint till its closure by SEBI would
be online in an automated environment and the complainant can view the status
of his complaint online.

381. Consider the following statements regarding “European Sky Shield Initiative’ (ESSI)”.
1. It is a project to build a ground-based integrated European air defense system

AMIGOS IAS 225


2. All European states are participating in this initiative.
3. Anti-ballistic missile capability is not put to use in this initiative.

Which of the above given Statements is/are correct?


A. 1 and 2 Only B. 3 Only
C. 2 and 3 Only D. 1 Only

Explanation : D
European Sky Shield Initiative (ESSI)
● European Sky Shield Initiative (ESSI) is a project to build a ground-based integrated
European air defense system which includes anti-ballistic missile capability.
● As of July 2023, nineteen European states participate in the initiative.
● The initiative aims to create a European air and missile defence system through the
common acquisition of air defence equipment and missiles by European nations to
strengthen NATO’s Integrated Air and Missile Defence (IAMD).

382. Recently, a study has indicated that Tropical Ocean waters are turning Green in colour due to
alteration in ?
A. Phytoplankton communities.
B. Green sea Turtle communities
C. Sea lion communities
D. Green sea crab communities
Explanation : A
● Climate change has altered the colour of 56 per cent of the world’s oceans, according to
a new study.
● The waters in the tropics have turned green, the study published in Nature stated. The
southern Indian Ocean, in particular, has seen a significant colour change.
● Green-coloured water indicates life, especially phytoplankton, which are microscopic
plant-like organisms
● “Changes in the ocean colour indicate alternations to the phytoplankton communities —
since phytoplankton are essential for most life in the ocean as the base of the marine food
web.

383. Consider the following statements regarding the National Research Foundation (NRF).
1. It is being established based on the recommendation of National Education Policy (NEP)
2020
2. The NRF will be presided over by the Union Minister for Science and Technology

AMIGOS IAS 226


3. NRF will be an additional body besides the Science and Engineering Research Board (SERB)
having separate role from the latter
How many of the above given Statements is/are correct ?
A. Only one B. Only two
C. All three D. None
Explanation: A
● The Union Cabinet approved the National Research Foundation (NRF) Bill 2023, which will
establish NRF as an apex body to provide high-level strategic direction to scientific
research in the country.

National Research Foundation(NRF)


● The National Research Foundation(NRF) was one of the key recommendations of the
National Education Policy 2020.
● The Science and Engineering Research Board (SERB) created by an act of Parliament in
2008, will stand repealed and subsumed into the NRF.
● The primary aim of NRF is to foster a culture of research and innovation throughout
India’s universities, colleges, research institutions, and R&D laboratories.

384. Consider the following statements regarding “Kui Language” that has been in the news recently.
1. It is a South-Eastern Dravidian language spoken by the Kandha community.
2. It is primarily spoken in the state of Chattisgarh.
3. Kui is traditionally written using the Devanagari script
Which of the above given Statements is/are correct ?
A. 1 and 2 Only B. 3 Only
C. 2 and 3 Only D. 1 Only

Explanation: D
● Odisha Cabinet gave nod to Inclusion of the Kui Language in 8th Schedule of Constitution.
● The Inclusion of the kui language is going to benefit the Kandha tribe in the State.

Kui Language:
● Kui (also known as Kandh, Khondi, Khond, Khondo), is a South-Eastern Dravidian language
spoken by the Kandha community.
● It is primarily spoken in the state of Odisha.
● It is closely related to other languages in the Dravidian family, such as Gondi and Kuvi. It
was also referred to as the Kuinga language during the historical period.

AMIGOS IAS 227


● With 941,988 registered native speakers, it figures at rank 29 in the 1991 Indian census.
● Kui is traditionally written using the Odia script, which is also used for writing the Odia
language.

385. A new study of the sediments from a seasonal lake in the Kaas Plateau has indicated a major shift
in the Indian Summer Monsoons towards dry and stressed conditions with low rainfall during the
Early-Mid–Holocene, around 8664 years BP. Kaas Plateau is located in ?
A. West Bengal B. Maharashtra
C. Madhya Pradesh D. Chattisgarh
Explanation : B
Kaas Plateau
● Kaas Plateau, nestled in the Western Ghats is about 140 km from Pune, was included in
the UNESCO World Natural Heritage Site in 2012.
● Known as Kaas Pathar in Marathi, its name is derived from the Kaasa tree, botanically
known as Elaeocarpus glandulosus (rudraksha family).
● Designated as a biodiversity hotspot, the Kaas Plateau comes to life with various seasonal
flowers forming a floral carpet over the entire lateritic crust during August and
September.
● Control measures have been implemented by the forest authorities to handle the
pressure of nature lovers visiting the Kaas Plateau.

386. Consider the following Pairs regarding Indian Athletes and the sport they are representing
Athlete Athletic Sport
1. Jyothi Yarraji Triple jump
2. Abdulla Aboobacker Hurdles
3. Ajay Kumar Saroj Long jump
4. Tejaswin Shankar Men's 1500m

How many of the above given pairs is/are matched correctly?


A. Only one B. Only two
C. All four D. None

Explanation : D
● In the ongoing Asian Athletics Championships in Bangkok, India have so far won three
gold and three bronze medals, in Thailand.
● India's Jyothi Yarraji earlier won gold in 100m hurdles, whereas Abdulla Aboobacker won
the gold in the men’s triple jump and Ajay Kumar Saroj in men’s 1500m.

AMIGOS IAS 228


● Tejaswin Shankar won the bronze medal in long jump 400m events and the other two
bronze medalists are Abhishek Pal and Aishwarya Kailash Mishra in men’s 10000m and
women’s 400m, respectively.
● With this India placed third on the medals table behind China and Japan.

387. Consider the following statements regarding Amrit Bharat Station Scheme (ABSS).
1. It envisages development of stations on a continuous basis with a long-term vision.
2. The scheme will subsume all previous redevelopment projects where work is yet to begin.
3. Development of only Minimum Essential Amenities is the objective of this scheme
Which of the above given Statements is/are correct ?
A. 1 Only B. 3 Only
C. 2 and 3 Only D. 1 and 2 Only

Explanation : D

Amrit Bharat Station Scheme (ABSS):


● It envisages development of stations on a continuous basis with a long-term vision.
● The scheme will subsume all previous redevelopment projects where work is yet to begin.
● The scheme aims at preparation of Master Plans of the Railway stations and
implementation of the Master Plan in phases to enhance the facilities including and
beyond the Minimum Essential Amenities.
● However, plans and consequent budgets will only be approved on the basis of factors
such as footfall and inputs from stakeholders.
● Zonal railways have been given the responsibility of selecting stations, which will then be
approved by a committee of senior railway officials.
● The model envisages low-cost redevelopment of stations which can be executed timely.
● The scheme shall cater for introduction of new amenities as well as upgradation and
replacement of existing amenities.

388. Consider the following statements regarding Indo-Pacific Economic Framework (IPEF).
1. It was launched in Tokyo, in May 2023
2. IPEF is a traditional trade agreement.
3. Both India and China are members of IPEF
4. India is also part of the Trade pillar of IPEF
How many of the above given Statements is/are correct ?
A. Only one B. Only two

AMIGOS IAS 229


C. Only three D. All four
Explanation : A
● As per reports, India has asked for better market access in order to be a part of the trade
pillar of the Indo-Pacific Economic Framework for Prosperity (IPEF).
IPEF
● Launched in Tokyo, in May 2023, IPEF aims to strengthen economic engagement among
the member countries to advance growth, peace and prosperity in the region.
● According to an insight paper on IPEF put out by the US Congressional Research Service,
the IPEF is not a traditional trade agreement.
● Rather, it would include different modules(four pillars) covering various aspects.
Four Pillars of IPEF
1. Pillar I - fair and resilient trade,
2. Pillar II - supply chain resilience,
3. Pillar III - infrastructure and decarbonization, and
4. Pillar IV - tax and anticorruption.
● The IPEF has 14 partner countries including Australia, Brunei, Fiji, India, Indonesia, Japan,
Republic of Korea, Malaysia, New Zealand, Philippines, Singapore, Thailand, Vietnam and
the US.
● However, India had decided to remain out of the trade pillar.

389. Consider the following statements regarding the National Bank for Agriculture and Rural
Development (NABARD).
1. NABARD was established on the recommendations of B. Sivaraman Committee in 1982
2. It is an apex regulatory body for overall regulation of regional rural banks and apex
cooperative banks in India.
3. It is under the jurisdiction of the Ministry of Agriculture ,Government of India.

Which of the above given Statements is/are correct?


A. 1 and 2 Only B. 2 and 3 Only
C. 1 and 3 Only D. 1,2 and 3
Explanation : A
NABARD
● NABARD was established on the recommendations of B. Sivaraman Committee in 1982 to
implement the National Bank for Agriculture and Rural Development Act 1981.
● It replaced the Agricultural Credit Department (ACD) and Rural Planning and Credit Cell
(RPCC) of RBI and Agricultural Refinance and Development Corporation (ARDC).
● It is an apex regulatory body for overall regulation of regional rural banks and apex
cooperative banks in India.

AMIGOS IAS 230


● NABARD supervises State Cooperative Banks (StCBs), District Cooperative Central Banks
(DCCBs), and Regional Rural Banks (RRBs) and conducts statutory inspections of these
banks.
● It is under the jurisdiction of the Ministry of Finance ,Government of India.

390. Consider the following statements regarding Generative Artificial Intelligence (AI)
1. Generative AI (GenAI) can create a wide variety of data, such as images, videos, audio,
text, and 3D models.
2. GenAI is capable of producing highly realistic and complex content that mimics human
creativity
3. It does this on its own without learning patterns from existing data.
Which of the above given Statements is/are correct ?
A. 1 and 2 Only B. 2 and 3 Only
C. 1 and 3 Only D. 1,2 and 3
Explanation : A
Google's parent company Alphabet is rolling out its artificial intelligence chatbot Bard in
Europe and Brazil.

What is Generative AI?


● Generative AI (GenAI) is a type of AI that can create a wide variety of data, such as images,
videos, audio, text, and 3D models.
● It does this by learning patterns from existing data, then using this knowledge to generate
new and unique outputs.
● GenAI is capable of producing highly realistic and complex content that mimics human
creativity, making it a valuable tool for many industries such as gaming, entertainment,
and product design.

391. Consider the following statements regarding Japan India Maritime Exercise 2023 (JIMEX 23).
1. This edition marks the 11th edition of JIMEX, since its inception in 2012.
2. Anti - Terror cooperation/drills is the main theme of this exercise.
Which of the above given Statements is/are incorrect ?
A. 1 Only B. 2 Only
C. Both 1 and 2 D. Neither 1 nor 2
Explanation : C
● JIMEX, is a series of maritime exercises conducted by the Indian Navy and the Japan
Maritime Self-Defence Force.
● The seventh edition of the bilateral Japan-India Maritime Exercise 2023 (JIMEX 23) hosted
by the Indian Navy, is being conducted at/ off Visakhapatnam from 05 -10 July 2023.

AMIGOS IAS 231


● This edition marks the 11th anniversary of JIMEX, since its inception in 2012.
● JIMEX-21 aims to develop a common understanding of operational procedures and
enhance inter-operability through the conduct of a multitude of advanced exercises,
across the entire spectrum of maritime operations.
● Multi-faceted tactical exercises involving weapon firings, cross-deck helicopter
operations and complex surface, anti-submarine and air warfare drills will consolidate
coordination developed by the two navies.

392. Consider the following statements with reference to Indian National Space Promotion and
Authorisation Centre(IN-SPACe)
1. IN-SPACe was created in the year 2020
2. It is an autonomous agency under the Department of Science and Technology
3. It is formed to enable facilitate the participation of private players in the space sector of
India
4. It is responsible for establishing new space infrastructure and facilities.

How many of the above given Statements is/are correct?

A. Only one B. Only two


C. Only three D. All four

Explanation : C
● Recently, the Indian National Space Promotion and Authorisation Centre or IN-SPACe,
the commercial arm of Indian Space Research Organisation, ISRO has called for an
Expression of Interest from Indian private Industries to manufacture Small Satellite
Launch Vehicle spares and sub-systems.

Indian National Space Promotion and Authorization Center (IN-SPACe)


● Indian National Space Promotion and Authorisation Centre (IN-SPACe) is a single-window,
independent, nodal agency that functions as an autonomous agency in the Department
of Space (DOS).
● It is formed following the Space sector reforms to enable and facilitate the participation
of private players.
● IN-SPACe is responsible to promote, enable authorize and supervise various space
activities of non-governmental entities including building launch vehicles & satellites and
providing space-based services; sharing space infrastructure and premises under the
control of DOS/ISRO; and establishing of new space infrastructure and facilities

AMIGOS IAS 232


393. Recently, India has started bilateral trade in rupees with which of the following Nations ?
A. Sri Lanka B. Bhutan
C. Afghanistan D. Bangladesh
Explanation : D
● India and Bangladesh have started bilateral trade in rupees. It was launched at an event
jointly organised by Bangladesh Bank and High Commission of India in Dhaka.
● Trade settlement through Indian rupee will only apply to export amounts from
Bangladesh while the rest of imports from India will be settled in US dollars.
● Bangladesh is India's biggest trade partner in South Asia and India is the second largest
trade partner of Bangladesh in Asia.
● The Reserve Bank of India has allowed banks from 18 countries to trade in rupees
including Germany, Russia, Singapore, Sri Lanka, the United Kingdom, Myanmar, Oman
and others; Bangladesh became the 19th country to trade with India in rupees.

394. Consider the following statements regarding “Lokmanya Tilak Award”.


1. This award is presented annually on the occasion of Lokmanya Tilak's Birth anniversary
2. Tessy Thomas popularly known as India's “missile woman” has been chosen as the 41st
recipient of the award.
3. No Business person of India received this award till date.
How many of the above given Statements is/are correct ?
A. Only one B. Only two
C. All three D. None
Explanation : D
● On August 1, the 103rd death anniversary of Lokmanya Tilak, the Tilak Smarak Mandir
Trust (Hind Swaraj Sangh) will confer the prestigious Lokmanya Tilak National Award upon
Prime Minister Narendra Modi.
● This esteemed award is presented annually by the Trust on the occasion of Lokmanya
Tilak's death anniversary.
● Prime Minister Narendra Modi has been chosen as the 41st recipient of this award, in
recognition of his exceptional leadership and his efforts in fostering a sense of patriotism
among citizens.
● In the previous year, the Lokmanya Tilak National Award was conferred upon senior
scientist Tessy Thomas, popularly known as India's “missile woman”
● List of some of the individuals who have been conferred the Loknayak Award:
1. Indira Gandhi 2. Atal Bihari Vajpayee
3. Sharad Pawar 4. Rahul Bajaj
5. Cyrus Poonawalla 6. Manmohan Singh

AMIGOS IAS 233


395. “Tehseen Poonawala case” Judgement has been in the news recently. The judgement lays down
punitive measures to deal with ?
A. Drug Peddling B. Lynching and Mob violence
C. Honour killings D. Caste Discrimination
Explanation : B
● Recently,Supreme Court directed the State Governments to file year-wise data, from
2018, regarding the complaints filed, FIRs registered and challans submitted in courts
concerning incidents of mob violence and lynchings.
● In the Tehseen Poonawala case judgment the Supreme court directed States to form
Special Task Forces (STFs) to collect intelligence on likely incidents of hate speeches, mob
violence and lynchings in districts.
● The judgement had made it clearly the duty of the Central and State Governments to take
steps to curb and stop the dissemination of explosive messages, videos, etc., which have
a “tendency to incite mob violence and lynching of any kind”.

396. The term “Majorana zero modes” has been making headlines in the recent times. It can provide
breakthroughs in ?
A. Carbon sequestration B. Lake conservation
C. Quantum computing D. Habitable exoplanet exploration
Explanation : C
● In a paper published on June 21, researchers at Microsoft announced that they had
figured out a way to create an elusive kind of particle that could potentially revolutionise
quantum computing.
● The claim pertains to particles called Majorana zero modes, whose unique properties
could help build quantum computers that are much less fragile than they are today,
making them computationally superior.
Fermions
● All particles have four quantum numbers associated with them. No two particles in the
same system can have the same four quantum numbers. The numbers are together like
each particle’s ID.
● The characteristic feature of fermions is that one of these numbers, called the quantum
spin, has only half-integer values, like 1/2, 3/2, 5/2, etc.
● Most of the rules that apply to single fermions also apply to these pairs, or bound states.
● When these bound states are their own antiparticles – i.e. if they meet, they annihilate
each other – they are Majorana fermions. Physicists call such bound states Majorana zero
modes, and have been looking for them for at least two decades.
https://www.thehindu.com/sci-tech/science/microsoft-topological-quantum-computer-
majorana-zero-modes/article67053462.ece

AMIGOS IAS 234


397. Consider the following statements regarding the institution Enforcement Directorate (ED).
1. It was established in the year 1976 as an ‘Enforcement Unit’ under the Department of
Home Affairs.
2. Presently, it is under the administrative control of the Department of Expenditure.
3. ED is empowered to undertake, search, seizure, survey but not arrest, and, against the
offences committed under PMLA.
4. The ED Director is appointed by the President on the recommendation of a committee
headed the Prime Minister.
How many of the above given Statements is/are correct ?
A. Only two B. Only three
C. All four D. None
Explanation : D
● Recently, The Supreme Court said that the third extension to Sanjay Kumar Mishra's
tenure as the Enforcement Directorate (ED) chief was illegal and in violation of its
judgment in 2021.
Enforcement Directorate (ED)
● The Enforcement Directorate was established in the year 1956 as an ‘Enforcement Unit’
under the Department of Economic Affairs.
● Later, in 1957, this Unit was renamed as ‘Enforcement Directorate’.
● Presently, it is under the administrative control of the Department of Revenue (under the
Ministry of Finance) for operational purposes.
● ED is responsible for enforcement of the Foreign Exchange Management Act, 1999
(FEMA), and certain provisions under the PMLA.
● ED has the power to attach the asset of the culprits found guilty of the violation of FEMA.
● It has also been empowered to undertake, search, seizure, arrest, and survey, etc. against
the offences committed under PMLA.
● The ED Director is appointed by the central government on the recommendation of a
committee:
○ chaired by the Central Vigilance Commissioner and
○ members comprising of Vigilance Commissioners, Home Secretary, Secretary DOPT and
Revenue Secretary.

398. Which of the following is correct with reference to the concept of “Money Muling” ?
A. It is a process of transferring or moving illlegally acquired money on behalf of someone
else.
B. It is a method in which small sizes of money is kept in several number of bank accounts
to hide the real identity of the real owner
C. It is the process of making large amounts of money generated by criminal activity and
make it appear to have come from a legitimate source.

AMIGOS IAS 235


D. It is a form of barter that involves a company selling "an unused asset to another
company, while at the same time agreeing to buy back the same or similar assets at about
the same price.
Explanation : A
● Young people under the age of 21 account for around one in five cases in the UK where
money muling activity is suspected, according to new figures.
Money Muling:
● A money mule is someone who transfers or moves illegally acquired money on behalf of
someone else.
● Criminals recruit money mules to help launder proceeds derived from online scams and
frauds or crimes like human trafficking and drug trafficking.
● Money mules add layers of distance between crime victims and criminals, which makes it
harder for law enforcement to accurately trace money trails.
● Money mules can move funds in various ways, including through bank accounts, cashier’s
checks, virtual currency, prepaid debit cards, or money service businesses.

399. As per new data released, India became the second country with the highest rejection rate for
the Schengen Visa last year. The Schengen area includes which of the following Nations?
1. Bulgaria 2. Cyprus
3. Ireland 4. Romania
5. Norway 6. Switzerland
Choose the correct answer from the below given codes :
A. 1,2,3 and 5 Only B. 5 and 6 Only
C. 1,2,3 and 4 Only D. 2, 4 and 5 Only
Explanation : B
● Schengen Visa is an official document mandatory for some non-Europeans to travel to all
the 27 countries which are part of the Schengen area.
Schengen Area?
● The Schengen Area, or Schengen Countries Zone, is a group of 27 European nations that
have abolished their internal borders, for the free and unrestricted movement of people.
● Members of this area include: 23 of the 27 EU member states (except for Bulgaria, Cyprus,
Ireland and Romania) and all members of the European Free Trade Association (Iceland,
Liechtenstein, Norway and Switzerland).
● Switzerland, Iceland, and Norway are not in the EU but are inside of the Schengen Area.

400. Consider the following statements regarding Senior Citizen Savings Scheme (SCSS).
1. It's aim is to provide senior citizens in India with a regular income after they attain the
age of 60 years.

AMIGOS IAS 236


2. Retirees in the age bracket of 55-60 years who have opted for Voluntary Retirement
Scheme (VRS) or Superannuation are not eligible under the scheme
3. Under SCSS, the interest amount is paid to the account holders monthly.
Which of the above given Statements is/are correct ?
A. 1 and 2 Only B. 2 and 3 Only
C. 1 Only D. 3 Only
Explanation : C
● The collections under the revamped Senior Citizen Savings Scheme (SCSS) jumped 176%
on year to Rs 55,000 crore in the first quarter of the current financial year.
Senior Citizen Savings Scheme (SCSS):
● SCSS was launched with the main aim of providing senior citizens in India a regular income
after they attain the age of 60 years old.
Who are eligible ?
● Indian citizens above the age of 60 years.
● Retirees in the age bracket of 55-60 years who have opted for Voluntary Retirement
Scheme (VRS) or Superannuation.
● Retired defence personnel above 50 years and below 60 years of age.
● It has a maturity period of five years. But, a depositor can extend one's maturity period
for another three years.

AMIGOS IAS 237

You might also like